Sie sind auf Seite 1von 642

BEHRIA UNIVERSITY ISLAMABAD

Multiple Choices Question


PROGRAM MBBS
SUBJECT Surgery
PROF Final
TOS – ID 17.2.11

Description RECTUM
Rectum is a common site for adenocarcinoma in significant number of middle age
patients, these tumors spread proximally and distally through sub mucosal plexus. What
is preventing its local invasion anteriorly into prostate, bladder and seminal vesicles?

Select the right option.

A Puborectalis muscle
B Fascia transversalis
C Waldyers fascia
D Denowiliers fascia
E Fibrous prostatic capsule

Correct Answer C

Domain: Recall [√] Interpretation [] Problem solving []

Reference(s) Bailey & Love 26th Edition


Prepared by
College Frontier Medical & Dental College, Abbottabad

Name: Prof. Dr. Johar Ali

Signature: –––––––––––––––––––––––––––––––

Date: 2018

Note: Diagrams/Charts/Graphs if any should attach along this


BEHRIA UNIVERSITY ISLAMABAD
Multiple Choices Question
PROGRAM MBBS
SUBJECT Surgery
PROF Final
TOS – ID 17.2.11

Description RECTUM
Stem During abdominoperineal resection for carcinoma rectum at its lower one third a patient
was re-explored for severe fresh bleeding from hollow of sacrum. Multiple transfusions
were given to compensate for losses.

What is the ultimate main source of this haemorrhage?

A Superior mesenteric artery


B Inferior mesenteric artery
C Intrnal iliac artery
D External iliac artery
E Internal pudendal artery

Correct Answer A

Domain: Recall [√] Interpretation [] Problem solving []

Reference(s) Bailey & Love 26th Edition


Prepared by
College Frontier Medical & Dental College, Abbottabad

Name: Prof. Dr. Johar Ali

Signature: –––––––––––––––––––––––––––––––

Date: 2018

Note: Diagrams/Charts/Graphs if any should attach along this


BEHRIA UNIVERSITY ISLAMABAD
Multiple Choices Question
PROGRAM MBBS
SUBJECT Surgery
PROF Final
TOS – ID 17.2.11

Description RECTUM
STEM A young boy was stabbed over the right gluteal region in a domestic fight 8 hours before.
He is complaining of lower abdominal pain and bleeding per rectum. Abdominal
tenderness and digital rectal examination confirms above symptoms. Patient is otherwise
haemodynamically stable.

What will be the best surgical option at this time?

A Fluid resuscitation and observation


B Parenteral antibiotics and blood transfusion
C Laparotomy, excision of the damage rectum and end to end anastomoses

D Laparotomy, rectal wound excision, primary repair with proximal de-functioning


colostomy
E Excision of damaged margins, and Hartmann colostomy

Correct Answer D

Domain: Recall [] Interpretation [] Problem solving [√]

Reference(s) Bailey and Love


Prepared by
College Frontier Medical & Dental College, Abbottabad

Name: Prof. Dr. Johar Ali

Signature: –––––––––––––––––––––––––––––––

Date: 2018

Note: Diagrams/Charts/Graphs if any should attach along this


BEHRIA UNIVERSITY ISLAMABAD
Multiple Choices Question
PROGRAM MBBS
SUBJECT Surgery
PROF Final
TOS – ID 17.2.11

Description RECTUM
STEM A female child was brought to surgical emergency with something coming out of anus
after an episode of acute gastroenteritis. Examiantion shows a weak malnourished and
dehydrated child with pink lump protruding per anum. Copious mucus discharge is
observed without blood staining of diapers. Attempted Digital rectal examiantion by little
finger fail to insinuate all around it.

What is your most likely diagnosis?

A Partial prolapse of rectum


B Complete rectal prolapse
C Iliocolic intusesuption
D Colocolic intusesuption
E Juvenile Rectal polyp

Correct Answer B

Domain: Recall [] Interpretation [√] Problem solving []

Reference(s) Bailey and Love


Prepared by
College Frontier Medical & Dental College, Abbottabad

Name: Prof. Dr. Johar Ali

Signature: –––––––––––––––––––––––––––––––

Date: 2018

Note: Diagrams/Charts/Graphs if any should attach along this


BEHRIA UNIVERSITY ISLAMABAD
Multiple Choices Question
PROGRAM MBBS
SUBJECT Surgery
PROF Final
TOS – ID 17.2.11

Description RECTUM
STEM An elderly male is admitted with biopsy proven moderately differentiated
adenocarcinoma and is being worked out for curative surgery. Tumor is located in middle
1/3 of rectum and mobile on digital rectal examiantion.

Which group of lymph nodes will generally be enlarged by metastasis?

A Superficial inguinal group of lymph nodes


B Internal iliac group of lymph nodes
C Para aortic lymph nodes
D Lymph node in femoral canal (lymph node of cloqut)
E Left supra-clavicularly lymph node

Correct Answer C

Domain: Recall [√] Interpretation [] Problem solving []

Reference(s) Bailey and Love


Prepared by
College Frontier Medical & Dental College, Abbottabad

Name: Prof. Dr. Johar Ali

Signature: –––––––––––––––––––––––––––––––

Date: 2018

Note: Diagrams/Charts/Graphs if any should attach along this


BEHRIA UNIVERSITY ISLAMABAD
Multiple Choices Question
PROGRAM MBBS
SUBJECT Surgery
PROF Final
TOS – ID 17.2.11

Description RECTUM
STEM Upon laparotomy a rectal tumor was found to be bowel wall confined and resectable in
its middle 1/3. Para rectal and along main artery lymph nodes are enlarged due to
metastasis. Tumor is average grade on histopathology.

What will be definitive surgical treatment ?

A High anterior resection with restorative anastomosis


B Low anterior resection and restorative anastomosis
C Abdominoperineal resection followed by adjuvant chemo radiotherapy
D Resection and radiotherapy
E Resection, restorative anastomosis and defunctioning iliostomy

Correct Answer E

Domain: Recall [] Interpretation [] Problem solving [√]

Reference(s) Bailey and love


Prepared by
College Frontier Medical & Dental College, Abbottabad

Name: Prof. Dr. Johar Ali

Signature: –––––––––––––––––––––––––––––––

Date: 2018

Note: Diagrams/Charts/Graphs if any should attach along this


BEHRIA UNIVERSITY ISLAMABAD
Multiple Choices Question
PROGRAM MBBS
SUBJECT Surgery
PROF Final
TOS – ID 17.2.11

Description RECTUM
STEM In colorectal cancers preoperative assessment and bowel preparation is an essential
requisite.

Which of the following option is less likely contributing in assessment?

A CT Scan of chest and liver


B Sigmoidoscopy and biopsy
C Colonoscopy to exclude synchronous cancers
D Magnetic Resonance Imaging and endo sonography
E Plain x-Ray abdomen in erect posture

Correct Answer E

Domain: Recall [] Interpretation [] Problem solving [√]

Reference(s) Bailey & Love


Prepared by
College Frontier Medical & Dental College, Abbottabad

Name: Prof. Dr. Johar Ali

Signature: –––––––––––––––––––––––––––––––

Date: 2018

Note: Diagrams/Charts/Graphs if any should attach along this


BEHRIA UNIVERSITY ISLAMABAD
Multiple Choices Question
PROGRAM MBBS
SUBJECT Surgery
PROF Final
TOS – ID 17.2.11

Description RECTUM
STEM TNM staging is the internationally recognized staging system for cancers. In rectal
carcinoma endoscopic ultrasonography shows T3 wall invasion. What will be it’s dukes
staging while peri operative assessment?

Select the best option

A Duke A stage
B Duke B stage
C Duke C 1 stage
D Duke C 2 stage
E Metastatic stage

Correct Answer A

Domain: Recall [] Interpretation [√] Problem solving []

Reference(s) Bailey and love


Prepared by
College Frontier Medical & Dental College, Abbottabad

Name: Prof. Dr. Johar Ali

Signature: –––––––––––––––––––––––––––––––

Date: 2018

Note: Diagrams/Charts/Graphs if any should attach along this


BEHRIA UNIVERSITY ISLAMABAD
Multiple Choices Question
PROGRAM MBBS
SUBJECT Surgery
PROF Final
TOS – ID 17.2.11

Description RECTUM
STEM A 35 years old male presented with complains of frequency of mucus diarrhea for the last
four months. He is feeling crampy aches and pains all over body. Digital rectal
evaluation reveals feeling of multiple finger like projections in posterior rectal wall I n
its lower 1/3 . His blood HB level is 10 grams per dl with normal blood urea.

What is your most likely diagnosis?

A Acute bacillary dysentery


B Ulcerative colitis
C Chrons disease
D Villous adenomas
E Mucinous adenocarcinoma

Correct Answer D

Domain: Recall [] Interpretation [√] Problem solving []

Reference(s) Bailey & Love


Prepared by
College Frontier Medical & Dental College, Abbottabad

Name: Prof. Dr. Johar Ali

Signature: –––––––––––––––––––––––––––––––

Date: 2018

Note: Diagrams/Charts/Graphs if any should attach along this


BEHRIA UNIVERSITY ISLAMABAD
Multiple Choices Question
PROGRAM MBBS
SUBJECT Surgery
PROF Final
TOS – ID 17.2.11

Description RECTUM
STEM A 60 Years Old Women Complains of bleeding per rectum associated with
defecation. She only pases flatus and mucous. Digital rectal examination is un
remarkable. Rigid sigmoidoscopy shows uniform inlamed looking mucosa.

What is the most likely suspicion?

A Familial adenomatous polyposis


B Acute proctitis
C Defuse rectal cancer
D Juvenile polyps
E Rectal tuberculosis

Correct Answer B

Domain: Recall [] Interpretation [√] Problem solving []

Reference(s) Bailey and love


Prepared by
College Frontier Medical & Dental College, Abbottabad

Name: Prof. Dr. Johar Ali

Signature: –––––––––––––––––––––––––––––––

Date: 2018

Note: Diagrams/Charts/Graphs if any should attach along this


BEHRIA UNIVERSITY ISLAMABAD
Multiple Choices Question
PROGRAM MBBS
SUBJECT Surgery
PROF Final
TOS – ID 17.2.11

Description RECTUM
STEM A 65 year old male presented with complains of early morning mucous diarrhea mixed
with fresh blood. It’s frequency is increased due to distressing incomplete sense of
bowel evacuation. Left iliac fossa palpation show mass which is indentable on pressing
with examiners index finger.

What is your most likely diagnosis?

A Second degree Haemorrhoids


B Annular carcinoma rectum
C Ulcerative type of ectal caner
D Volvulus
E Carcionoma anus

Correct Answer B

Domain: Recall [] Interpretation [√] Problem solving []

Reference(s) Bailey and Love


Prepared by
College Frontier Medical & Dental College, Abbottabad

Name: Prof. Dr. Johar Ali

Signature: –––––––––––––––––––––––––––––––

Date: 2018

Note: Diagrams/Charts/Graphs if any should attach along this


BEHRIA UNIVERSITY ISLAMABAD
Multiple Choices Question
PROGRAM MBBS
SUBJECT SURGERY
PROF Final
TOS – ID 17.2.12

Description ANUS & ANAL CANAL


STEM A 70 year old male presented with bright red painless bleeding per rectum. He has
history of associated altered bowel habits and complains of incomplete emptying of
rectum. He looks pale and has recent weight loss.

What is your diagnosis?

A Diverticulitis
B Amoebic dysentery
C Proctitis
D Rectal cancer
E Haemorrhoids

Correct Answer D

Domain: Recall [] Interpretation [√] Problem solving []

Reference(s) Bailey & Love


Prepared by
College Frontier Medical & Dental College, Abbottabad

Name: Prof. Dr. Johar Ali

Signature: –––––––––––––––––––––––––––––––

Date: 2018

Note: Diagrams/Charts/Graphs if any should attach along this


BEHRIA UNIVERSITY ISLAMABAD
Multiple Choices Question
PROGRAM MBBS
SUBJECT Surgery
PROF Final
TOS – ID 17.2.12

Description ANUS & ANAL CANAL


STEM An expecting mother with full term pregnancy presented in OPD with tendency towards
constipation and fresh bleeding per rectum. She is worried about something protruding
per anum after defecation.

What is your most likely clinical diagnosis?

A Anal cancer
B Rectal cancer
C Haemorrhoids
D Fistula in ano
E Pilonidal sinus

Correct Answer C

Domain: Recall [] Interpretation [√] Problem solving []

Reference(s) Bail & Love


Prepared by
College Frontier Medical & Dental College, Abbottabad

Name: Prof. Dr. Johar Ali

Signature: –––––––––––––––––––––––––––––––

Date: 2018

Note: Diagrams/Charts/Graphs if any should attach along this


BEHRIA UNIVERSITY ISLAMABAD
Multiple Choices Question
PROGRAM MBBS
SUBJECT Surgery
PROF Final
TOS – ID 17.2.12

Description ANUS & ANAL CANAL


STEM A young tailor is referred with severe anal pain associated with defecation. He noticed
bright red blood streak over hardened stools. Pain persists for one to two hours.

What is your diagnosis?

A Second degree haemorrhoids


B Acute anal fissures
C Cancer anal canal
D Perianal fistula
E Ulcerative colitis

Correct Answer B

Domain: Recall [] Interpretation [] Problem solving [√]

Reference(s) Bailey & Love


Prepared by
College Frontier Medical & Dental College, Abbottabad

Name: Prof. Dr. Johar Ali

Signature: –––––––––––––––––––––––––––––––

Date: 2018

Note: Diagrams/Charts/Graphs if any should attach along this


BEHRIA UNIVERSITY ISLAMABAD
Multiple Choices Question
PROGRAM MBBS
SUBJECT Surgery
PROF Final
TOS – ID 17.2.12

Description ANUS AND ANAL CANAL


STEM A 25 years old lady was referred for 6 month history of diarrhea and bleeding per rectum.
She has lost almost 4 kg weight. Her mother also suffered with same problem and
underwent surgery, after that she is living with stoma over right iliac fossa.

What is your diagnosis?

A Carcinoma rectum
B 3rd degree haemorrhoids
C Acute proctitis
D Ulcerative colitis
E Acute Diverticulitis

Correct Answer D

Domain: Recall [] Interpretation [√] Problem solving []

Reference(s) Bailey and Love


Prepared by
College Frontier Medical & Dental College, Abbottabad

Name: Prof. Dr. Johar Ali

Signature: –––––––––––––––––––––––––––––––

Date: 2018

Note: Diagrams/Charts/Graphs if any should attach along this


BEHRIA UNIVERSITY ISLAMABAD
Multiple Choices Question
PROGRAM MBBS
SUBJECT Surgery
PROF Final
TOS – ID 17.2.12

Description ANUS AND ANAL CANAL


STEM A 75 years old elderly male is brought to hospital with recent change in bowel habits and
weight loss with non tender soft abdomen.

What next investigation will help in your diagnosis?

A Proctosocpy
B Anoscopy
C Sigmoidoscopy
D Colonoscopy
E Gasteroscopy

Correct Answer D

Domain: Recall [√] Interpretation [] Problem solving []

Reference(s) BAILEY AND LOVE


Prepared by
College Frontier Medical & Dental College, Abbottabad

Name: Prof. Dr. Johar Ali

Signature: –––––––––––––––––––––––––––––––

Date: 2018

Note: Diagrams/Charts/Graphs if any should attach along this


BEHRIA UNIVERSITY ISLAMABAD
Multiple Choices Question
PROGRAM MBBS
SUBJECT Surgery
PROF Final
TOS – ID 17.2.12

Description ANUS AND ANAL CANAL


STEM A young lady who recently delivered is referred to surgical OPD with complains of
severe painful defecation and haematochasia. Examination reveals an ant sentinel pile.
Digital rectal examiantion was abounded due to painful nature of underlying pathology.

What is your preferred choice of treatment?

A Lateral internal anal sphincterotomy


B Topical application of local anaesthetics
C Topical glycerol trinitrate or diftzium
D Oral diclofenac sodium
E Injecitons of local anaesthetic

Correct Answer A

Domain: Recall [] Interpretation [] Problem solving [√]

Reference(s) Bailey & Love


Prepared by
College Frontier Medical & Dental College, Abbottabad

Name: Prof. Dr. Johar Ali

Signature: –––––––––––––––––––––––––––––––

Date: 2018

Note: Diagrams/Charts/Graphs if any should attach along this


BEHRIA UNIVERSITY ISLAMABAD
Multiple Choices Question
PROGRAM MBBS
SUBJECT Surgery
PROF Final
TOS – ID 17.2.12

Description ANUS AND ANAL CANAL


STEM A 50 years female presented with prolapsing haemorrhoids. She looks pale and short of
breath due to anemia.

What will your treatment of choice

A Stapled haemorrhoidopexy
B Excision and ligation
C Injection sclerotherapy
D Rubber band ligaiton
E Conservative treatment

Correct Answer A

Domain: Recall [] Interpretation [] Problem solving [√]

Reference(s) BAILEY & LOVE


Prepared by
College Frontier Medical & Dental College, Abbottabad

Name: Prof. Dr. Johar Ali

Signature: –––––––––––––––––––––––––––––––

Date: 2018

Note: Diagrams/Charts/Graphs if any should attach along this


BEHRIA UNIVERSITY ISLAMABAD
Multiple Choices Question
PROGRAM MBBS
SUBJECT Surgery
PROF Final
TOS – ID 17.2.12

Description ANUS AND ANAL CANAL


STEM A 45 years gentle man presented with complaints of staining of underclothes due to pus
and blood discharging from perianal area. It starts with painful swelling associated with
fever followed by leaking in few days time. Local examination shows pus discharge via
small opening at 6 o clocks Posteriorly.

How will you treat him?

A Antibiotics and analgesics


B Laying open of track
C Injection of methylene blue
D Insertion of seton
E Octereotide injections

Correct Answer B

Domain: Recall [] Interpretation [] Problem solving []√

Reference(s) Bailey & Love


Prepared by
College Frontier Medical & Dental College, Abbottabad

Name: Prof. Dr. Johar Ali

Signature: –––––––––––––––––––––––––––––––

Date: 2018

Note: Diagrams/Charts/Graphs if any should attach along this


BEHRIA UNIVERSITY ISLAMABAD
Multiple Choices Question
PROGRAM MBBS
SUBJECT Surgery
PROF Final
TOS – ID 17.2.8

Description LARGE GUT OBSTRUCTION -COLON


STEM A young motorist is admitted Intensive Care Unit for paraplegia due to fracture of
thoracic spine at T4 level. He is on ventilatory support. After 4th day of admission nurse
reported doctor on duty about his distention of abdomen. His abdomen is not tense or
guarded but bowel sounds are absent. His Hb is 15 gm/dl, Blood urea 45 mg/dl, S. cretin
is 1-2mg/dl with normal electrolyte levels. What will the best management option?

A Flatus tube insertion


B Nasogastric decompresses
C Laproscopy
D Laprotomy
E I/V drops, suction and intake out put monitoring

Correct Answer e

Domain: Recall [] Interpretation [] Problem solving []√

Reference(s)
Prepared by
College Frontier Medical & Dental College, Abbottabad

Name: Prof. Dr. Johar Ali

Signature: –––––––––––––––––––––––––––––––

Date: 2018

Note: Diagrams/Charts/Graphs if any should attach along this


BEHRIA UNIVERSITY ISLAMABAD
Multiple Choices Question
PROGRAM MBBS
SUBJECT Surgery
PROF Final
TOS – ID 17.2.8

Description SMALL INTESTINAL OBSTRUCTION


STEM A young girl was operated for appendicitis 6 years back she presented with central
abdominal pain, distension and vomiting. She is dehydrated and has rapid pulse with
110/75 mmHg blood pressure. Her bowel sounds are Audible from distance.

What is the underlying pathology of this disease?

A Peritonitis
B Worm infestation
C Trichobazoar/phytosazoars
D Fibrous adhesive and bands
E Paralytic illus

Correct Answer D

Domain: Recall [] Interpretation [√] Problem solving []

Reference(s) Bailey & Love


Prepared by
College Frontier Medical & Dental College, Abbottabad

Name: Prof. Dr. Johar Ali

Signature: –––––––––––––––––––––––––––––––

Date: 2018

Note: Diagrams/Charts/Graphs if any should attach along this


BEHRIA UNIVERSITY ISLAMABAD
Multiple Choices Question
PROGRAM MBBS
SUBJECT Surgery
PROF Final
TOS – ID 17.2.8

Description SMALL INTESTINAL OBSTRUCTION


STEM An elderly patient had swelling over her right groin for the last four years, that was
disappearing on lying down for the last 12 hours. Swelling is red and tender, she I s
complaining of vomiting and abdominal distension.

What is your diagnosis?

A Ascites
B Distension due to electrolytes imbalance
C Paralytic illus due to uremia
D Dynamic intestinal obstruction
E Metastatic colon tumor

Correct Answer D

Domain: Recall [] Interpretation [√] Problem solving []

Reference(s)
Prepared by
College Frontier Medical & Dental College, Abbottabad

Name: Prof. Dr. Johar Ali

Signature: –––––––––––––––––––––––––––––––

Date: 2018

Note: Diagrams/Charts/Graphs if any should attach along this


BEHRIA UNIVERSITY ISLAMABAD
Multiple Choices Question
PROGRAM MBBS
SUBJECT Surgery
PROF Final
TOS – ID 17.2.8

Description LARGE INTESTINAL OBSTRUCTION


STEM An elderly patient presented with constipation and abdominal distension located
peripherally over right lower abdomen. Examination show hyper-resonant percussta note
over right iliac fossa. Barium anemia reveals narrowing at hepatic flexure of colon. Plain
x-ray abdomen show huge gas shadow over same area.

What is your diagnosis?

A Acute appendicitis
B Localized peritonitis
C Closed loop obstruction
D Ulcerative collotis
E Carcinoma ceacum

Correct Answer C

Domain: Recall [] Interpretation [] Problem solving []

Reference(s) Bailey U Love


Prepared by
College Frontier Medical & Dental College, Abbottabad

Name: Prof. Dr. Johar Ali

Signature: –––––––––––––––––––––––––––––––

Date: 2018

Note: Diagrams/Charts/Graphs if any should attach along this


BEHRIA UNIVERSITY ISLAMABAD
Multiple Choices Question
PROGRAM MBBS
SUBJECT Surgery
PROF Final
TOS – ID 17.2.8

Description SMALL INTESTINAL OBSTRUCTION


STEM A neonate of 8 days old is referred to sonologist for abdominal ultrasonography.
Ultrasound Scan showed only distended gas filled loop. Baby had nasogastric tube in
place with intravenous canola inserted peripherally in left Dorsum of hand. There is
history of no passage of meconium and bilious vomiting. X-ray abdomen shows multiple
centrally placed distended Intestinal loops almost occupying whole abdomen. There is no
evidence of free air under diaphragm.

What is your most likely diagnosis?


A Illiocalic Intussusception
B Duodenal atresia
C Illeal atresia
D Mesenteric adenitis
E Entero colitis

Correct Answer C

Domain: Recall [] Interpretation [] Problem solving [√]

Reference(s) Bailey & Love 26th Edition


Prepared by
College Frontier Medical & Dental College, Abbottabad

Name: Prof. Dr. Johar Ali

Signature: –––––––––––––––––––––––––––––––

Date: 2018

Note: Diagrams/Charts/Graphs if any should attach along this


BEHRIA UNIVERSITY ISLAMABAD
Multiple Choices Question
PROGRAM MBBS
SUBJECT Surgery
PROF Final
TOS – ID

Description
STEM A lady was operated for perforated duodenal ulcer six months back she was brought to
hospital with complaints of central abdominal pain, distension and vomiting. Pain is
colicky in nature and come on paroxysms. A sub-acute recurrent small intestinal
obstruction was suspected.

What will be your diagnostic investigation of choice?

A Abdominal ultrasonography
B CT Scan abdomen
C Laproscopy
D Barium meal and fallow through
E Barium enema

Correct Answer D

Domain: Recall [] Interpretation [] Problem solving [√]

Reference(s) Bailey & Love Page 1193 26 Edition


Prepared by
College Frontier Medical & Dental College, Abbottabad

Name: Prof. Dr. Johar Ali

Signature: –––––––––––––––––––––––––––––––

Date: 2018

Note: Diagrams/Charts/Graphs if any should attach along this


BEHRIA UNIVERSITY ISLAMABAD
Multiple Choices Question
PROGRAM MBBS
SUBJECT Surgery
PROF Final
TOS – ID 17.2.8

Description SMALL INTESTINLA OBSTRUCTION


STEM 9 month old bay girl is admitted with vomiting and episodic abdominal resulting in
screaming and folding of legs. Ultrasonography shows “Target Sign” in the central
abdomen.

What conservative method will result in correction of underlying pathology?

A Nasogastric decompression and IV fluid replacement


B Analgesia & antibiotic observation
C Laprotomy
D Abdominal massages with olive oil
E Hydrostatic reduction rectally.

Correct Answer E

Domain: Recall [] Interpretation [] Problem solving [√]

Reference(s) Bailey & Love 26th Edition


Prepared by
College Frontier Medical & Dental College, Abbottabad

Name: Prof. Dr. Johar Ali

Signature: –––––––––––––––––––––––––––––––

Date: 2018

Note: Diagrams/Charts/Graphs if any should attach along this


BEHRIA UNIVERSITY ISLAMABAD
Multiple Choices Question
PROGRAM MBBS
SUBJECT Surgery
PROF Final
TOS – ID 17.2.8

Description SMALL & LARGE INTESTINAL OBSTRUCTION


STEM An elderly man from Kohistan presented with constipation and abdominal distension.
Digital rectal examination reveals no mass growth or feacal matter. X-ray abdomen
shows dialated loops of intestine measuring more than 10 cmm in diameter, mimicking
“Coffe Bean” shape.

What is your diagnosis?

A Small intestinal volvolus


B Ceacal volvulus
C Transvers colon obstruction
D Toxic mega colon
E Sigmoid volvulus

Correct Answer E

Domain: Recall [] Interpretation [√] Problem solving []

Reference(s) Bailey & Love 26th Edition


Prepared by
College Frontier Medical & Dental College, Abbottabad

Name: Prof. Dr. Johar Ali

Signature: –––––––––––––––––––––––––––––––

Date: 2018

Note: Diagrams/Charts/Graphs if any should attach along this


BEHRIA UNIVERSITY ISLAMABAD
Multiple Choices Question
PROGRAM MBBS
SUBJECT Surgery
PROF Final
TOS – ID 17.2.8

Description SMALL & LARGE INTESTINAL OBSTRUCITON


STEM 76 years female is admitted repeatedly for fever with chills and rigors, along with central
abdominal pain distension and vomiting. She gets relived with the use of antibiotic,
analgesic, and I/V Hydration. Plain x-ray shows signs of small intestine obstruction a soft
minal density at right lower quadrant and air in extra hepatic Biliary channel. Her pulse is
110/min and B.P is 150/80 mmHg. Her laboratory data shows 15,000 cmm, WBC, with
polymorph nuclear neutrophilia Blood urea is 60 mg/dl with normal serum critine.

What is your diagnosis?


A Peritonitis due to viscose perforation
B Sub pherenic abcess
C Small intestinal obstruction with Pneumoperitonitis
D Large intestinal obstruction with cholecystocolic fistula
E Small instestinal obstruction with cholecystoduodenal fistula

Correct Answer E

Domain: Recall [] Interpretation [√] Problem solving []

Reference(s) Bailey & Love , Page 1189 26th Eidtion


Prepared by
College Frontier Medical & Dental College, Abbottabad

Name: Prof. Dr. Johar Ali

Signature: –––––––––––––––––––––––––––––––

Date: 2018

Note: Diagrams/Charts/Graphs if any should attach along this


BEHRIA UNIVERSITY ISLAMABAD
Multiple Choices Question
PROGRAM MBBS
SUBJECT Surgery
PROF Final
TOS – ID 17.2.5

Description GALL BLADDER


STEM A known case of gall stone disease patient presented with pain right upper quadrant
radiating to Rt. Shoulder and interscapular region. Deep palpation reveals tenderness
over right upper quadrant. His eyes are normal for ANEMIA and jaundice.

What is most suspected clinical diagnosis?

A Acute mesenteric adenitis


B Acute appendicitis
C Acute peritonitis
D Acute cholecystitis
E Chronic cholecysititis

Correct Answer D

Domain: Recall [] Interpretation [] Problem solving [√]

Reference(s) Bailey & Love Page 1189 26th Edition


Prepared by
College Frontier Medical & Dental College, Abbottabad

Name: Prof. Dr. Johar Ali

Signature: –––––––––––––––––––––––––––––––

Date: 2018

Note: Diagrams/Charts/Graphs if any should attach along this


BEHRIA UNIVERSITY ISLAMABAD
Multiple Choices Question
PROGRAM MBBS
SUBJECT Surgery
PROF Final
TOS – ID 17.2.5

Description GALL BLADDER


STEM A young lady of fertile age had biliary colic intermitted due to small calculous in gall
bladder her liver functions are normal with otherwise no concomitant medical disease.
What will the best management option.

A Laproscopic cholecystectomy
B Conservative treatment with cholic and chemoderxy cholic acids
C Open cholecystectomy
D Observation only
E Endoscopic retrograde cholangiopanaeatography

Correct Answer A

Domain: Recall [] Interpretation [] Problem solving [√]

Reference(s)
Prepared by
College Frontier Medical & Dental College, Abbottabad

Name: Prof. Dr. Johar Ali

Signature: –––––––––––––––––––––––––––––––

Date: 2018

Note: Diagrams/Charts/Graphs if any should attach along this


BEHRIA UNIVERSITY ISLAMABAD
Multiple Choices Question
PROGRAM MBBS
SUBJECT Surgery
PROF Final
TOS – ID 17.2.8

Description Small Intestinal obstruction


STEM An elderly diabetic and hypertensive man is admitted with sever abdominal pain, and
vomiting. Pain is out of proportion that is not responding to any narcotic analgesia. His
blood pressure is 100 mmHg and pulse is 130/min. Abdomen is tender to deep palpation
with absent bowl sounds. Inguinal orificess and digital rectal examiantion findings are un
remarkable. His lab data shows BUN 50 mg/dl, serum critine 1.2mg/dl. HB 12 gm/dl,
serum amylase 100 mg/dl, x-ray chest shows no free air under right dome of diaphragm.
He is on insulin with clopedigrol . Long lead II ECG paper show ST elevation and
bizarre QRS complete.

What is your most likely clinical diagnosis?


A Small intestinal obstruction due to volvolus
B Peritonitis secondary to perforated duodenal ulcer
C Gangrene of sigmoid volvulus
D Mesenteric Ischemia resulting in Ischemic gangrene of small intestine
E Acute Necrotising pancreatitis

Correct Answer D

Domain: Recall [] Interpretation [√] Problem solving []

Reference(s) Bailey & Love 26th Edition


Prepared by
College Frontier Medical & Dental College, Abbottabad

Name: Prof. Dr. Johar Ali

Signature: –––––––––––––––––––––––––––––––

Date: 2018

Note: Diagrams/Charts/Graphs if any should attach along this


BEHRIA UNIVERSITY ISLAMABAD
Multiple Choices Question
PROGRAM MBBS
SUBJECT Surgery
PROF Final
TOS – ID 17.2.1

Description ESOPHAGUS
STEM 60 years old patient repeatedly presents with dyspepsia and heart burn for the last one
year. His symptoms were masked with antacids and proton pump inhibitors for some
time then recurred.

What next investigation you will do?

A 24 hours PH.metery

B Gastroscopy and biopsy

C X-Ray Abdomen

D Helicobacter Pylori test


E Abdominal ultrasonography

Correct Answer A

Domain: Recall [] Interpretation [√] Problem solving []

Reference(s) Bailey & Love 26th Edition


Prepared by
College Frontier Medical & Dental College, Abbottabad

Name: Prof. Dr. Johar Ali

Signature: –––––––––––––––––––––––––––––––

Date: 2018

Note: Diagrams/Charts/Graphs if any should attach along this


BEHRIA UNIVERSITY ISLAMABAD
Multiple Choices Question
PROGRAM MBBS
SUBJECT Surgery
PROF Final
TOS – ID 17.2.2

Description SMALL INTESTINE DOUDENUM


STEM A known case of duodenal ulcer patient presented in emergency department with severe
hematemesis melena and heamdynamic instability. Emergency gasteroendoscopy shows
bleeding posterior duodenal ulcer.

What surgical option to treat this case?

A Laprotomy, doudenostomy with under running of gasteroduodenal arty


B Gastrostomy
C Omentopxy
D Laprotomy and peritoneal toileting
E Laprotomy with Truncal vagotomy

Correct Answer A

Domain: Recall [] Interpretation [√] Problem solving []

Reference(s) Bailey & Love 26th Edition


Prepared by
College Frontier Medical & Dental College, Abbottabad

Name: Prof. Dr. Johar Ali

Signature: –––––––––––––––––––––––––––––––

Date: 2018

Note: Diagrams/Charts/Graphs if any should attach along this


BEHRIA UNIVERSITY ISLAMABAD
Multiple Choices Question
PROGRAM MBBS
SUBJECT Surgery
PROF Final
TOS – ID 17.2.2

Description STOMACH & DOUDENUM


STEM A known case of duodenal ulcer presented with vomiting few minutes after intake of
food. Barium meal shows gastric outlet obstruction gastroscopy and biopsy shows
Benign duodenal stricture.

What will be the best management option?

A Billroth-I operation
B Billroth II operation
C Truncal vagotomy
D Gasterojejunostomy with Truncal vagotomy
E Highly selective gagotomy

Correct Answer D

Domain: Recall [] Interpretation [] Problem solving [√]

Reference(s) Bailey & Love 26th Edtion


Prepared by
College Frontier Medical & Dental College, Abbottabad

Name: Prof. Dr. Johar Ali

Signature: –––––––––––––––––––––––––––––––

Date: 2018

Note: Diagrams/Charts/Graphs if any should attach along this


BEHRIA UNIVERSITY ISLAMABAD
Multiple Choices Question
PROGRAM MBBS
SUBJECT Surgery
PROF Final
TOS – ID 17.2.2

Description STOMACH & DOUDENUM


STEM A young male was brought to surgical emergency in last week of Ramadan with sudden
and severe epigastric pain radiating to whole abdomen. He gives history of frequent use
of antacids for indigestion. His abdomen is guarded and tender on palpation Auscultation
reveals no bowel sounds.

What is your most likely clinical diagnosis?

A Peritonitis due to duodenal perforation


B Gastric perforation
C Acute gastritis
D Gasteroesophageal reflux
E Gastric cancer

Correct Answer A

Domain: Recall [] Interpretation [] Problem solving [√]

Reference(s) Bailey & Love 26th Edition


Prepared by
College Frontier Medical & Dental College, Abbottabad

Name: Prof. Dr. Johar Ali

Signature: –––––––––––––––––––––––––––––––

Date: 2018

Note: Diagrams/Charts/Graphs if any should attach along this


BEHRIA UNIVERSITY ISLAMABAD
Multiple Choices Question
PROGRAM MBBS
SUBJECT Surgery
PROF Final
TOS – ID 17.2.8

Description SMALL INSTINE


STEM A patient with duodenal ulcer perforation is admitted in surgical emergency ward you
advised an x-ray abdomen erect to make a diagnosis.

What radiological sign will be diagnostic of duodenal perforation?

A Fluid collection
B Air fluid levels in small intestine
C Free air under right dome of diaphragm
D Gasteroesophageal reflux
E Gasric cancer

Correct Answer C

Domain: Recall [] Interpretation [√] Problem solving []

Reference(s)
Prepared by
College Frontier Medical & Dental College, Abbottabad

Name: Prof. Dr. Johar Ali

Signature: –––––––––––––––––––––––––––––––

Date: 2018

Note: Diagrams/Charts/Graphs if any should attach along this


BEHRIA UNIVERSITY ISLAMABAD
Multiple Choices Question
PROGRAM MBBS
SUBJECT Surgery
PROF Final
TOS – ID 17.2.1

Description ESOPHGUS
STEM A 50 year old patient complains of chest pain on swallowing. This is usually episodic. He
has had these symptoms for many years by recently they have become more frequent and
severe.

What is your diagnosis?

A Peptic stricture
B Carcinoma
C Achalasia
D Diffuse oesophageal spasm
E Schatazki’s ring

Correct Answer C

Domain: Recall [] Interpretation [√] Problem solving []

Reference(s) Bailey & Love 26th Edition


Prepared by
College Frontier Medical & Dental College, Abbottabad

Name: Prof. Dr. Johar Ali

Signature: –––––––––––––––––––––––––––––––

Date: 2018

Note: Diagrams/Charts/Graphs if any should attach along this


BEHRIA UNIVERSITY ISLAMABAD
Multiple Choices Question
PROGRAM MBBS
SUBJECT Surgery
PROF Final
TOS – ID 17.2.1

Description ESOPHGUS
STEM A 45 years old male patient complains of dysphagia of 2 years’ duration. He seems to
have greater difficulty swallowing liquids than solids. Recently he has noticed that at
night he is woken up by coughing. He has lost about 7-8 kg in weight in 6 months.

What is your most likely diagnosis?

A Esophageal shortening
B Cancer
C Achalasia
D Nuteracker esophagus
E Esophageal web

Correct Answer C

Domain: Recall [] Interpretation [√] Problem solving []

Reference(s) Bailey & Love 26th Edition


Prepared by
College Frontier Medical & Dental College, Abbottabad

Name: Prof. Dr. Johar Ali

Signature: –––––––––––––––––––––––––––––––

Date: 2018

Note: Diagrams/Charts/Graphs if any should attach along this


BEHRIA UNIVERSITY ISLAMABAD
Multiple Choices Question
PROGRAM MBBS
SUBJECT Surgery
PROF Final
TOS – ID 17.2.1

Description ESOPHGUS
STEM A 35 years old female complaints of occasional dysphagia with food sticking in the lower
retrosternal region. She has had occasional heartburn for many years.

Select the right option?

A Ca esophagus
B Benign esophageal stricture
C Chagas disease
D GORD
E Schataxki’s ring

Correct Answer B

Domain: Recall [] Interpretation [] Problem solving []

Reference(s) Bailey & Love 26th Edition


Prepared by
College Frontier Medical & Dental College, Abbottabad

Name: Prof. Dr. Johar Ali

Signature: –––––––––––––––––––––––––––––––

Date: 2018

Note: Diagrams/Charts/Graphs if any should attach along this


BEHRIA UNIVERSITY ISLAMABAD
Multiple Choices Question
PROGRAM MBBS
SUBJECT Surgery
PROF Final
TOS – ID 17.2.1

Description ESOPHAGUS
STEM A 65 year old male, a heavy smoker, complains of dysphagia to solid food of 2 months’
duration. At present he can only take fluids. He has quite severe cough when he tries to
swallow any food or fluids. He feels this different to his ‘smoker’s cough’ which he
suffered for many years. He has lost about 10 kg in weight since the onset of these
symptoms.

Select the right option?

A Hiatus hernia
B Carcinoma Esophagus
C Esophageal diverticulum
D Diffuse oesophageal spasm
E Pharyngeal (Zenker’s) divertuculum

Correct Answer B

Domain: Recall [] Interpretation [√] Problem solving []

Reference(s) Bailey & Love 26th Edition


Prepared by
College Frontier Medical & Dental College, Abbottabad

Name: Prof. Dr. Johar Ali

Signature: –––––––––––––––––––––––––––––––

Date: 2018

Note: Diagrams/Charts/Graphs if any should attach along this


BEHRIA UNIVERSITY ISLAMABAD
Multiple Choices Question
PROGRAM MBBS
SUBJECT Surgery
PROF Final
TOS – ID 17.2.1

Description ESOPHAGUS
STEM A 60 year old male complains of quite a severe cough which occurs when he lied in bed.
At times he is woken up because of incessant cough when food material seems to project
out. He is embarrassed by bad breath. He also has some dysphagia. His doctor has been
treating him for chest infections.

What is your diagnoses?

A Gasgteritis
B Duodenitis
C GORD
D Pulsion diverticulum
E Pharyngeal (Zenker’s) diverticulum

Correct Answer C

Domain: Recall [] Interpretation [√] Problem solving []

Reference(s) Bailey & Love 26th Edition


Prepared by
College Frontier Medical & Dental College, Abbottabad

Name: Prof. Dr. Johar Ali

Signature: –––––––––––––––––––––––––––––––

Date: 2018

Note: Diagrams/Charts/Graphs if any should attach along this


BEHRIA UNIVERSITY ISLAMABAD
Multiple Choices Question
PROGRAM MBBS
SUBJECT Surgery
PROF Final
TOS – ID 17.2.10

Description APPENDIX
STEM 59 year old present with right lower quadrant pain, nausea and vomiting. She undergoes
un-complicated laproscopic appendectomy. Post operatively the pathology reveals a 2.5
cm mucinous adenocarcinoma with lymphatic invasion. Staging workup, including
colonoscopy, chest x-ray and computed tomography scan of the abdomen and pelvis is
negative. Which of the following is the most appropriate next step, in her management?

A No further investigation at this time following every 6 months for 2 years


B Chemotherapy alone
C Neoadjuvant chemotherapy followed by right hemicoloectomy
D Illiocecectomy
E Right hemicoloectomy

Correct Answer E

Domain: Recall [] Interpretation [] Problem solving [√]

Reference(s) Bailey &Love 26th Edition


Prepared by
College Frontier Medical & Dental College, Abbottabad

Name: Prof. Dr. Johar Ali

Signature: –––––––––––––––––––––––––––––––

Date: 2018

Note: Diagrams/Charts/Graphs if any should attach along this


BEHRIA UNIVERSITY ISLAMABAD
Multiple Choices Question
PROGRAM MBBS
SUBJECT Surgery
PROF Final
TOS – ID 17.2.1

Description ESOPHAGUS
STEM A 41 year old man complains of regurgitation of saliva and of undigested food. A barium
swallow reveals a dilated esophagus and bird’s beak deformity. Manometry shows a
hypertensive lower esophageal sphinter with failure to relax with deglutition. Which of
the following is the safest and most effective treatment of this condition?

A Medical treatment with sublingual nitroglycerine, nitrates or calcium channel blockers


B Repeated dilatations with Boogies
C Injection of Bolox in the lower esophageal spinter
D Balloon dilation
E Surgical esophagomyotomy

Correct Answer D

Domain: Recall [] Interpretation [] Problem solving [√]

Reference(s) Bailey & Love 26th Edition


Prepared by
College Frontier Medical & Dental College, Abbottabad

Name: Prof. Dr. Johar Ali

Signature: –––––––––––––––––––––––––––––––

Date: 2018

Note: Diagrams/Charts/Graphs if any should attach along this


BEHRIA UNIVERSITY ISLAMABAD
Multiple Choices Question
PROGRAM MBBS
SUBJECT Surgery
PROF Final
TOS – ID 17.2.10

Description APPENDIX/CARCIONOID
STEM A 32 year old woman undergoes an uncomplicated appendectomy for acute appendicitis.
The pathology report notes the presence of a 1 cm carcinoid tumour in the tip of the
appendix.

Which of the following is the most appropriate management of this patient?

A Right hemicolocectomy
B Right hemicoloectomy and chemotherapy
C Chemotherapy only
D Appendicectomy
E Watch full waiting

Correct Answer D

Domain: Recall [] Interpretation [] Problem solving [√]

Reference(s)
Prepared by
College Frontier Medical & Dental College, Abbottabad

Name: Prof. Dr. Johar Ali

Signature: –––––––––––––––––––––––––––––––

Date: 2018

Note: Diagrams/Charts/Graphs if any should attach along this


BEHRIA UNIVERSITY ISLAMABAD
Multiple Choices Question
PROGRAM MBBS
SUBJECT Surgery
PROF Final
TOS – ID 17.2.1

Description ESOPHAGUS
STEM A 62 year old man has been noticing progressive difficulty swallowing, first solid food
and now liquids as well. A barium study shows a ragged narrowing just below the
CARINAL level. Endosocpic biopsy confirms squamous cell carcinoma.

Which of the following statements concerning carcinoma of the esophagus is true?

A Alcohol has not been implicated as a precipitating factory


B Squamous carcinoma is the most common type at the cardioesophageal junction
C It has a higher incidence in females
D It occurs more commonly
E The standard of care is radiation therapy

Correct Answer B

Domain: Recall [] Interpretation [√] Problem solving []

Reference(s) Bailey & Love 26th Edition


Prepared by
College Frontier Medical & Dental College, Abbottabad

Name: Prof. Dr. Johar Ali

Signature: –––––––––––––––––––––––––––––––

Date: 2018

Note: Diagrams/Charts/Graphs if any should attach along this


BEHRIA UNIVERSITY ISLAMABAD
Multiple Choices Question
PROGRAM MBBS
SUBJECT Surgery
PROF Final
TOS – ID 17.2.10

Description APPENDIX
STEM Classical tenderness and rebound tenderness is seldom seen in which anatomically placed
inflamed appendix

A Subceacal
B Retroceacal
C Pelvic
D Para ceacal
E Preilial

Correct Answer B

Domain: Recall [√] Interpretation [] Problem solving []

Reference(s) Bailey & Love 26th Edition


Prepared by
College Frontier Medical & Dental College, Abbottabad

Name: Prof. Dr. Johar Ali

Signature: –––––––––––––––––––––––––––––––

Date: 2018

Note: Diagrams/Charts/Graphs if any should attach along this


BEHRIA UNIVERSITY ISLAMABAD
Multiple Choices Question
PROGRAM MBBS
SUBJECT Surgery
PROF Final
TOS – ID 17.2.10

Description APPENDIX
STEM A married female of 26 presented with lower urinary symptoms anorexia and vomiting.
Diagnosis of acute pelvic appendicitis is suspected.

What diagnostic sign will be helpful?

A Rebound tenderness
B Abdominal distension
C Diarrheoa
D Pointing sign
E Digital rectal examiantion revealing tenderness at right of pouch of duglous

Correct Answer E

Domain: Recall [] Interpretation [] Problem solving [√]

Reference(s) Bailey & Love 26th Edition


Prepared by
College Frontier Medical & Dental College, Abbottabad

Name: Prof. Dr. Johar Ali

Signature: –––––––––––––––––––––––––––––––

Date: 2018

Note: Diagrams/Charts/Graphs if any should attach along this


BEHRIA UNIVERSITY ISLAMABAD
Multiple Choices Question
PROGRAM MBBS
SUBJECT Surgery
PROF Final
TOS – ID 17.2.10

Description
STEM An elderly patient of 80 years presented with pain right iliac fossa and mass, he is
aneamic and has loss of weight.

What is your most likely clinical diagnosis?

A Chron’s disease
B Acute appendicitis due to ceacal tumor
C Psoas abcess
D Ulcerative colitis
E Acute peptic ulcer

Correct Answer B

Domain: Recall [] Interpretation [] Problem solving [√]

Reference(s) Bailey & Love 26th Edition


Prepared by
College Frontier Medical & Dental College, Abbottabad

Name: Prof. Dr. Johar Ali

Signature: –––––––––––––––––––––––––––––––

Date: 2018

Note: Diagrams/Charts/Graphs if any should attach along this


BEHRIA UNIVERSITY ISLAMABAD
Multiple Choices Question
PROGRAM MBBS
SUBJECT Surgery
PROF Final
TOS – ID 17.2.10

Description APPENDIX
STEM The most common cause of pain abdomen which is confused with acute appendicitis in
children

A Bronchopneumonia
B Gastroenteritis
C Meckeles diverticulum
D Mesenteric lymphadenitis
E Illeocolic Intussusception

Correct Answer D

Domain: Recall [] Interpretation [] Problem solving [√]

Reference(s) Bailey & Love 26th Edition


Prepared by
College Frontier Medical & Dental College, Abbottabad

Name: Prof. Dr. Johar Ali

Signature: –––––––––––––––––––––––––––––––

Date: 2018

Note: Diagrams/Charts/Graphs if any should attach along this


BEHRIA UNIVERSITY ISLAMABAD
Multiple Choices Question
PROGRAM MBBS
SUBJECT Surgery
PROF Final
TOS – ID 17.2.10

Description APPENDIX
STEM A young boy is admitted with history of periumbilical pain that radiated to right
illiac fossa. Deep palpation reveals ill defined tender mass.

What will your best management plan?

A Immediate appendicectomy
B Concervative treatment followed by interval appendicectomy
C Laproscopic appendicectomy
D Ultra sound guided drainage
E Laprotomy

Correct Answer B

Domain: Recall [] Interpretation [] Problem solving [√]

Reference(s) Bailey & Love 26th Edition


Prepared by
College Frontier Medical & Dental College, Abbottabad

Name: Prof. Dr. Johar Ali

Signature: –––––––––––––––––––––––––––––––

Date: 2018

Note: Diagrams/Charts/Graphs if any should attach along this


BEHRIA UNIVERSITY ISLAMABAD
Multiple Choices Question
PROGRAM MBBS
SUBJECT Surgery
PROF Final
TOS – ID 17.2.10

Description APPENDIX
STEM An adult male was operated for perforated appendicitis via grid iron incision four days
back. He is running high grade hectic pyrexia and passes watery stools.

What is your most likely clinical suspicion?

A Gastroenteritis
B Antibiotic induce diarrhea
C Pelvic abcess
D Bacillary dysentery
E Amoebic dysentery

Correct Answer C

Domain: Recall [] Interpretation [√] Problem solving []

Reference(s) Bailey & Love 26th Edition


Prepared by
College Frontier Medical & Dental College, Abbottabad

Name: Prof. Dr. Johar Ali

Signature: –––––––––––––––––––––––––––––––

Date: 2018

Note: Diagrams/Charts/Graphs if any should attach along this


BEHRIA UNIVERSITY ISLAMABAD
Multiple Choices Question
PROGRAM MBBS
SUBJECT Surgery
PROF Final
TOS – ID 17.2.10

Description APPENDIX
STEM Immunocampromised states are well recognized risk factors for perforation of appendix.
Mark the option that is not contributory to early perforation.

A Extremes of age
B Diabetes mellitus
C Faecolith obstruction
D Pelvic appendix
E Gastroenteritis

Correct Answer E

Domain: Recall [] Interpretation [√] Problem solving []

Reference(s) Bailey & Love 26th Edition


Prepared by
College Frontier Medical & Dental College, Abbottabad

Name: Prof. Dr. Johar Ali

Signature: –––––––––––––––––––––––––––––––

Date: 2018

Note: Diagrams/Charts/Graphs if any should attach along this


BEHRIA UNIVERSITY ISLAMABAD
Multiple Choices Question
PROGRAM MBBS
SUBJECT Surgery
PROF Final
TOS – ID 17.2.10

Description APPENDIX
STEM Rebound tenderness is an impressive sign in diagnosing acute appendicitis. Select the
option least contributory in diagnosis of above mentioned pathology.

A Pointing sign
B Rovsing sign
C Psoas sign
D Obturator sign
E Murphy’s sign

Correct Answer E

Domain: Recall [] Interpretation [] Problem solving []√

Reference(s) Bailey & Love 26th Edition


Prepared by
College Frontier Medical & Dental College, Abbottabad

Name: Prof. Dr. Johar Ali

Signature: –––––––––––––––––––––––––––––––

Date: 2018

Note: Diagrams/Charts/Graphs if any should attach along this


BEHRIA UNIVERSITY ISLAMABAD
Multiple Choices Question
PROGRAM MBBS
SUBJECT Surgery
PROF Final
TOS – ID 17.2.10

Description APPENDIX
STEM During surgery verimiform appendix was found to have carcinoid tumor with
illioceacal lymph node involvement .

What is your best management option?

A Appendectomy
B Caecectomy
C Left hemicoloectomy
D Right hemicoloectomy
E Transverse colostomy

Correct Answer D

Domain: Recall [] Interpretation [] Problem solving [√]

Reference(s) Bailey & Love 26th Edition


Prepared by
College Frontier Medical & Dental College, Abbottabad

Name: Prof. Dr. Johar Ali

Signature: –––––––––––––––––––––––––––––––

Date: 2018

Note: Diagrams/Charts/Graphs if any should attach along this


BEHRIA UNIVERSITY ISLAMABAD
Multiple Choices Question
PROGRAM MBBS
SUBJECT Surgery
PROF Final
TOS – ID 17.2.10

Description APPENDIX
STEM A female was operated for perforated appendicitis three days back. She is complaining of
severe throbbing pain over operation site and running high grade fever. On examination
surgical site reveals redness and buried skin stitches due to edema.

What will be your first step in management of this complication?

A Broad spectrum IV antibiotics


B IV normal saline
C Daily change of antiseptic dressing
D Removal of skin stitches and wound toilet
E Oral analgesics only

Correct Answer D

Domain: Recall [] Interpretation [] Problem solving [√]

Reference(s) Bailey & Love 26th Edition


Prepared by
College Frontier Medical & Dental College, Abbottabad

Name: Prof. Dr. Johar Ali

Signature: –––––––––––––––––––––––––––––––

Date: 2018

Note: Diagrams/Charts/Graphs if any should attach along this


BEHRIA UNIVERSITY ISLAMABAD
Multiple Choices Question
PROGRAM MBBS
SUBJECT Surgery
PROF Final
TOS – ID 17.2.10

Description APPENDIX
STEM One day before the mid point of her menstrual cycle , a young healthy lady of 18 years
presented with pain lower abdomen and was tender to deep palpation . Her TLC is
10000/cmm. Digital rectal examination elicits right sided tenderness too. Her temperature
is normal.

What will the most probable diagnosis?

A Pelvic inflammatory diseases


B Acute urinary tract infection
C Acute appendicitis
D Ruptured grafian follicle
E Meckels divertitculitis

Correct Answer D

Domain: Recall [] Interpretation [√] Problem solving []

Reference(s) Bailey & Love 26th Edition


Prepared by
College Frontier Medical & Dental College, Abbottabad

Name: Prof. Dr. Johar Ali

Signature: –––––––––––––––––––––––––––––––

Date: 2018

Note: Diagrams/Charts/Graphs if any should attach along this


BEHRIA UNIVERSITY ISLAMABAD
Multiple Choices Question
PROGRAM MBBS
SUBJECT Surgery
PROF Final
TOS – ID 17.2.10

Description APPENDIX
STEM Appendicitis is the most common abdominal pathology, in which decade of life a patient
is most likely to get it?

A First
B Second
C Third
D Fifth
E Seventh

Correct Answer C

Domain: Recall [] Interpretation [] Problem solving [√]

Reference(s) Bailey & Love 26th Edition


Prepared by
College Frontier Medical & Dental College, Abbottabad

Name: Prof. Dr. Johar Ali

Signature: –––––––––––––––––––––––––––––––

Date: 2018

Note: Diagrams/Charts/Graphs if any should attach along this


BEHRIA UNIVERSITY ISLAMABAD
Multiple Choices Question
PROGRAM MBBS
SUBJECT Surgery
PROF Final
TOS – ID 17.2.10

Description APPENDIX
STEM A middle aged man was found to have a firm yellowish swelling of less than 2cm at the
tip of appendix during open surgery. Lymphnodes around llieoceacal junction were not
palpable.

What will be the best surgical management option?

A Appendicectomy
B Right hemicolocectomy
C Colectomy
D Closure of abdomen and chemotherapy
E Tube colostomy

Correct Answer A

Domain: Recall [] Interpretation [] Problem solving [√]

Reference(s) Bailey & Love 26th Edition


Prepared by
College Frontier Medical & Dental College, Abbottabad

Name: Prof. Dr. Johar Ali

Signature: –––––––––––––––––––––––––––––––

Date: 2018

Note: Diagrams/Charts/Graphs if any should attach along this


BEHRIA UNIVERSITY ISLAMABAD
Multiple Choices Question
PROGRAM MBBS
SUBJECT Surgery
PROF Final
TOS – ID 17.2.10

Description APPENDIX
STEM A middle age lady presented with abdominal distension and pain, exploratory lepratomy
was performed which revealed that whole abdominal cavity is filled with mucous like
fluid. Examination of appendix reveals a distended and perforated appendix at the tip.
Diagnosis of pseudomyxoma peritonea was entertained.

What will the choice of treatment?

A Abdominal closure and observation


B Systemic chemotherapy
C Radiotherapy
D Surgical debulking
E Intraperitoneal chemotherapy

Correct Answer D

Domain: Recall [] Interpretation [] Problem solving [√]

Reference(s) Bailey & Love 26th Edition


Prepared by
College Frontier Medical & Dental College, Abbottabad

Name: Prof. Dr. Johar Ali

Signature: –––––––––––––––––––––––––––––––

Date: 2018

Note: Diagrams/Charts/Graphs if any should attach along this


BEHRIA UNIVERSITY ISLAMABAD
Multiple Choices Question
PROGRAM MBBS
SUBJECT Surgery
PROF Final
TOS – ID 17.2.10

Description PELIC ABCESS


STEM Pelvic abscess after perforated appendix surgery is a recognized complication.

The treatment of choice is

A IV antibiotics
B Oral antibiotics
C Leprotomy
D Radiologically guided percutaneous drainage
E Trans rectal incision and drainage

Correct Answer D

Domain: Recall [] Interpretation [] Problem solving √ []

Reference(s) Bailey & Love 26th Edition


Prepared by
College Frontier Medical & Dental College, Abbottabad

Name: Prof. Dr. Johar Ali

Signature: –––––––––––––––––––––––––––––––

Date: 2018

Note: Diagrams/Charts/Graphs if any should attach along this


BEHRIA UNIVERSITY ISLAMABAD
Multiple Choices Question
PROGRAM MBBS
SUBJECT Surgery
PROF Final
TOS – ID 17.2.1

Description ESOPHAGUS
STEM A 35 year old female patient complains of dysphagia for last 6 months more for liquids
than solids, Barium swallow shows bird beak deformity. What is likely diagnosis?

A Zenker’s diverticulum
B Achalasia
C Ca esophagus
D Peptic ulcer disease
E GORD

Correct Answer B

Domain: Recall [] Interpretation [√] Problem solving []

Reference(s) Bailey & Love 26th Edition


Prepared by
College Frontier Medical & Dental College, Abbottabad

Name: Prof. Dr. Johar Ali

Signature: –––––––––––––––––––––––––––––––

Date: 2018

Note: Diagrams/Charts/Graphs if any should attach along this


BEHRIA UNIVERSITY ISLAMABAD
Multiple Choices Question
PROGRAM MBBS
SUBJECT Surgery
PROF Final
TOS – ID 17.2.1

Description ESOPHAGUS
STEM Which of t he following is considered to be pre malignant lesion?

A Esophageal candidacies
B Barrets esophagus
C Schatzki rings
D Moniliasis
E Mallory Weiss Tear

Correct Answer B

Domain: Recall [] Interpretation [√] Problem solving []

Reference(s) Bailey & Love 26th Edition


Prepared by
College Frontier Medical & Dental College, Abbottabad

Name: Prof. Dr. Johar Ali

Signature: –––––––––––––––––––––––––––––––

Date: 2018

Note: Diagrams/Charts/Graphs if any should attach along this


BEHRIA UNIVERSITY ISLAMABAD
Multiple Choices Question
PROGRAM MBBS
SUBJECT Surgery
PROF Final
TOS – ID 17.2.1

Description ESOPHAGUS
STEM A patient presented in surgical OPD having severe cough especially at bed time and
regurgitation of undigested food material. He is also embarrassed by bad breath.

What is likely diagnosis?

A Ca Esophagus
B Pharyngeal puch
C Peptic stricture
D Barrets Esophagus
E GORD

Correct Answer E

Domain: Recall [] Interpretation [] Problem solving []

Reference(s) Bailey & Love 26th Edition


Prepared by
College Frontier Medical & Dental College, Abbottabad

Name: Prof. Dr. Johar Ali

Signature: –––––––––––––––––––––––––––––––

Date: 2018

Note: Diagrams/Charts/Graphs if any should attach along this


BEHRIA UNIVERSITY ISLAMABAD
Multiple Choices Question
PROGRAM MBBS
SUBJECT Surgery
PROF Final
TOS – ID 17.2.1

Description ESOPHAGUS
STEM A 50year old lady presented to surgical OPD complaining of progressive dysphagia,
weight loss and anemia. On examiantion patient is pale and emaciated.

What is likely diagnosis?

A GORD
B Achalasia
C Ca Esophagus
D Ca stomach
E PUD

Correct Answer C

Domain: Recall [] Interpretation [√] Problem solving []

Reference(s) Bailey & Love 26th Edition


Prepared by
College Frontier Medical & Dental College, Abbottabad

Name: Prof. Dr. Johar Ali

Signature: –––––––––––––––––––––––––––––––

Date: 2018

Note: Diagrams/Charts/Graphs if any should attach along this


BEHRIA UNIVERSITY ISLAMABAD
Multiple Choices Question
PROGRAM MBBS
SUBJECT Surgery
PROF Final
TOS – ID 17.2.1

Description ESOPHAGUS
STEM A 35 year old female patient complains of dysphagia for last 6 months more for liquids
than solids.

Which investigation will confirm diagnosis

A Barium Sallow
B Barium meal
C CT Scan
D Chest X-ray
E Ultra sound chest

Correct Answer A

Domain: Recall [] Interpretation [] Problem solving [√]

Reference(s) Bailey & Love 26th Edition


Prepared by
College Frontier Medical & Dental College, Abbottabad

Name: Prof. Dr. Johar Ali

Signature: –––––––––––––––––––––––––––––––

Date: 2018

Note: Diagrams/Charts/Graphs if any should attach along this


BEHRIA UNIVERSITY ISLAMABAD
Multiple Choices Question
PROGRAM MBBS
SUBJECT Surgery
PROF Final
TOS – ID 17.2.12

Description Anal Canal


STEM An elderly patient presented with fresh bleeding per rectum due to prolapsed thrombosed
haemorrhoids. He is running high grade fever with tachycardia. Blood urea level is
normal with low heamoglobin of 7 Gm/dl.

What will be you management option?

A Blood transfusion
B IV antibiotics analagesia and application of ice packs
C Immediate surgery
D Topical application of glycerol trinitrate
E High fibre diet

Correct Answer B

Domain: Recall [] Interpretation [] Problem solving [√]

Reference(s) Bailey & Love 26th Edition


Prepared by
College Frontier Medical & Dental College, Abbottabad

Name: Prof. Dr. Johar Ali

Signature: –––––––––––––––––––––––––––––––

Date: 2018

Note: Diagrams/Charts/Graphs if any should attach along this


BEHRIA UNIVERSITY ISLAMABAD
Multiple Choices Question
PROGRAM MBBS
SUBJECT Surgery
PROF Final
TOS – ID 17.2.12

Description Anal Canal


STEM An adult was operated for prolapsed haemorrhoids by junior registrar two months back.
He is now complaining of constipation and feels difficulty in evacuation even after
maximum abdominal straining. Digital rectal examiantion is not possible due difficulty to
well lubricated finger insinuation.

What post operative complication is it be?

A Anal stenosis
B Anal stricture
C Anal fissure
D Carcinoma
E Post operative abcess

Correct Answer A

Domain: Recall [] Interpretation [] Problem solving √ []

Reference(s) Bailey & Love 26th Edition


Prepared by
College Frontier Medical & Dental College, Abbottabad

Name: Prof. Dr. Johar Ali

Signature: –––––––––––––––––––––––––––––––

Date: 2018

Note: Diagrams/Charts/Graphs if any should attach along this


BEHRIA UNIVERSITY ISLAMABAD
Multiple Choices Question
PROGRAM MBBS
SUBJECT Surgery
PROF Final
TOS – ID 17.2.12

Description Anal Canal


STEM A known patient of Chrons disease presented with high level of fistula in ano diagnosed
on contrast MRI. His blood sugar is 400 mg/dl with normal urea’s nitrogen. What will be
your choice of management?

A Fistulotomy
B Fistulectomy
C Application of fibrin glue
D Insertion of seton
E Four finger dilatation

Correct Answer D

Domain: Recall [] Interpretation [] Problem solving [√]

Reference(s) Bailey & Love 26th Edition


Prepared by
College Frontier Medical & Dental College, Abbottabad

Name: Prof. Dr. Johar Ali

Signature: –––––––––––––––––––––––––––––––

Date: 2018

Note: Diagrams/Charts/Graphs if any should attach along this


BEHRIA UNIVERSITY ISLAMABAD
Multiple Choices Question
PROGRAM MBBS
SUBJECT Surgery
PROF Final
TOS – ID 17.2.10

Description APPENDIX
STEM A 15 year old boy is brought to you with the history of pain abdomen nausea/anorexia
since 24 hours, O/E his temp. 99ºF and his right iliac fossa is tender.

What is the most likely diagnosis?

A Acute appendicitis
B Acute cholecysititis
C Rt. Ureteric colic
D Peptic ulcers disease
E Intestinal obstruction

Correct Answer A

Domain: Recall [] Interpretation [√] Problem solving []

Reference(s) Bailey & Love 26th Edition


Prepared by
College Frontier Medical & Dental College, Abbottabad

Name: Prof. Dr. Johar Ali

Signature: –––––––––––––––––––––––––––––––

Date: 2018

Note: Diagrams/Charts/Graphs if any should attach along this


BEHRIA UNIVERSITY ISLAMABAD
Multiple Choices Question
PROGRAM MBBS
SUBJECT Surgery
PROF Final
TOS – ID 17.2.10

Description APPENDIX
STEM A 16 years old boy was diagnosed as a case of acute appendicitis and was advised
surgery. His parents denied surgery and took the boy home against medical advice. He
was brought again after one week with high fever and severe abdominal pain and
absolute constipation. His abdominal examiantion revealed tense and tender abdomen
with absent bowl sounds.

What is your diagnosis?

A Peritonitis due to appendicitis perforation


B Peritonitis due to enteric perforation
C Acute intestinal obstruction
D Perforated DU
E Meckle’s Diverticulitis

Correct Answer A

Domain: Recall [] Interpretation [] Problem solving []

Reference(s) Bailey & Love 26th Edition


Prepared by
College Frontier Medical & Dental College, Abbottabad

Name: Prof. Dr. Johar Ali

Signature: –––––––––––––––––––––––––––––––

Date: 2018

Note: Diagrams/Charts/Graphs if any should attach along this


BEHRIA UNIVERSITY ISLAMABAD
Multiple Choices Question
PROGRAM MBBS
SUBJECT Surgery
PROF Final
TOS – ID 17.2.10

Description APPENDIX
STEM A 20 years old lady underwent appendicectomy for acute gangrenous appendicitis and
discharged home after 02 days. She presented again after a week with fever and rigors
and loose motions. Abdominal examination is normal.

What complication has happened to this lady?

A Wound infection
B Gastroenteritis/food poisoning
C Pelvic abscess
D Pseudo membranes colitis
E Ulcerative colitis

Correct Answer

Domain: Recall [] Interpretation [√] Problem solving []

Reference(s) Bailey & Love 26th Edition


Prepared by
College Frontier Medical & Dental College, Abbottabad

Name: Prof. Dr. Johar Ali

Signature: –––––––––––––––––––––––––––––––

Date: 2018

Note: Diagrams/Charts/Graphs if any should attach along this


BEHRIA UNIVERSITY ISLAMABAD
Multiple Choices Question
PROGRAM MBBS
SUBJECT Surgery
PROF Final
TOS – ID 17.2.10

Description APPENDIX
STEM A 25 years old man was diagnosed as a case of acute appendicitis and his abdomen
opened for surgery. Operative findings revealed 1.5 cm swelling at the tip of the
vermiform appendix with normal appendicullar base. Which of the following procedures
you will do

A Appendicectomy
B Rt. Hemicoloectomy
C Appendicectomy and histopathology
D Mesenteric lymph-node biopsy
E Close the abdomen without doing anything

Correct Answer A

Domain: Recall [] Interpretation [] Problem solving []

Reference(s) Bailey & Love 26th Edition


Prepared by
College Frontier Medical & Dental College, Abbottabad

Name: Prof. Dr. Johar Ali

Signature: –––––––––––––––––––––––––––––––

Date: 2018

Note: Diagrams/Charts/Graphs if any should attach along this


BEHRIA UNIVERSITY ISLAMABAD
Multiple Choices Question
PROGRAM MBBS
SUBJECT Surgery
PROF Final
TOS – ID 17.2.10

Description APPENDIX
STEM A 15 year old girl is c/o pain abdomen, anorexia, Nausea since 12 hours. She was found
to have a temp. 99º F and pulse 80/min and tender right iliac fossa.

Which of the following investigation is most helpful is making the diagnosis?

A TLC
B CT Scan abdomen
C LFTs
D USG abdomen
E X=ray KUB

Correct Answer B

Domain: Recall [] Interpretation [] Problem solving [√]

Reference(s) Bailey & Love 26th Edition


Prepared by
College Frontier Medical & Dental College, Abbottabad

Name: Prof. Dr. Johar Ali

Signature: –––––––––––––––––––––––––––––––

Date: 2018

Note: Diagrams/Charts/Graphs if any should attach along this


BEHRIA UNIVERSITY ISLAMABAD
Multiple Choices Question
PROGRAM MBBS
SUBJECT Surgery
PROF Final
TOS – ID 17.2.10

Description APPENDIX
STEM A 30 years old man was operated upon for appendicular abscess through Rutherford
Morrison’s incision. Abscess was drain and appendicectomy also performed. The patient
developed greenish discharge of semisolid consistency from the wound after 05 days.

What complication has happened to the patient?

A Wound infection
B Clostridial wound infection
C Fecal fistula
D Recurrence of abscess I n right iliac fossa
E Wound dehiscence

Correct Answer C

Domain: Recall [] Interpretation [] Problem solving [√]

Reference(s) Bailey & Love 26th Edition


Prepared by
College Frontier Medical & Dental College, Abbottabad

Name: Prof. Dr. Johar Ali

Signature: –––––––––––––––––––––––––––––––

Date: 2018

Note: Diagrams/Charts/Graphs if any should attach along this


BEHRIA UNIVERSITY ISLAMABAD
Multiple Choices Question
PROGRAM MBBS
SUBJECT Surgery
PROF Final
TOS – ID 17.2.10

Description APPENDIX
STEM In Alvarado scoring for acute appendicitis which of the following parameter carry more
weightage?

A Nausea
B Pain abdomen
C Anorexia
D Fever low grade
E Tender RIF

Correct Answer E

Domain: Recall [] Interpretation [√] Problem solving []

Reference(s) Bailey & Love 26th Edition


Prepared by
College Frontier Medical & Dental College, Abbottabad

Name: Prof. Dr. Johar Ali

Signature: –––––––––––––––––––––––––––––––

Date: 2018

Note: Diagrams/Charts/Graphs if any should attach along this


BEHRIA UNIVERSITY ISLAMABAD
Multiple Choices Question
PROGRAM MBBS
SUBJECT Surgery
PROF Final
TOS – ID 17.2.10

Description APPENDIX
STEM In thin, lean females with uncomplicated appendicitis, which of the following incisions
you will choose for appendicectomy

A Lanze incision
B Rt. Gridiron incision
C Rutherford Morrison’s Incision
D Pfanenstiel incision
E Rt. Para median incision

Correct Answer A

Domain: Recall [] Interpretation [] Problem solving []

Reference(s) Bailey & Love 26th Edition


Prepared by
College Frontier Medical & Dental College, Abbottabad

Name: Prof. Dr. Johar Ali

Signature: –––––––––––––––––––––––––––––––

Date: 2018

Note: Diagrams/Charts/Graphs if any should attach along this


BEHRIA UNIVERSITY ISLAMABAD
Multiple Choices Question
PROGRAM MBBS
SUBJECT Surgery
PROF Final
TOS – ID 17.2.10

Description
STEM An obese unmarried lady of 25 years is suspected of having acute appendicitis. Her
Alverado’s Scroe is 7. USG is normal.

Which of the following options you will opt for.

A Observation + monitoring
B Laparoscopic appendicictomy
C Open appendicectomy through Lanze incision
D Appendicectomy through Rutherford Morrison’s incision
E Advise CT Scan abdomen

Correct Answer B

Domain: Recall [] Interpretation [√] Problem solving []

Reference(s) Bailey & Love 26th Edition


Prepared by
College Frontier Medical & Dental College, Abbottabad

Name: Prof. Dr. Johar Ali

Signature: –––––––––––––––––––––––––––––––

Date: 2018

Note: Diagrams/Charts/Graphs if any should attach along this


BEHRIA UNIVERSITY ISLAMABAD
Multiple Choices Question
PROGRAM MBBS
SUBJECT Surgery
PROF Final
TOS – ID 17.2.10

Description APPENDIX
STEM Acute appendicitis is one of the common cause of acute abdomen.

Which of the following is most common position of appendix?

A Pelvic
B Paracecal
C Prelieal
D Retrocecal
E Subcecal

Correct Answer D

Domain: Recall [] Interpretation [√] Problem solving []

Reference(s) Bailey & Love 26th Edition


Prepared by
College Frontier Medical & Dental College, Abbottabad

Name: Prof. Dr. Johar Ali

Signature: –––––––––––––––––––––––––––––––

Date: 2018

Note: Diagrams/Charts/Graphs if any should attach along this


BEHRIA UNIVERSITY ISLAMABAD
Multiple Choices Question
PROGRAM MBBS
SUBJECT Surgery
PROF Final
TOS – ID 17.2.10

Description ESOPHAGUS
STEM A young boy had pain epigasterium and nausea four days back. Pain is now in right iliac
fossa with hectic high grade fever. Examiantion of right iliac fossa shows fluctuant
swelling His TLC is 20,000 cmm. Ultrasound abdomen shows cystic collection that is
well localized. What is your diagnosis?

A Apppendicular mass
B Appendicular abscess
C Localized pelvic peritonitis
D Illioceeacel tuberculosis
E Colonic tumor

Correct Answer B

Domain: Recall [] Interpretation [√] Problem solving []

Reference(s) Bailey & Love 26th Edition


Prepared by
College Frontier Medical & Dental College, Abbottabad

Name: Prof. Dr. Johar Ali

Signature: –––––––––––––––––––––––––––––––

Date: 2018

Note: Diagrams/Charts/Graphs if any should attach along this

BEHRIA UNIVERSITY ISLAMABAD


Multiple Choices Question
PROGRAM MBBS
SUBJECT Surgery
PROF Final
TOS – ID 17.3.1

Description Kidney, Ureter


STEM A young adult is presented with constant background dull pain over left lumber region
with acute severe episodes with gross haematuria. Patient is writhing with agonizing pain
and moves around in different postures to gain comfort.

What is your most likely diagnosis?

A Urinary tract infection


B Nephrolithiasis
C Stone in renal pelvis
D Stone at middle half ureter
E Stone in lower one thid of ureter

Correct Answer D

Domain: Recall [] Interpretation [] Problem solving []

Reference(s) Bailey & Love 26th Edition


Prepared by
College Frontier Medical & Dental College, Abbottabad

Name: Prof. Dr. Johar Ali

Signature: –––––––––––––––––––––––––––––––

Date: 2018

Note: Diagrams/Charts/Graphs if any should attach along this


BEHRIA UNIVERSITY ISLAMABAD
Multiple Choices Question
PROGRAM MBBS
SUBJECT Surgery
PROF Final
TOS – ID 17.3.1

Description KIDNEY, URTER


STEM A suspected case of renal stone is brought to you with severe pain and haematuria. You
give him intramuscular diclofenac sodium for his pain.

What next investigation you will prescribe for this patient?

A X-ray and ultrasound kidney, ureter and bladder


B Intravenous urography
C Spiral Ct Scan
D Magnetic resonance imaging
E Radio isotope scanning

Correct Answer C

Domain: Recall [] Interpretation [] Problem solving []

Reference(s) Bailey & Love 26th Edition


Prepared by
College Frontier Medical & Dental College, Abbottabad

Name: Prof. Dr. Johar Ali

Signature: –––––––––––––––––––––––––––––––

Date: 2018

Note: Diagrams/Charts/Graphs if any should attach along this


BEHRIA UNIVERSITY ISLAMABAD
Multiple Choices Question
PROGRAM MBBS
SUBJECT Surgery
PROF Final
TOS – ID 17.3.1

Description KIDNEY, URETER


STEM A patient was investigated for pain abdomen and incidentally found to have absent left
kidney on ultrasound with contra lateral enlarged kidney. There is no history of past renal
surgery. His renal function tests are normal.

What is the incidence of this congenital abnormality?

A Two in 1200
B Three in 1300
C Four in 1400
D Five in 1500
E One in 1400

Correct Answer E

Domain: Recall [] Interpretation [] Problem solving []

Reference(s) Bailey & Love 26th Edition


Prepared by
College Frontier Medical & Dental College, Abbottabad

Name: Prof. Dr. Johar Ali

Signature: –––––––––––––––––––––––––––––––

Date: 2018

Note: Diagrams/Charts/Graphs if any should attach along this


BEHRIA UNIVERSITY ISLAMABAD
Multiple Choices Question
PROGRAM MBBS
SUBJECT Surgery
PROF Final
TOS – ID 17.3.1

Description KIDNEY, URETER


STEM A young patient is presented in OPD with recurrent attacks of urinary tract infection and
pain right iliac fossa. Initially appendicualr mass is suspected but ultrasound examination
shows kidney like mass with dilated calyces at the leve of diagnosis?

What is your diagnosis?

A Horse shoe kidney


B Renal ectopia
C Crossed dystopia
D Crohn disease
E Appendicular mass/abcess

Correct Answer B

Domain: Recall [] Interpretation [] Problem solving []

Reference(s) Bailey & Love 26th Edition


Prepared by
College Frontier Medical & Dental College, Abbottabad

Name: Prof. Dr. Johar Ali

Signature: –––––––––––––––––––––––––––––––

Date: 2018

Note: Diagrams/Charts/Graphs if any should attach along this


BEHRIA UNIVERSITY ISLAMABAD
Multiple Choices Question
PROGRAM MBBS
SUBJECT Surgery
PROF Final
TOS – ID 17.3.4

Description PENIS AND URETHRA


STEM A known patient of urethral injury with supra-pubic systoestomy Cather in situ is brought
to hospital for removal of Cather. Which investigation is mandatory before it’s removal
to look for underlying urethral pathology.

Select the right option?

A X-ray pelvis
B Intravenous urography
C Radio isotope scan
D Retro grade urethrogram
E Micturating cystogram

Correct Answer D

Domain: Recall [] Interpretation [] Problem solving []

Reference(s) Bailey & Love 26th Edition


Prepared by
College Frontier Medical & Dental College, Abbottabad

Name: Prof. Dr. Johar Ali

Signature: –––––––––––––––––––––––––––––––

Date: 2018

Note: Diagrams/Charts/Graphs if any should attach along this


BEHRIA UNIVERSITY ISLAMABAD
Multiple Choices Question
PROGRAM MBBS
SUBJECT Surgery
PROF Final
TOS – ID 17.3.1

Description KIDNEY, URETER


STEM A young footballer was hit by opponent player in left renal area during annual
tournament. He is complaining of pain and haematuria. A closed renal injury is
suspected.

What investigation will be helpful in diagnosis of underlying damage?

A Ultrasonography of kidney
B Contrast enhanced CT scan
C Intravenous urography
D Radio isotope scanning
E CT Scan

Correct Answer B

Domain: Recall [] Interpretation [] Problem solving []

Reference(s) Bailey & Love 26th Edition


Prepared by
College Frontier Medical & Dental College, Abbottabad

Name: Prof. Dr. Johar Ali

Signature: –––––––––––––––––––––––––––––––

Date: 2018

Note: Diagrams/Charts/Graphs if any should attach along this


BEHRIA UNIVERSITY ISLAMABAD
Multiple Choices Question
PROGRAM MBBS
SUBJECT Surgery
PROF Final
TOS – ID 17.3.1

Description KIDNEY, URETER


STEM A male child of 4 years is repeatedly has urinary tract infections. He has been on multiple
antibiotics. Ultrasonography reveals bilateral moderate hydro nephrosis with no evidence
of renal stones.

What is most likely underlying congenital abnormality?

A Ectopia vesicae
B Urinary tract infection
C Urinary bladder diverticulum
D Idiopathic vesicoureteric reflux
E Posterior urethral valves

Correct Answer E

Domain: Recall [] Interpretation [] Problem solving []

Reference(s) Bailey & Love 26th Edition


Prepared by
College Frontier Medical & Dental College, Abbottabad

Name: Prof. Dr. Johar Ali

Signature: –––––––––––––––––––––––––––––––

Date: 2018

Note: Diagrams/Charts/Graphs if any should attach along this


BEHRIA UNIVERSITY ISLAMABAD
Multiple Choices Question
PROGRAM MBBS
SUBJECT Surgery
PROF Final
TOS – ID 17.3.3

Description Prostate and seminal vesicles


STEM In an elderly patient who has difficulty in urination was found to have impaired
renal functions along bilateral hydronephrosis.

What is most likely underlying pathology?

A Benign prostatic enlargement


B Bladder stone
C Bladder tumor
D Interstitial cystitis
E Urethral stricture

Correct Answer

Domain: Recall [] Interpretation [] Problem solving []

Reference(s) Bailey & Love 26th Edition


Prepared by
College Frontier Medical & Dental College, Abbottabad

Name: Prof. Dr. Johar Ali

Signature: –––––––––––––––––––––––––––––––

Date: 2018

Note: Diagrams/Charts/Graphs if any should attach along this


BAHRIA UNIVERSITY ISLAMABAD

MULTIPLE CHOICE QUESTIONS

Program MBBS

Subject General Surgery

Prof Final

TOS ID 17.2.1

Description GIT ( Esophagus )

stem A 35 year old female patient complains of dysphagia for last 6 months more for liquids
than solids. Barium swallow shows bird beak deformity. What is likely diagnosis

Item A Zenkers diverticulum

Item B Achalasia

Item C Ca esophagus

Item D Peptic ulcer disease

Item E GORD

Correct answer B

Domain; Recall Interpretation Problem solving

Reference; Bailey and love Edition 26 page 1014

Prepared by;

College; Frontier Medical and Dental College, Abbottabad.

Name; Prof.Dr.Johar Ali

Signature

Date;
BAHRIA UNIVERSITY ISLAMABAD

MULTIPLE CHOICE QUESTIONS

Program MBBS

Subject General Surgery

Prof Final

TOS ID 17.2.1

Description GIT ( Esophagus )

Stem Which of the following is considered to be pre malignant lesion

Item A Esophageal candidiasis

Item B Barrets esophagus

Item C Schatzki rings

Item D Moniliasis

Item E Mallory Weiss tear

Correct answer B

Domain; Recall Interpretation Problem solving


Reference; Bailey and love Edition 26 page 1000

Prepared by;

College; Frontier Medical and Dental College, Abbottabad

Name; Prof.Dr.Johar Ali

Signature

Date;

BAHRIA UNIVERSITY ISLAMABAD

MULTIPLE CHOICE QUESTIONS

Program MBBS

Subject General Surgery

Prof Final

TOS ID 17.2.1

Description GIT ( Esophagus )

stem A patient presented in surgical OPD having severe cough especially at bed time and
regurgitation of undigested food material. He is also embarrassed by bad breath. What is
likely diagnosis

Item A Ca Esophagus
Item B Pharyngeal pouch

Item C Peptic stricture

Item D Barrets Esophagus

Item E GORD

Correct Answer B

Domain; Recall Interpretation Problem solving

Reference; Bailey and love Edition 26 page 1018

Prepared by;

College; Frontier Medical and Dental College, Abbottabad.

Name; Prof.Dr.Johar Ali

Signature

Date;

BAHRIA UNIVERSITY ISLAMABAD

MULTIPLE CHOICE QUESTIONS

Program MBBS

Subject General Surgery


Prof Final

TOS ID 17.2.1

Description GIT ( Esophagus )

stem A 50 year old lady presented to surgical OPD complaining of progressive dysphagia, weight
loss and anemia. On examination patient is pale and emaciated. What is likely diagnosis

Item A GORD

Item B Achalasia

Item C Ca Esophagus

Item D Ca Stomach

Item E PUD

Correct answer C

Domain; Recall Interpretation Problem solving

Reference; Bailey and love Edition 26 page 1004

Prepared by;

College; Frontier Medical and Dental College, Abbottabad.

Name; Prof.Dr.Johar Ali

Signature

Date;
BAHRIA UNIVERSITY ISLAMABAD

MULTIPLE CHOICE QUESTIONS

Program MBBS

Subject General Surgery

Prof Final

TOS ID 17.2.1

Description GIT ( Esophagus )

stem A 50 year old lady presented to surgical OPD complaining of progressive dysphagia,
weight loss and anemia. On examination patient is pale and emaciated. Which
investigation will confirm diagnosis

Item A Ultra sound abdomen

Item B Chest X Ray

Item C CT Scan

Item D Endoscopy

Item E Barium Meal

Correct answer D

Domain; Recall Interpretation Problem solving

Reference; Bailey and love Edition 26 page 1007

Prepared by;

College; Frontier Medical and Dental College, Abbottabad.

Name; Prof.Dr.Johar Ali


Signature

Date;

BAHRIA UNIVERSITY ISLAMABAD

MULTIPLE CHOICE QUESTIONS

Program MBBS

Subject General Surgery

Prof Final

TOS ID 17.2.1

Description GIT ( Esophagus )

stem A 35 year old female patient complains of dysphagia for last 6 months more for liquids
than solids. Which investigation will confirm diagnosis

Item A Barium Sallow

Item B Barium meal

Item C CT scan

Item D Chest X Ray

Item E Ultra sound chest

Correct answer A
Domain; Recall Interpretation Problem solving

Reference; Bailey and love Edition 26 page 1014

Prepared by;

College; Frontier Medical and Dental College, Abbottabad

Name; Prof.Dr.Johar Ali

Signature

Date;

BAHRIA UNIVERSITY ISLAMABAD

MULTIPLE CHOICE QUESTIONS

Program MBBS

Subject General Surgery

Prof Final

TOS ID 17.2.1

Description GIT ( Esophagus )

stem A 35 year old female patient complains of dysphagia for last 6 months more for liquids
than solids. Barium swallow shows bird beak deformity and proximal dilated esophagus.
What is treatment option
Item A Esophagectomy

Item B Heller’s Cardiomyotomy

Item C Endoscopic balloon dilatation

Item D Stenting

Item E Fundoplication

Correct answer C

Domain; Recall Interpretation Problem solving

Reference; Bailey and love Edition 26 page 1015

Prepared by;

College; Frontier Medical and Dental College, Abbottabad

Name; Prof.Dr.Johar Ali

Signature

Date;

BAHRIA UNIVERSITY ISLAMABAD

MULTIPLE CHOICE QUESTIONS


Program MBBS

Subject General Surgery

Prof Final

TOS ID 17.2.2

Description GIT ( Stomach & duodenum)

stem A 55 year old male known smoker presented in emergency with history of vomiting and
upper abdominal pain for last few hours. Examination revealed pallor, pulse 110/min, BP
110/70 & tense and tender abdomen. What is likely cause

Item A Acute pancreatitis

Item B Acute Gastritis

Item C Perforated duodenal ulcer

Item D Acute cholecystitis

Item E Acute appendicitis

Correct answer C

Domain; Recall Interpretation Problem solving

Reference; Bailey and love Edition 26 page 1041

Prepared by;

College: Frontier Medical and Dental College, Abbottabad.

Name; Prof.Dr.Johar Ali

Signature

Date;
BAHRIA UNIVERSITY ISLAMABAD

MULTIPLE CHOICE QUESTIONS

Program MBBS

Subject General Surgery

Prof Final

TOS ID 17.2.2

Description GIT ( Stomach & duodenum)

stem A 55 year old male known smoker presented in emergency with history of vomiting and
upper abdominal pain for last few hours. Examination revealed pallor, pulse 110/min, BP
110/70 & tense and tender abdomen. Which investigation will help in diagnosis

Item A TLC

Item B X Ray erect abdomen

Item C CXR

Item D Barium swallow

Item E Ultra sound abdomen

Correct answer B

Domain; Recall Interpretation Problem solving

Reference; Bailey and love Edition 26 page 1042


Prepared by;

College: Frontier Medical and Dental College, Abbottabad.

Name; Prof.Dr.Johar Ali

Signature

Date;

BAHRIA UNIVERSITY ISLAMABAD

MULTIPLE CHOICE QUESTIONS

Program MBBS

Subject General Surgery

Prof Final

TOS ID 17.2.2

Description GIT ( Stomach & duodenum)

stem A 55 year old female presented in OPD with history of hematamesis and malena for last
few days. She is also using NSAIDs for arthritis for few years. What is likely cause

Item A CA Esophagus

Item B Stress Gastritis

Item C CA Stomach

Item D Erosive Gastritis


Item E Gastric outlet obstruction

Correct answer D

Domain; Recall Interpretation Problem solving

Reference; Bailey and love Edition 26 page 1032

Prepared by;

College; Frontier Medical and Dental College, Abbottabad.

Name; Prof.Dr.Johar Ali

Signature

Date;

BAHRIA UNIVERSITY ISLAMABAD

MULTIPLE CHOICE QUESTIONS

Program MBBS

Subject General Surgery

Prof Final
TOS ID 17.2.2

Description GIT ( Stomach & duodenum)

stem A 45 year old male presented in OPD with history of non bilious vomiting 2-3 hours after
meal containing digested food. He has also lost weight in recent days. Examination
revealed dehydration, pallor and palpable mass in epigastrium. What is probable diagnosis

Item A Gastritis

Item B CA esophagus

Item C Ca Stomach

Item D Ca head of pancreas

Item D GORD

Correct answer C

Domain; Recall Interpretation Problem solving

Reference; Bailey and love Edition 26 page 1046

Prepared by;

College: Frontier Medical and Dental College, Abbottabad.

Name; Prof.Dr.Johar Ali

Signature

Date;
BAHRIA UNIVERSITY ISLAMABAD

MULTIPLE CHOICE QUESTIONS

Program MBBS

Subject General Surgery

Prof Final

TOS ID 17.2.2

Description GIT ( Stomach & duodenum)

stem A 54 year old male had previous total gastrectomy presents with anemia & tingling in
arms and legs. Peripheral blood smear was done that showed macrocytic anemia.
What is likely cause

Item A Early dumping syndrome

Item B Recurrent ulceration causing bleeding

Item C Malignancy

Item D B12 deficiency

Item E Bleeding varices

Correct answer D

Domain; Recall Interpretation Problem solving

Reference; Bailey and love Edition 26 page 1053

Prepared by;

College; Frontier Medical and Dental College, Abbottabad

Name; Prof.Dr.Johar Ali


Signature

Date;

BAHRIA UNIVERSITY ISLAMABAD

MULTIPLE CHOICE QUESTIONS

Program MBBS

Subject General Surgery

Prof Final

TOS ID 17.2.2

Description GIT ( Stomach & duodenum)

stem A 45 year old male had previous surgery for peptic ulcer presents with explosive
diarrhoe that is not responding to routine anti diarrhoe medication. What is likely cause

Item A H-Pylori infection

Item B Malignancy

Item C Dumping syndrome

Item D Post vagotomy diarrhoe

Item E B12 deficiency

Correct answer D
Domain; Recall Interpretation Problem solving

Reference; Bailey and love Edition 26 page 1039

Prepared by;

College; Frontier Medical and Dental College, Abbottabad

Name; Prof.Dr.Johar Ali

Signature

Date;

BAHRIA UNIVERSITY ISLAMABAD

MULTIPLE CHOICE QUESTIONS

Program MBBS

Subject General Surgery

Prof Final

TOS ID 17.2.2

Description GIT ( Stomach & duodenum)

Stem Which of the following best explains a longitudinal tear below gastro esophageal
junction induced by repetitive & strenuous vomiting that causes hematamesis
Item A Gastric volvolus

Item B Esophagael varices

Item C Mallory Weiss tear

Item D Gastric ulcer

Item E Gastric polyp

Correct answer C

Domain; Recall Interpretation Problem solving

Reference; Bailey and love Edition 26 page 1043

Prepared by;

College; Frontier Medical and Dental College, Abbottabad

Name; Prof.Dr.Johar Ali

Signature

Date;

BAHRIA UNIVERSITY ISLAMABAD


MULTIPLE CHOICE QUESTIONS

Program MBBS

Subject General Surgery

Prof Final

TOS ID 17.2.2

Description GIT ( Stomach & duodenum)

Stem What is common site of peptic ulcer caused by H-pylori

Item A Greater curvature of stomach

Item B 1st part of duodenum

Item C Esophago gastric junction

Item D Esophagus

Item E DJ junction

Correct answer B

Domain; Recall Interpretation Problem solving

Reference; Bailey and love Edition 26 page 1032

Prepared by;

College; Frontier Medical and Dental College, Abbottabad

Name; Prof.Dr.Johar Ali

Signature

Date;
BAHRIA UNIVERSITY ISLAMABAD

MULTIPLE CHOICE QUESTIONS

Program MBBS

Subject General Surgery

Prof Final

TOS ID 17.2.2

Description GIT ( Stomach & duodenum)

Stem Troisiers sign is enlarged

Item A Left axillary lymph node

Item B Left supra clavicular lymph node

Item C Left inguinal lymph node

Item D Right supra clavicle lymph node

Item E Cervical lymph node

Correct answer B

Domain; Recall Interpretation Problem solving

Reference; Bailey and love Edition 26 page 1048


Prepared by;

College; Frontier Medical and Dental College, Abbottabad

Name; Prof.Dr.Johar Ali

Signature

Date;

BAHRIA UNIVERSITY ISLAMABAD

MULTIPLE CHOICE QUESTIONS

Program MBBS

Subject General Surgery

Prof Final

TOS ID 17.2.2

Description GIT ( Stomach & duodenum)

Stem A 45 year old male presented in OPD with history of non bilious vomiting 2-3 hours
after meal containing digested food. He has also lost weight in recent days.
Examination revealed dehydration, pallor and palpable mass in epigastrium and
positive succession splash. Which investigation will help in diagnosis

Item A X- Ray Abdomen

Item B Ultra sound abdomen


Item C CT scan abdomen

Item D Endoscopy

Item E Barium swallow

Correct answer D

Domain; Recall Interpretation Problem solving

Reference; Bailey and love Edition 26 page 1027

Prepared by;

College; Frontier Medical and Dental College, Abbottabad

Name; Prof.Dr.Johar Ali

Signature

Date;

BAHRIA UNIVERSITY ISLAMABAD

MULTIPLE CHOICE QUESTIONS

Program MBBS

Subject General Surgery


Prof Final

TOS ID 17.2.3

Description GIT ( Liver )

stem The commonest benign tumor of liver is

Item A Fibroma

Item B Hemangioma

Item C Adenoma

Item D Lymphoma

Item E Hematoma

Correct answer B

Domain; Recall Interpretation Problem solving

Reference; Bailey and love Edition 26 page 1083

Prepared by;

College; Frontier Medical and Dental College, Abbottabad

Name; Prof.Dr.Johar Ali

Signature

Date;
BAHRIA UNIVERSITY ISLAMABAD

MULTIPLE CHOICE QUESTIONS

Program MBBS

Subject General Surgery

Prof Final

TOS ID 17.2.3

Description GIT ( Liver)

Stem Budd Chiari Syndrome refers to

Item A Portal vein thrombosis

Item B Bile duct obstruction

Item C Hepatic vein thrombosis

Item D Hepatic artery thrombosis

Item E Mesenteric artery thrombosis

Correct answer C

Domain; Recall Interpretation Problem solving

Reference; Bailey and love Edition 26 page 1077

Prepared by;

College; Frontier Medical and Dental College, Abbottabad


Name; Prof.Dr.Johar Ali

Signature

Date;

BAHRIA UNIVERSITY ISLAMABAD

MULTIPLE CHOICE QUESTIONS

Program MBBS

Subject General Surgery

Prof Final

TOS ID 17.2.3

Description GIT ( Liver)

stem A 45 year old diabetic patient complains of anorexia, high grade fever, malaise and
right upper quadrant discomfort. On examination there is tender hepatomegaly.
Examination revealed multiloculated cystic mass in liver. What is likely diagnosis

Item A Amoebic liver abscess

Item B Hydatid cyst

Item C Pyogenic liver abscess

Item D Viral hepatitis

Item E Ascending cholangitis


Correct answer C

Domain; Recall Interpretation Problem solving

Reference; Bailey and love Edition 26 page 1081

Prepared by;

College; Frontier Medical and Dental College, Abbottabad

Name; Prof.Dr.Johar Ali

Signature

Date;

BAHRIA UNIVERSITY ISLAMABAD

MULTIPLE CHOICE QUESTIONS

Program MBBS

Subject General Surgery

Prof Final

TOS ID 17.2.3

Description GIT ( Liver)

stem A 50 year old farmer presents in OPD with continuous dull ache in Right
hypochondrium for few days. Examination shows palpable mass arising from right
lobe liver. Blood count shows increased Eosinophils. CT scan shows lesion in right
lobe with floating membrane. What is likely diagnosis

Item A Hepatoma

Item B Hydatid liver cyst

Item C Ascending cholangitis

Item D Hemangioma

Item E Amoebic abscess

Correct answer B

Domain; Recall Interpretation Problem solving

Reference; Bailey and love Edition 26 page 1081

Prepared by;

College; Frontier Medical and Dental College, Abbottabad

Name; Prof.Dr.Johar Ali

Signature

Date;

BAHRIA UNIVERSITY ISLAMABAD


MULTIPLE CHOICE QUESTIONS

Program MBBS

Subject General Surgery

Prof Final

TOS ID 17.2.3

Description GIT (Liver)

Stem A 40 year old male presents with persistent upper abdominal pain, malaise and
weight loss. Examination reveals palpable mass in right hypochondrium. Alpha
fetoprotein was raised. What is likely diagnosis

Item A Focal nodular hyperplasia

Item B Hemangioma

Item C Hepatocellular Carcinoma

Item D Hepatic adenoma

Item E Liver cirrhosis

Correct answer C

Domain; Recall Interpretation Problem solving

Reference; Bailey and love Edition 26 page 1085

Prepared by;

College; Frontier Medical and Dental College, Abbottabad

Name; Prof.Dr.Johar Ali

Signature

Date;
BAHRIA UNIVERSITY ISLAMABAD

MULTIPLE CHOICE QUESTIONS

Program MBBS

Subject General Surgery

Prof Final

TOS ID 17.2.3

Description GIT (Liver)

Stem Charcot’s triad refers to

Item A Fever, Deranged LFTs , Anemia

Item B Fever, Pain right hypochondrium, jaundice

Item C Anemia, hepatomegaly, Jaundice

Item D Anemia, Pain right hypochondrium, jaundice

Item E Anemia, fever, jaundice

Correct answer B

Domain; Recall Interpretation Problem solving


Reference; Bailey and love Edition 26 page 1080

Prepared by;

College; Frontier Medical and Dental College, Abbottabad

Name; Prof.Dr.Johar Ali

Signature

Date;

BAHRIA UNIVERSITY ISLAMABAD

MULTIPLE CHOICE QUESTIONS

Program MBBS

Subject General Surgery

Prof Final

TOS ID 17.2.3

Description GIT ( Liver)

stem A 60 year old male presents with persistent upper abdominal pain, malaise and weight loss
& jaundice. He was operated 4 months back for CA Cecum and right hemicolectomy was
done. U/S shows two solid lesions in right lobe of liver. What is likely diagnosis

Item A Hepatocellular CA

Item B Hydatid cyst


Item C Hepatic adenoma

Item D Liver metastasis

Item E Amoebic liver abscess

Correct answer D

Domain; Recall Interpretation Problem solving

Reference; Bailey and love Edition 26 page 1084

Prepared by;

College; Frontier Medical and Dental College, Abbottabad

Name; Prof.Dr.Johar Ali

Signature

Date;

BAHRIA UNIVERSITY ISLAMABAD

MULTIPLE CHOICE QUESTIONS

Program MBBS

Subject General Surgery

Prof Final
TOS ID 17.2.3

Description GIT (Liver)

stem Name the tumor marker of liver carcinoma

Item A CEA

Item B Alpha fetoprotein

Item C HCG

Item D LDH

Item E CA19-9

Correct answer B

Domain; Recall Interpretation Problem solving

Reference; Bailey and love Edition 26 page 1085

Prepared by;

College; Frontier Medical and Dental College, Abbottabad

Name; Prof.Dr.Johar Ali

Signature

Date;
BAHRIA UNIVERSITY ISLAMABAD

MULTIPLE CHOICE QUESTIONS

Program MBBS

Subject General Surgery

Prof Final

TOS ID 17.2.3

Description GIT ( Liver)

stem A 50 year old farmer presents in OPD with continuous dull ache in Right hypochondrium
for few days. Examination shows palpable mass arising from right lobe liver. Blood count
shows increased Eosinophils. CT scan shows lesion in right lobe with multiple septae about
4×5 cm. What is treatment option

Item A Anti Helminthic

Item B PAIR

Item C Open Surgery

Item D Observation

Item E Laparoscopy

Correct answer B

Domain; Recall Interpretation Problem solving

Reference; Bailey and love Edition 26 page 1081

Prepared by;

College; Frontier Medical and Dental College, Abbottabad


NameProf.Dr.Johar Ali;

Signature

Date;

BAHRIA UNIVERSITY ISLAMABAD

MULTIPLE CHOICE QUESTIONS

Program MBBS

Subject General Surgery

Prof Final

TOS ID 17.2.4

Description GIT ( Spleen)

stem A 12 year old school student presents in OPD with history of upper abdominal pain for
few days. Examination revealed marked pallor and enlarged spleen. U/Sound abdomen
shows multiple small calculi and splenomegaly. What is likely diagnosis

Item A Idiopathic thrombocytopenic purpura

Item B Hereditary spherocytosis

Item C Splenic Infarction

Item D Infective endocarditis

Item E Malaria
Correct answer B

Domain; Recall Interpretation Problem solving

Reference; Bailey and love Edition 26 page 1092

Prepared by; Prof.Dr.Johar Ali

College; Frontier Medical and Dental College, Abbottabad

Name;

Signature

Date;

BAHRIA UNIVERSITY ISLAMABAD

MULTIPLE CHOICE QUESTIONS

Program MBBS

Subject General Surgery

Prof Final

TOS ID 17.2.4

Description GIT ( Spleen)

stem A young cyclist fell down and had bruising over left hypochondrium. He presented in A&E
in state of shock. What is likely cause

Item A Liver injury

Item B Spleen injury

Item C Pancreatic injury

Item D Renal injury

Item E Gut injury

Correct answer B

Domain; Recall Interpretation Problem solving

Reference; Bailey and love Edition 26 page 1090

Prepared by;

College; Frontier Medical and Dental College, Abbottabad

Name; Prof.Dr.Johar Ali

Signature

Date;
BAHRIA UNIVERSITY ISLAMABAD

MULTIPLE CHOICE QUESTIONS

Program MBBS

Subject General Surgery

Prof Final

TOS ID 17.2.4

Description GIT ( Spleen)

Stem A young patient had splenectomy for Splenic trauma. He is likely to develop which early
complication after surgery

Item A Reactionary hemorrhage

Item B OPSI

Item C Thrombocytopenia

Item D Leucopenia

Item E Chronic gastric dilatation

Correct answer A

Domain; Recall Interpretation Problem solving

Reference; Bailey and love Edition 26 page 1096

Prepared by;

College; Frontier Medical and Dental College, Abbottabad

Name; Prof.Dr.Johar Ali

Signature

Date;
BAHRIA UNIVERSITY ISLAMABAD

MULTIPLE CHOICE QUESTIONS

Program MBBS

Subject General Surgery

Prof Final

TOS ID 17.2.4

Description GIT ( Spleen)

Stem A young patient had splenectomy for Splenic trauma. At time of discharge he was given
different vaccines to prevent which of the following

Item A SIRS

Item B SEPSIS

Item C OPSI

Item D Pancreatitis

Item E Thrombocytopenia

Correct answer C

Domain; Recall Interpretation Problem solving

Reference; Bailey and love Edition 26 page 1096


Prepared by;

College; Frontier Medical and Dental College, Abbottabad

Name; Prof.Dr.Johar Ali

Signature

Date;

BAHRIA UNIVERSITY ISLAMABAD

MULTIPLE CHOICE QUESTIONS

Program MBBS

Subject General Surgery

Prof Final

TOS ID 17..4

Description GIT (Liver)

stem A young cyclist fell down and had bruising over left hypochondrium. He presented in A&E
in state of shock. After resuscitation splenectomy was done for grade IV Splenic injury. On
3rd post operative day surgeon noticed serous fluid discharge from wound. Which
complication has occurred

Item A Gastric injury

Item B Adrenal injury


Item C Pancreatic injury

Item D Reactionary hemorrhage

Item E Wound infection

Correct answer C

Domain; Recall Interpretation Problem solving

Reference; Bailey and love Edition 26 page 1096

Prepared by;

College; Frontier Medical and Dental College, Abbottabad

Name; Prof.Dr.Johar Ali

Signature

Date;

BAHRIA UNIVERSITY ISLAMABAD

MULTIPLE CHOICE QUESTIONS

Program MBBS

Subject General Surgery


Prof Final

TOS ID 17.2.5

Description GIT ( Gall Bladder and bile duct )

Stem A 40 year old multiparous fatty female, presented in surgical OPD with history of
recurrent attacks of pain in RHC especially after fatty meals and flatulent dyspepsia.
What is likely cause

Item A Acute cholecystitis

Item B Acute pancreatitis

Item C Acute peritonitis

Item D Biliary colic

Item E Perforated duodenal ulcer

Correct answer D

Domain; Recall Interpretation Problem solving

Reference; Bailey and love Edition 26 page 1107

Prepared by;

College; Frontier Medical and Dental College, Abbottabad

Name; Prof.Dr.Johar Ali

Signature

Date;
BAHRIA UNIVERSITY ISLAMABAD

MULTIPLE CHOICE QUESTIONS

Program MBBS

Subject General Surgery

Prof Final

TOS ID 17.2.5

Description GIT ( Gall Bladder and bile duct )

stem A 40 year old multiparous fatty female, presented in surgical OPD with history of pain in
RHC for few hours associated with few episodes of non bilious vomiting. Examination
showed tachycardia and tenderness in RHC. What is likely cause

Item A Acute cholecystitis

Item B Acute pancreatitis

Item C Acute peritonitis

Item D Biliary colic

Item E Perforated duodenal ulcer

Correct answer A

Domain; Recall Interpretation Problem solving

Reference; Bailey and love Edition 26 page 1108

Prepared by;
College; Frontier Medical and Dental College, Abbottabad

Name; Prof.Dr.Johar Ali

Signature

Date;

BAHRIA UNIVERSITY ISLAMABAD

MULTIPLE CHOICE QUESTIONS

Program MBBS

Subject General Surgery

Prof Final

TOS ID 17.2.5

Description GIT ( Gall Bladder and bile duct )

Stem A 40 year old multiparous fatty female, presented in surgical OPD with history of pain in
RHC for few hours associated with few episodes of non bilious vomiting. Examination
showed tachycardia and tenderness in RHC. Which investigation will help in diagnosis

Item A CXR

Item B X Ray abdomen

Item C Ultra sound abdomen

Item D TLC
Item E Barium study

Correct answer C

Domain; Recall Interpretation Problem solving

Reference; Bailey and love Edition 26 page 1107

Prepared by;

College; Frontier Medical and Dental College, Abbottabad

Name; Prof.Dr.Johar Ali

Signature

Date;

BAHRIA UNIVERSITY ISLAMABAD

MULTIPLE CHOICE QUESTIONS

Program MBBS

Subject General Surgery

Prof Final

TOS ID 17.2.5
Description GIT ( Gall Bladder and bile duct )

stem A 35 year old female presented in OPD for complete medical examination. All
investigations were normal except for multiple small gall stones which were
asymptomatic. What is management options

Item A Open cholecystectomy

Item B Laparoscopic cholecystectomy

Item C Observation

Item D IV antibiotics

Item E IV analgesics

Correct answer C

Domain; Recall Interpretation Problem solving

Reference; Bailey and love Edition 26 page 1108

Prepared by;

College; Frontier Medical and Dental College, Abbottabad

Name; Prof.Dr.Johar Ali

Signature

Date;

BAHRIA UNIVERSITY ISLAMABAD


MULTIPLE CHOICE QUESTIONS

Program MBBS

Subject General Surgery

Prof Final

TOS ID 17.2.5

Description GIT ( Gall Bladder and bile duct )

stem A 35 year old female presented in OPD for complete medical examination. Her Random
blood sugar was 400 mg/dl with ++ glucose in urine and multiple small gall stones which
were asymptomatic. What is management options

Item A Open cholecystectomy

Item B Laparoscopic cholecystectomy

Item C Observation

Item D IV antibiotics

Item E IV analgesics

Correct answer B

Domain; Recall Interpretation Problem solving

Reference; Bailey and love Edition 26 page 1108

Prepared by;

College; Frontier Medical and Dental College, Abbottabad

Name; Prof.Dr.Johar Ali

Signature
Date;

BAHRIA UNIVERSITY ISLAMABAD

MULTIPLE CHOICE QUESTIONS

Program MBBS

Subject General Surgery

Prof Final

TOS ID 17.2.5

Description GIT ( Gall Bladder and bile duct )

Stem A 40 year old multiparous fatty female, presented in surgical OPD with signs of acute
cholecystitis. She was admitted and managed conservatively. When cholecystectomy
should be offered

Item A Immediate laparotomy

Item B Interval cholecystectomy

Item C After one year

Item D No need

Item E After 5 years

Correct answer B
Domain; Recall Interpretation Problem solving

Reference; Bailey and love Edition 26 page 1108

Prepared by;

College; Frontier Medical and Dental College, Abbottabad

Name; Prof.Dr.Johar Ali

Signature

Date;

BAHRIA UNIVERSITY ISLAMABAD

MULTIPLE CHOICE QUESTIONS

Program MBBS

Subject General Surgery

Prof Final

TOS ID 17.2.5

Description GIT ( Gall Bladder and bile duct )

Stem Most common type of gall stones is

Item A Pigment stones


Item B Cholesterol stones

Item C Mixed stones

Item D Brown

Item E Black

Correct answer B

Domain; Recall Interpretation Problem solving

Reference; Bailey and love Edition 26 page 1106

Prepared by;

College; Frontier Medical and Dental College, Abbottabad

Name; Prof.Dr.Johar Ali

Signature

Date;

BAHRIA UNIVERSITY ISLAMABAD

MULTIPLE CHOICE QUESTIONS

Program MBBS
Subject General Surgery

Prof Final

TOS ID 17.2.5

Description GIT ( Gall Bladder and bile duct )

Stem A 40 year old multiparous fatty female, presented in surgical OPD with signs of acute
cholecystitis. Which sign helps in clinical diagnosis

Item A Psoas sign

Item B Obturator sign

Item C Murphys sign

Item D Murphys punch

Item E Pointing sign

Correct answer C

Domain; Recall Interpretation Problem solving

Reference; Bailey and love Edition 26 page 1107

Prepared by;

College; Frontier Medical and Dental College, Abbottabad

Name; Prof.Dr.Johar Ali

Signature

Date;
BAHRIA UNIVERSITY ISLAMABAD

MULTIPLE CHOICE QUESTIONS

Program MBBS

Subject General Surgery

Prof Final

TOS ID 17.2.5

Description GIT ( Gall Bladder and bile duct )

Stem A patient presented in OPD with H/O jaundice for few days. She is a known case of gall
stones. Surgeon diagnosed her as case of obstructive jaundice. Which investigation will
help

Item A ALT

Item B AST

Item C Serum Bilirubin

Item D Alkaline phosphatase

Item E Serum amylase

Correct answer D

Domain; Recall Interpretation Problem solving

Reference; Bailey and love Edition 26 page 1111

Prepared by;
College; Frontier Medical and Dental College, Abbottabad

Name; Prof.Dr.Johar Ali

Signature

Date;

BAHRIA UNIVERSITY ISLAMABAD

MULTIPLE CHOICE QUESTIONS

Program MBBS

Subject General Surgery

Prof Final

TOS ID 17.2.5

Description GIT ( Gall Bladder and bile duct )

Stem A patient presented in OPD with H/O jaundice for few days. U/sound showed dilated CBD
& stone in lower CBD. What is treatment option

Item A Open cholecystectomy

Item B Laparoscopic cholecystectomy

Item C CBD exploration

Item D ERCP
Item E Exploratory Laparotomy

Correct answer D

Domain; Recall Interpretation Problem solving

Reference; Bailey and love Edition 26 page 1111

Prepared by;

College; Frontier Medical and Dental College, Abbottabad

Name; Prof.Dr.Johar Ali

Signature

Date;

BAHRIA UNIVERSITY ISLAMABAD

MULTIPLE CHOICE QUESTIONS

Program MBBS

Subject General Surgery

Prof Final

TOS ID 17.2.6

Description GIT ( Pancreas )

Stem Which of the following signs is considered to be positive in acute hemorrhagic


pancreatitis

Item A Grey Turners sign

Item B Boas sign

Item C Trousseaus’ sign

Item D Courvoiser’s sign

Item E Murphy’s sign

Correct answer A

Domain; Recall Interpretation Problem solving

Reference; Bailey and love Edition 26 page 1128

Prepared by;

College; Frontier Medical and Dental College, Abbottabad.

Name; Prof.Dr.Johar Ali

Signature

Date;

BAHRIA UNIVERSITY ISLAMABAD

MULTIPLE CHOICE QUESTIONS


Program MBBS

Subject General Surgery

Prof Final

TOS ID 17.2.6

Description GIT ( Pancreas )

Stem A 60 years old male presented in OPD with history of painless jaundice and itching for last
2-3 weeks. He has also lost some weight. Examination revealed jaundice, pallor and
palpable mass in epigastrium which is hard and fixed and resonant on percussion. What is
likely diagnosis

Item A Acute cholangitis

Item B Ca Head of pancreas

Item C Chronic pancreatitis

Item D Hepatocellular CA

Item E CA Stomach

Correct answer B

Domain; Recall Interpretation Problem solving

Reference; Bailey and love Edition 26 page 1137

Prepared by;

College; Frontier Medical and Dental College, Abbottabad

Name; Prof.Dr.Johar Ali

Signature
Date;

BAHRIA UNIVERSITY ISLAMABAD

MULTIPLE CHOICE QUESTIONS

Program MBBS

Subject General Surgery

Prof Final

TOS ID 17.2.6

Description GIT ( Pancreas )

Stem A 45 year old female presented in OPD with upper abdominal pain radiating to back for
last 3 days. She is a known case of gall stones. On examination abdomen is distended and
tender in epigastrium. S/lipase is increased. What is likely diagnosis

Item A Biliary colic

Item B Acute cholecystitis

Item C Acute pancreatitis

Item D Acute hepatitis

Item E Acute gastritis

Correct answer C

Domain; Recall Interpretation Problem solving


Reference; Bailey and love Edition 26 page 1127

Prepared by;

College; Frontier Medical and Dental College, Abbottabad

Name; Prof.Dr.Johar Ali

Signature

Date;

BAHRIA UNIVERSITY ISLAMABAD

MULTIPLE CHOICE QUESTIONS

Program MBBS

Subject General Surgery

Prof Final

TOS ID 17.2.6

Description GIT ( Pancreas)

stem A10 year old girl presented in surgical OPD with History of Upper abdominal pain and
swelling for few days. It is also associated with nausea and vomiting. Examination reveals
distended epigastrium with smooth mass with ill defined edges that are resonant on
percussion. What is likely diagnosis

Item A CA head of pancreas


Item B Acute Cholecystitis

Item C CA Stomach

Item D Pancreatic Pseudocyst

Item E Empyema Gall bladder

Correct answer D

Domain; Recall Interpretation √ Problem solving

Reference; Bailey and love Edition 26 page 1133

Prepared by;

College; Frontier Medical and Dental College, Abbottabad

Name; Prof.Dr.Johar Ali

Signature

Date;

BAHRIA UNIVERSITY ISLAMABAD

MULTIPLE CHOICE QUESTIONS

Program MBBS
Subject General Surgery

Prof Final

TOS ID 17.2.6

Description GIT ( Pancreas)

stem Ectopic pancreatic tissue can be seen in which of the following

Item A Esophagus

Item B Tonsils

Item C Cecum

Item D Meckels diverticulum

Item E Lesser sac

Correct answer D

Domain; Recall √ Interpretation Problem solving

Reference; Bailey and love Edition 26 page 1125

Prepared by;

College; Frontier Medical and Dental College, Abbottabad

Name; Prof.Dr.Johar Ali

Signature

Date;
BAHRIA UNIVERSITY ISLAMABAD

MULTIPLE CHOICE QUESTIONS

Program MBBS

Subject General Surgery

Prof Final

TOS ID 17.2.6

Description GIT (Pancreas)

stem A 58year old female was diagnosed as case of acute pancreatitis. Investigations were
TLC 17000, BSR 250 mg/dl, LDH 720, AST 150, S/Ca 1.5 mmol/L and oxygen saturation 75
%. What is Ranson score

Item A 2

Item B 3

Item C 4

Item D 5

Item E 6

Correct answer D

Domain; Recall Interpretation Problem solving √

Reference; Bailey and love Edition 26 page 1129


Prepared by;

College; Frontier Medical and Dental College, Abbottabad

Name; Prof.Dr.Johar Ali

Signature

Date;

BAHRIA UNIVERSITY ISLAMABAD

MULTIPLE CHOICE QUESTIONS

Program MBBS

Subject General Surgery

Prof Final

TOS ID 17.2.6

Description GIT ( Pancreas)

stem A 58year old female was diagnosed as case of acute pancreatitis. Which of the following
criteria is most effective in determining severity index

Item A Glasgow

Item B Ranson

Item C APACHE
Item D POSSUM

Item E Parsonet

Correct answer C

Domain; Recall √ Interpretation Problem solving

Reference; Bailey and love Edition 26 page 1128

Prepared by;

College; Frontier Medical and Dental College, Abbottabad

Name; Prof.Dr.Johar Ali

Signature

Date;

BAHRIA UNIVERSITY ISLAMABAD

MULTIPLE CHOICE QUESTIONS

Program MBBS

Subject General Surgery

Prof Final
TOS ID 17.2.8

Description GIT ( Small and large intestine)

Stem A 60 year old female complains of feeling tired while doing routine household work for
last 8 weeks. Examination showed pallor and mass in RIF. Her Hb was 7.5 gm/dl. What is
likely diagnosis

Item A CA Stomach

Item B Appendicular mass

Item C Rectus sheath hematoma

Item D CA Cecum

Item E Mittleschemerz

Correct answer D

Domain; Recall Interpretation √ Problem solving

Reference; Bailey and love Edition 26 page 1166

Prepared by;

College; Frontier Medical and Dental College, Abbottabad

Name; Prof.Dr.Johar Ali

Signature

Date;

BAHRIA UNIVERSITY ISLAMABAD

MULTIPLE CHOICE QUESTIONS


Program MBBS

Subject General Surgery

Prof Final

TOS ID 17.2.8

Description GIT ( Small and large intestine)

Stem A young patient presented in OPD having purulent discharge from peri anal region for last
3 months. He also gave history of fistulotomy 1 year ago. He is also having loose motions
containing blood and mucous. What is most likely cause

Item A Diverticular disease

Item B CA rectum

Item C Vesicorectal fistula

Item D Crohns disease

Item E Gluteal abscess

Correct answer D

Domain; Recall Interpretation √ Problem solving

Reference; Bailey and love Edition 26 page 1152

Prepared by;

College; Frontier Medical and Dental College, Abbottabad

Name; Prof.Dr.Johar Ali

Signature

Date;
BAHRIA UNIVERSITY ISLAMABAD

MULTIPLE CHOICE QUESTIONS

Program MBBS

Subject General Surgery

Prof Final

TOS ID 17.2.8

Description GIT ( Small and large intestine)

Stem A 65 year old male presents in surgical OPD complaining of passage of flatus and stools in
urine. What is pathology

Item A Vesicourethral fistula

Item B Enteroenteral fistula

Item C Enterovesical fistula

Item D Enterocutaneous fistula

Item E Vesicocutaneous fistula

Correct answer D

Domain; Recall Interpretation √ Problem solving


Reference; Bailey and love Edition 26 page 1170

Prepared by;

College; Frontier Medical and Dental College, Abbottabad

Name; Prof.Dr.Johar Ali

Signature

Date;

BAHRIA UNIVERSITY ISLAMABAD

MULTIPLE CHOICE QUESTIONS

Program MBBS

Subject General Surgery

Prof Final

TOS ID 17.2.8

Description GIT ( Small and large intestine)

Stem Intestinal TB affects which part of gut commonly

Item A Stomach

Item B DJ junction
Item C Ileocecal junction

Item D Transverse colon

Item E Sigmoid colon

Correct answer C

Domain; Recall √ Interpretation Problem solving

Reference; Bailey and love Edition 26 page 1158

Prepared by;

College; Frontier Medical and Dental College, Abbottabad

Name; Prof.Dr.Johar Ali

Signature

Date;

BAHRIA UNIVERSITY ISLAMABAD

MULTIPLE CHOICE QUESTIONS

Program MBBS

Subject General Surgery

Prof Final
TOS ID 17.2.12

Description GIT ( Anus and anal canal)

Stem A young patient presented in OPD having purulent discharge from peri anal region for last
3 months. Examination revealed swelling at 9 O clock positions with signs of inflammation.
What is likely diagnosis

Item A Peri anal fistula

Item B Anal Fissure

Item C Hemorrhoids

Item D Peri anal abscess

Item E Rectal cancer

Correct answer D

Domain; Recall Interpretation √ Problem solving

Reference; Bailey and love Edition 26 page 1259

Prepared by;

College; Frontier Medical and Dental College, Abbottabad

Name; Prof.Dr.Johar Ali

Signature

Date;

BAHRIA UNIVERSITY ISLAMABAD


MULTIPLE CHOICE QUESTIONS

Program MBBS

Subject General Surgery

Prof Final

TOS ID 17.2.9

Description GIT ( Intestinal Obstruction)

Stem Which of the following is type of adynamic intestinal obstruction

Item A Adhesions

Item B Paralytic ileus

Item C Worms

Item D Stones

Item E Tumor

Correct answer B

Domain; Recall √ Interpretation Problem solving

Reference; Bailey and love Edition 26 page 1193

Prepared by;

College; Frontier Medical and Dental College, Abbottabad

Name; Prof.Dr.Johar Ali

Signature

Date;
BAHRIA UNIVERSITY ISLAMABAD

MULTIPLE CHOICE QUESTIONS

Program MBBS

Subject General Surgery

Prof Final

TOS ID 17.2.9

Description GIT (Intestinal obstruction)

Stem Intusussception most commonly occurs in which age group

Item A < 4 months

Item B 5-10 months

Item C 1-2 year

Item D 2-5 year

Item E > 5 year

Correct answer B

Domain; Recall √ Interpretation Problem solving


Reference; Bailey and love Edition 26 page 114

Prepared by;

College; Frontier Medical and Dental College, Abbottabad

Name; Prof.Dr.Johar Ali

Signature

Date;

BAHRIA UNIVERSITY ISLAMABAD

MULTIPLE CHOICE QUESTIONS

Program MBBS

Subject General Surgery

Prof Final

TOS ID 17.2.9

Description GIT ( Intestinal obstruction)

Stem A 12 year old female presents in OPD with abdominal mass and vomiting for last few days.
Examination reveals distended epigastrium with firm mass that is mobile and non
Epigastric hernia tender. What is likely diagnosis

Item A Undigested fruit and vegetables

Item B Hairs
Item C Congenital Hypertrophic Pyloric stenosis

Item D Worms

Item E Annular Pancreas

Correct answer B

Domain; Recall Interpretation √ Problem solving

Reference; Bailey and love Edition 26 page 1183

Prepared by;

College; Frontier Medical and Dental College, Abbottabad

Name; Prof.Dr.Johar Ali

Signature

Date;

BAHRIA UNIVERSITY ISLAMABAD

MULTIPLE CHOICE QUESTIONS

Program MBBS

Subject General Surgery


Prof Final

TOS ID 17.2.10

Description GIT ( Appendix)

Stem A 5 year old female was diagnosed as having acute appendicitis. On examination when
surgeon presses LIF patient experiences pain in RIF. What is sign called

Item A Pointing

Item B Rovsings

Item C Psoas

Item D Obturator

Item E Murphy’s

Correct answer B

Domain; Recall Interpretation √ Problem solving

Reference; Bailey and love Edition 26 page 1203

Prepared by;

College; Frontier Medical and Dental College, Abbottabad

Name; Prof.Dr.Johar Ali

Signature

Date;
BAHRIA UNIVERSITY ISLAMABAD

MULTIPLE CHOICE QUESTIONS

Program MBBS

Subject General Surgery

Prof Final

TOS ID 17.2.10

Description GIT ( appendix)

stem A 15 year old female presented in OPD having pain in Umbilicus that radiated to RIF. She is
also having nausea and anorexia. TLC was 11500.On examination when surgeon presses
LIF patient experiences pain in RIF and also tenderness and rebound tenderness in RIF.
What is Alvarado score

Item A 2

Item B 3

Item C 5

Item D 7

Item E 8

Correct answer E

Domain; Recall Interpretation Problem solving √

Reference; Bailey and love Edition 26 page 1207

Prepared by;
College; Frontier Medical and Dental College, Abbottabad

Name; Prof.Dr.Johar Ali

Signature

Date;

BAHRIA UNIVERSITY ISLAMABAD

MULTIPLE CHOICE QUESTIONS

Program MBBS

Subject General Surgery

Prof Final

TOS ID 17.2.10

Description GIT ( Appendix)

Stem A 15 year old female presented in OPD having pain in Umbilicus that radiated to RIF. Her
TLC was 7500.On examination when surgeon presses LIF patient experiences pain in RIF.
There was also tenderness and rebound tenderness in RIF. What is Alvarado score

Item A 2

Item B 3

Item C 4

Item D 5

Item E 6

Correct answer C
Domain; Recall Interpretation Problem solving √

Reference; Bailey and love Edition 26 page 1207

Prepared by;

College; Frontier Medical and Dental College, Abbottabad

Name; Prof.Dr.Johar Ali

Signature

Date;

BAHRIA UNIVERSITY ISLAMABAD

MULTIPLE CHOICE QUESTIONS

Program MBBS

Subject General Surgery

Prof Final

TOS ID 17.2.10

Description GIT ( Appendix)

Stem A 20 year old female presented on surgical OPD having pain in RIF for few hours. Pain
started suddenly and radiates in lower abdomen. Her LPMP was 2 weeks ago. Examination
was unremarkable. What is likely cause
Item A Diverticulitis

Item B Mittleschemerz

Item C Perforated peptic ulcer

Item D Ruptured ectopic pregnancy

Item E CA Cecum

Correct answer B

Domain; Recall Interpretation √ Problem solving

Reference; Bailey and love Edition 26 page 1205

Prepared by;

College; Frontier Medical and Dental College, Abbottabad

Name; Prof.Dr.Johar Ali

Signature

Date;

BAHRIA UNIVERSITY ISLAMABAD

MULTIPLE CHOICE QUESTIONS


Program MBBS

Subject General Surgery

Prof Final

TOS ID 17.2.10

Description GIT (Appendix)

Stem A 15 year old female presented in OPD having pain in Umbilicus that radiated to RIF. She
is also having nausea and anorexia. TLC was 11500.On examination when surgeon presses
LIF patient experiences pain in RIF and also tenderness and rebound tenderness in RIF.
What is likely diagnosis

Item A Acute Appendicitis

Item B Right ureteric colic

Item C Ruptured ectopic pregnancy

Item D Mittleschemerz

Item E Perforated peptic ulcer

Correct answer A

Domain; Recall Interpretation √ Problem solving

Reference; Bailey and love Edition 26 page 1203

Prepared by;

College; Frontier Medical and Dental College, Abbottabad

Name; Prof.Dr.Johar Ali

Signature

Date;
BAHRIA UNIVERSITY ISLAMABAD

MULTIPLE CHOICE QUESTIONS

Program MBBS

Subject General Surgery

Prof Final

TOS ID 17.2.10

Description GIT (Appendix)

stem A 15 year old female presented in OPD having pain in Umbilicus that radiated to RIF. Her
TLC was 7500.On examination when surgeon presses LIF patient experiences pain in RIF.
There was also tenderness and rebound tenderness in RIF. What is treatment

Item A No treatment

Item B Conservative management

Item C Appendicectomy

Item D Exploratory Laparotomy

Item E Right Hemicolectomy

Correct answer A

Domain; Recall Interpretation √ Problem solving


Reference; Bailey and love Edition 26 page 1207

Prepared by;

College; Frontier Medical and Dental College, Abbottabad

Name; Prof.Dr.Johar Ali

Signature

Date;

BAHRIA UNIVERSITY ISLAMABAD

MULTIPLE CHOICE QUESTIONS

Program MBBS

Subject General Surgery

Prof Final

TOS ID 17.2.10

Description GIT (Appendix)

stem An 18 year old female complains of generalized colicky abdominal pain for few hours. She
is anorexic and also has vomited for couple of times. Examination reveals tenderness and
rebound tenderness in RIF. What is treatment option

Item A No treatment

Item B Conservative management

Item C Appendicectomy

Item D Exploratory Laparotomy

Item E Right Hemicolectomy


Correct answer C

Domain; Recall Interpretation Problem solving √

Reference; Bailey and love Edition 26 page 1208

Prepared by;

College; Frontier Medical and Dental College, Abbottabad

Name; Prof.Dr.Johar Ali

Signature

Date;

BAHRIA UNIVERSITY ISLAMABAD

MULTIPLE CHOICE QUESTIONS

Program MBBS

Subject General Surgery

Prof Final

TOS ID 17.2.10

Description GIT (Appendix)


stem An 18 year old female complains of generalized colicky abdominal pain for few hours. She
is anorexic and also has vomited for couple of times. Examination reveals tenderness and
rebound tenderness in RIF. What is treatment option

Item A No treatment

Item B Conservative management

Item C Appendicectomy

Item D Exploratory Laparotomy

Item E Right Hemicolectomy

Correct answer C

Domain; Recall Interpretation Problem solving √

Reference; Bailey and love Edition 26 page 1208

Prepared by;

College; Frontier Medical and Dental College, Abbottabad

Name; Prof.Dr.Johar Ali

Signature

Date;

BAHRIA UNIVERSITY ISLAMABAD


MULTIPLE CHOICE QUESTIONS

Program MBBS

Subject General Surgery

Prof Final

TOS ID 17.2.10

Description GIT ( Appendix)

stem A 30 year old male was admitted with appendicular mass. After 24 hours of conservative
management he developed high grade fever and increase in size of mass. What has
happened

Item A Carcinoid tumor

Item B CA Cecum

Item C Appendicular Abscess

Item D Generalized Peritonitis

Item E Maloney gangrene

Correct answer C

Domain; Recall Interpretation Problem solving√

Reference; Bailey and love Edition 26 page 1211

Prepared by;

College; Frontier Medical and Dental College, Abbottabad

Name; Prof.Dr.Johar Ali


Signature

Date;

BAHRIA UNIVERSITY ISLAMABAD

MULTIPLE CHOICE QUESTIONS

Program MBBS

Subject General Surgery

Prof Final

TOS ID 17.2.11

Description GIT ( Rectum)

Stem A 65 year old male complains of early morning diarrhea for last 4 months. He is also having
tenesmus and bleeding per rectum. He has also lost some weight. What is likely cause

Item A 2nd degree Hemorrhoids

Item B Anal Fissure

Item C CA Rectum

Item D CA Cecum

Item E Diverticular disease

Correct answer C
Domain; Recall Interpretation √ Problem solving

Reference; Bailey and love Edition 26 page 1227

Prepared by;

College; Frontier Medical and Dental College, Abbottabad

Name; Prof.Dr.Johar Ali

Signature

Date;

BAHRIA UNIVERSITY ISLAMABAD

MULTIPLE CHOICE QUESTIONS

Program MBBS

Subject General Surgery

Prof Final

TOS ID 17.2.11

Description GIT (Rectum)

Stem A 62 year old male complains of early morning diarrhea for last 4 months. He is also
having tenesmus and bleeding per rectum. He has also lost some weight. Which anterior
examination will help in diagnosis
Item A DPL

Item B Anoscopy

Item C Thoracoscopy

Item D Sigmoidoscopy

Item E Gastro-duodenoscopy

Correct answer D

Domain; Recall Interpretation Problem solving √

Reference; Bailey and love Edition 26 page 1228

Prepared by;

College; Frontier Medical and Dental College, Abbottabad

Name; Prof.Dr.Johar Ali

Signature

Date;

BAHRIA UNIVERSITY ISLAMABAD

MULTIPLE CHOICE QUESTIONS


Program MBBS

Subject General Surgery

Prof Final

TOS ID 17.2.11

Description GIT (Rectum)

Stem A 62 year old male complains of early morning diarrhea for last 4 months. He is also having
tenesmus and bleeding per rectum. He has also lost some weight. Sigmoidoscopy showed
growth in upper part of rectum. Which investigation will stage the disease

Item A U/Sound abdomen

Item B CXR (PA view)

Item C CT Scan Abdomen and pelvis

Item D Angiography

Item E MRCP

Correct answer C

Domain; Recall Interpretation Problem solving√

Reference; Bailey and love Edition 26 page 1228

Prepared by;

College; Frontier Medical and Dental College, Abbottabad

Name; Prof.Dr.Johar Ali

Signature

Date;
BAHRIA UNIVERSITY ISLAMABAD

MULTIPLE CHOICE QUESTIONS

Program MBBS

Subject General Surgery

Prof Final

TOS ID 17.2.11

Description GIT (Rectum)

stem A mother brought her 4 year old boy with history of fresh bleeding per rectum and
something coming out of rectum. What is likely cause

Item A Anal Fissure

Item B Hemorrhoids

Item C Rectal polyp

Item D CA Rectum

Item E Proctitis

Correct answer C

Domain; Recall Interpretation √ Problem solving

Reference; Bailey and love Edition 26 page 1224

Prepared by;

College; Frontier Medical and Dental College, Abbottabad

Name; Prof.Dr.Johar Ali

Signature
Date;

BAHRIA UNIVERSITY ISLAMABAD

MULTIPLE CHOICE QUESTIONS

Program MBBS

Subject General Surgery

Prof Final

TOS ID 17.2.11

Description GIT (Anus and Anal canal)

Stem A 28 year old male presented with severe anal pain during defecation. He has also streaks
of fresh blood per rectum and history of chronic constipation. What is likely diagnosis

Item A Hemorrhoids

Item B Proctitis

Item C Anal Fissure

Item D Diverticulitis

Item E Ischemic Colitis


Correct answer C

Domain; Recall Interpretation √ Problem solving

Reference; Bailey and love Edition 26 page 1248

Prepared by;

College; Frontier Medical and Dental College, Abbottabad

Name; Prof.Dr.Johar Ali

Signature

Date;

BAHRIA UNIVERSITY ISLAMABAD

MULTIPLE CHOICE QUESTIONS

Program MBBS

Subject General Surgery

Prof Final

TOS ID 17.2.11

Description GIT (Anus and Anal canal)


Stem A 45 year old female presented in surgical OPD complaining of bleeding per rectum with
something coming out of rectum that needs manual reduction. She is also having chronic
constipation. What is likely cause

Item A Anal fissure

Item B Hemorrhoids

Item C CA Rectum

Item D Colonic polyp

Item E Anal wart

Correct answer B

Domain; Recall Interpretation √ Problem solving

Reference; Bailey and love Edition 26 page 1251

Prepared by;

College; Frontier Medical and Dental College, Abbottabad

Name; Prof.Dr.Johar Ali

Signature

Date;

BAHRIA UNIVERSITY ISLAMABAD

MULTIPLE CHOICE QUESTIONS


Program MBBS

Subject General Surgery

Prof Final

TOS ID 17.2.11

Description GIT (Anus and Anal canal)

Stem A 45 year old female presented in surgical OPD complaining of bleeding per rectum with
something coming out of rectum that needs manual reduction. She is also having chronic
constipation. What is degree of hemorrhoids

Item A Ist Degree

Item B 2nd degree

Item C 3rd degree

Item D 4th degree

Item E 5th degree

Correct answer C

Domain; Recall Interpretation √ Problem solving

Reference; Bailey and love Edition 26 page 1252

Prepared by;

College; Frontier Medical and Dental College, Abbottabad

Name; Prof.Dr.Johar Ali

Signature
Date;

BAHRIA UNIVERSITY ISLAMABAD

MULTIPLE CHOICE QUESTIONS

Program MBBS

Subject General Surgery

Prof Final

TOS ID 17.2.11

Description GIT (Anus and Anal canal)

Stem A 45 year old female presented in surgical OPD complaining of bleeding per rectum with
something coming out of rectum that needs manual reduction. She is also having chronic
constipation. What is treatment option

Item A Conservative management

Item B Inj Sclerotherapy

Item C Rubber band ligation

Item D Hemorrhoidectomy

Item E Thiersch stitch

Correct answer D

Domain; Recall Interpretation Problem solving √


Reference; Bailey and love Edition 26 page 1253

Prepared by;

College; Frontier Medical and Dental College, Abbottabad

Name; Prof.Dr.Johar Ali

Signature

Date;

Program MBBS

Subject General Surgery

Prof Final

TOS ID 17.2.11

Description GIT (Anus and Anal canal)

Stem A 45 year old female presented in surgical OPD complaining of bleeding per rectum with
something coming out of rectum that reduces spontaneously. She is also having chronic
constipation. What is treatment option

Item A Conservative management

Item B Inj Sclerotherapy

Item C Rubber band ligation

Item D Hemorrhoidectomy

Item E Thiersch stitch

Correct answer C
Domain; Recall Interpretation Problem solving √

Reference; Bailey and love Edition 26 page 1253

Prepared by;

College; Frontier Medical and Dental College, Abbottabad

Name; Prof.Dr.Johar Ali

Signature

Date;

Program MBBS

Subject General Surgery

Prof Final

TOS ID 17.2.11

Description GIT (Anus and Anal canal)

Stem A 45 year old female presented in surgical OPD complaining of bleeding per rectum and
nothing coming out of rectum. She is also having chronic constipation. Proctoscopy
showed 1st degree hemorrhoids. What is treatment option

Item A Lords operation

Item B Inj Sclerotherapy


Item C Rubber band ligation

Item D Hemorrhoidectomy

Item E Thiersch stitch

Correct answer B

Domain; Recall Interpretation Problem solving √

Reference; Bailey and love Edition 26 page 1252

Prepared by;

College; Frontier Medical and Dental College, Abbottabad

Name; Prof.Dr.Johar Ali

Signature

Date;

Program MBBS

Subject General Surgery

Prof Final
TOS ID 17.2.11

Description GIT (Anus and Anal canal)

Stem A 28 year old male presented with severe anal pain during defecation. He has also streaks
of fresh blood per rectum and history of chronic constipation. What is treatment option

Item A Lords Operation

Item B Inj Sclerotherapy

Item C Lateral internal sphincterotomy

Item D Hemorrhoidectomy

Item E Thiersch stitch

Correct answer C

Domain; Recall Interpretation Problem solving √

Reference; Bailey and love Edition 26 page 1249

Prepared by;

College; Frontier Medical and Dental College, Abbottabad

Name; Prof.Dr.Johar Ali

Signature

Date;
Program MBBS

Subject General Surgery

Prof Final

TOS ID 17.2.11

Description GIT (Anus and Anal canal)

Stem A 45 year old female presented in surgical OPD complaining of bleeding per rectum with
something coming out of rectum that needs manual reduction. She is also having chronic
constipation. What is treatment option

Item A Conservative management

Item B Inj Sclerotherapy

Item C Rubber band ligation

Item D Hemorrhoidectomy

Item E Thiersch stitch

Correct answer D

Domain; Recall Interpretation Problem solving √

Reference; Bailey and love Edition 26 page 1253

Prepared by;

College; Frontier Medical and Dental College, Abbottabad

Name; Prof.Dr.Johar Ali

Signature

Date;
Program MBBS

Subject General Surgery

Prof Final

TOS ID 17.2.11

Description GIT (Anus and Anal canal)

Stem A 28 year old male presented with severe anal pain during defecation. She was operated
for anal fissure. Post operative she developed flatus incontinence. Which procedure was
done

Item A Lateral internal sphincterotomy

Item B Inj Sclerotherapy

Item C Rubber band ligation

Item D Hemorrhoidectomy

Item E Thiersch stitch

Correct answer A

Domain; Recall Interpretation Problem solving √


Reference; Bailey and love Edition 26 page 1249

Prepared by;

College; Frontier Medical and Dental College, Abbottabad

Name; Prof.Dr.Johar Ali

Signature

Date;

Program MBBS

Subject General Surgery

Prof Final

TOS ID 17.2.11

Description GIT (Anus and Anal canal)

Stem A 45 year old female presented in surgical OPD complaining of bleeding per rectum with
something coming out of rectum that needs manual reduction. She is also having chronic
constipation. Which of the following will help in diagnosis

Item A Ultra sound pelvis

Item B Sinogram

Item C Colonoscope

Item D Proctoscope
Item E MRI

Correct answer D

Domain; Recall Interpretation Problem solving √

Reference; Bailey and love Edition 26 page 1240

Prepared by;

College; Frontier Medical and Dental College, Abbottabad

Name; Prof.Dr.Johar Ali

Signature

Date;
Program MBBS

Subject General Surgery

Prof Final

TOS ID 17.4.1

Description Thyroid

Stem Which of the following is commonest thyroid malignancy

Item A Papillary thyroid carcinoma

Item B Medullary thyroid carcinoma

Item C Anaplastic carcinoma

Item D Thyroid lymphoma

Item E Metastatic carcinoma

Correct answer A

Domain; Recall √ Interpretation Problem solving

Reference; Bailey and love Edition 26 page763

Prepared by;

College; Frontier Medical and Dental College, Abbottabad.

Name;

Signature

Date;
Program MBBS

Subject General Surgery

Prof Final

TOS ID 17.4.1

Description Thyroid

Stem A 2 year old boy was brought to OPD for a mid line cystic neck swelling lying just below
hyoid bone that moves with swallowing & tongue protrusion. What is likely diagnosis

Item A Branchial cyst

Item B Cervical cyst

Item C Thyroglossal cyst

Item D Cervical cyst

Item E Dermoid cyst

Correct answer C

Domain; Recall Interpretation √ Problem solving

Reference; Bailey and love Edition 26 page 752


Prepared by;

College; Frontier Medical and Dental College, Abbottabad

Name;

Signature

Date;

Program MBBS

Subject General Surgery

Prof Final

TOS ID 17.4.1

Description Thyroid

Stem A 40 year old lady was operated for simple MNG and subtotal thyroidectomy was done.
On 1st post operative day she felt perioral numbness and tingling in hands and feet. Which
complication has occurred

Item A Damage to recurrent laryngeal nerve

Item B Damage to external laryngeal nerve

Item C Hypocalcemia
Item D Hypercalcemia

Item E Thyroid failure

Correct answer C

Domain; Recall Interpretation√ Problem solving

Reference; Bailey and love Edition 26 page 762

Prepared by;

College; Frontier Medical and Dental College, Abbottabad

Name;

Signature

Date;

Program MBBS

Subject General Surgery

Prof Final
TOS ID 17.4.1

Description Thyroid

Stem A 40 year old lady singer by profession was offered subtotal thyroidectomy for simple
MNG. Regarding informed consent which of the following post operative complication she
should be aware of

Item A Hoarseness of voice

Item B Decrease in pitch

Item C Hypothyroidism

Item D Hypocalcemia

Item E Thyroid storm

Correct answer B

Domain; Recall Interpretation √ Problem solving

Reference; Bailey and love Edition 26 page 762

Prepared by;

College; Frontier Medical and Dental College, Abbottabad

Name;

Signature

Date;
Program MBBS

Subject General Surgery

Prof Final

TOS ID 17.4.1

Description Thyroid

Stem A 44 year old lady was operated for simple MNG and subtotal thyroidectomy was done.
On 3rd post operative day she felt that pitch of her voice is decreased. Which complication
has occurred

Item A Damage to External Laryngeal nerve

Item B Damage to Recurrent Laryngeal nerve

Item C Damage to main vagal trunk

Item D Damage to hypoglossal nerve

Item E Hypocalcemia

Correct answer A

Domain; Recall Interpretation √ Problem solving

Reference; Bailey and love Edition 26 page 762

Prepared by;

College; Frontier Medical and Dental College, Abbottabad

Name;

Signature

Date;
Program MBBS

Subject General Surgery

Prof Final

TOS ID 17.4.3

Description Salivary gland

Stem A young female presented in surgical OPD having a painful swelling in front of left ear
which is non tender, firm, slightly mobile. What is likely diagnosis

Item A Mumps

Item B Pleomorphic adenoma

Item C Parotid carcinoma

Item D Parotid abscess

Item e Metastatic lesion in parotid

Correct answer B

Domain; Recall Interpretation √ Problem solving

Reference; Bailey and love Edition 26 page 742

Prepared by;

College; Frontier Medical and Dental College, Abbottabad

Name;

Signature
Date;

Program MBBS

Subject General Surgery

Prof Final

TOS ID 17.4.3

Description Salivary glands

Stem A young boy developed painful bilateral parotid swelling. Which of the following
complication can occur in this patient

Item A Salpingitis

Item B Orchitis

Item C Typhlitis

Item D Proctitis

Item E Oophoritis

Correct answer B

Domain; Recall √ Interpretation Problem solving


Reference; Bailey and love Edition 26 page 744

Prepared by;

College; Frontier Medical and Dental College, Abbottabad

Name;

Signature

Date;

Program MBBS

Subject General Surgery

Prof Final

TOS ID 17.4.3

Description Salivary glands

Stem Which of the following is common cause of bacterial sialadenitis

Item A Streptococcal pyogenes

Item B Atypical mycobacteria

Item C Staph aureus


Item D Pseudomonas aeruginosa

Item E klebsiella

Correct answer C

Domain; Recall √ Interpretation Problem solving

Reference; Bailey and love Edition 26 page 741

Prepared by;

College; Frontier Medical and Dental College, Abbottabad

Name;

Signature

Date;
Program MBBS

Subject General Surgery

Prof Final

TOS ID 17.4.3

Description Salivary glands

Stem Which of the following nerve is considered to be more vulnerable to injury during parotid
surgery

Item A Hypoglossal nerve

Item B Glossopharyngeal nerve

Item C Fascial nerve

Item D Vagus nerve

Item D Lingual nerve

Correct answer C

Domain; Recall √ Interpretation Problem solving

Reference; Bailey and love Edition 26 page 747

Prepared by;

College; Frontier Medical and Dental College, Abbottabad.

Name;

Signature

Date;
Program MBBS

Subject General Surgery

Prof Final

TOS ID 17.4.3

Description Salivary glands

Stem Following superficial parotidectomy patient complains of numbness in pinna of ear.


Which nerve is damaged

Item A Hypoglossal nerve

Item B Glossopharyngeal nerve

Item C Fascial nerve

Item D Great auricular nerve

Item E Vagus nerve

Correct answer D

Domain; Recall Interpretation √ Problem solving

Reference; Bailey and love Edition 26 page 747

Prepared by;

College; Frontier Medical and Dental College, Abbottabad


Name;

Signature

Date;
Program MBBS

Subject General Surgery

Prof Final

TOS ID 17.7

Description Breast

Stem A 30 year old woman complains of pain in right breast for 4 days. Examination reveals hot, tender
area in upper outer quadrant that is fluctuant. She has Cessarean section 3 weeks ago. What is likely
diagnosis

Item A Fibroadenoma

Item B Traumatic fat necrosis

Item C Breast abscess

Item D CA Breast

Item D Phylloides tumor

Correct answer C

Domain; Recall Interpretation √ Problem solving

Reference; Bailey and love Edition 26 page 803

Prepared by;

College; Frontier Medical and Dental College, Abbottabad.

Name;

Signature

Date;
Program MBBS

Subject General Surgery

Prof Final

TOS ID 17.7

Description Breast

Stem Which of the following has increased risk of developing breast carcinoma

Item A Breast cyst

Item B Breast abscess

Item C Fibroadenoma

Item D Duct hyperplasia

Item D Galactocele

Correct answer D

Domain; Recall √ Interpretation Problem solving

Reference; Bailey and love edition 26 page 809

Prepared by;

College; Frontier Medical and Dental College, Abbottabad.

Name;
Signature

Date;

Program MBBS

Subject General Surgery

Prof Final

TOS ID 17.7

Description Breast

Stem A 50 year old female presents in surgical clinic having lump in right breast. Lump is 3×3 cm
occupying upper outer quadrant, firm, mobile and non tender and there is no lymphadenopathy in
axilla. What is likely stage

Item A Stage I

Item B Stage II

Item C Stage III

Item D Stage IV

Item D Stage V

Correct answer B

Domain; Recall Interpretation √ Problem solving


Reference; Bailey and love Edition 25 page 800

Prepared by;

College; Frontier Medical and Dental College, Abbottabad.

Name;

Signature

Date;
Program MBBS

Subject General Surgery

Prof Final

TOS ID 17.7

Description Breast

Stem A 56 year old female presents in surgical clinic having lump in right breast. Lump is 5×4cm
occupying upper outer quadrant, firm to hard, fixed and also has ipsilateral fixed lymph nodes. What
is stage of tumor

Item A Stage I

Item B Stage II

Item C Stage III

Item D Stage IV

Item D Stage V

Correct answer C

Domain; Recall Interpretation √ Problem solving

Reference; Bailey and love Edition 25 page 800

Prepared by;

College; Frontier Medical and Dental College, Abbottabad

Name;

Signature

Date;
Program MBBS

Subject General Surgery

Prof Final

TOS ID 17.7

Description Breast

Stem A 50 year old female presents in surgical clinic having lump in right breast. Lump is 3×3 cm
occupying upper outer quadrant, firm, mobile and non tender and there is no lymphadenopathy in
axilla. Which investigation will help in diagnosis

Item A CXR

Item B Ultra sound Chest

Item C FNAC

Item D CT scan

Item D Open biopsy

Correct answer C

Domain; Recall Interpretation Problem solving √

Reference; Bailey and love Edition 26 page 800

Prepared by;
College; Frontier Medical and Dental College, Abbottabad.

Name;

Signature

Date;

Multiple Choices Question


PROGRAM MBBS
SUBJECT Surgery
PROF Annual/Supply 2015
TOS – ID 17.2 & 17.3

Description Upper & Lower GIT


A 41 year old man complains of regurgitation of saliva and of undigested food. An
STEM esophagram reveals a bird’s-beak deformity. Which of the following statements is true
about this condition?

ITEM A Chest pain is common in the advanced stages of this disease


ITEM B More patients are improved by forceful dilation than by surgical intervention
ITEM C Manometry can be expected to show high resting pressures of the lower esophageal
sphincter (Les)
ITEM D Surgical treatment consists primarily of resection of the distal esophagus with
reanastomosis to the stomach above the diaphragm
ITEM E Patients with this disease are at no increased risk for the development of
Carcinoma

Correct Answer C

Domain: Recall [] Interpretation [] Problem solving []


Reference(s) Bailey & Love
Prepared by
College Frontier Medical & Dental College, Abbottabad

Name:

Signature: –––––––––––––––––––––––––––––––

Date:

Note: Diagrams/Charts/Graphs if any should attach along this

BEHRIA UNIVERSITY ISLAMABAD


Multiple Choices Question
PROGRAM MBBS
SUBJECT Surgery
PROF Annual/Supply 2015
TOS – ID 17.2 & 17.3

Description Upper & Lower GIT


A 62 year old man has been noticing kprogressive difficulty swallowing, first solid food
STEM and now liquids as well. A barium study shows a ragged narrowing just below the cannal
level. Endoscopic biopsy confirms squamous cell carcinoma. Which of the following
statements concerning carcinoma of the esophagus is true?

ITEM A Alcohol has not been implicated as a precipitating factor


ITEM B Squamous carcinoma is the most common type at the cardioesophageal junction
ITEM C It has a higher incidence in females
ITEM D It occurs more commonly
ITEM E The standard of care is radiation herapy

Correct Answer D

Domain: Recall [] Interpretation [] Problem solving []

Reference(s) Bailey & Love


Prepared by
College Frontier Medical & Dental College, Abbottabad
Name:

Signature: –––––––––––––––––––––––––––––––

Date:

Note: Diagrams/Charts/Graphs if any should attach along this

BEHRIA UNIVERSITY ISLAMABAD


Multiple Choices Question
PROGRAM MBBS
SUBJECT Surgery
PROF Annual/Supply 2015
TOS – ID 17.2 & 17.3

Description Upper & Lower GIT


A 62 year old man has been noticing progressive difficulty swallowing, first solid food
STEM and now liquids as well. A barium study shows a ragged narrowing just below the cannal
level. Endoscopic biopsy confirms squamous cell carcinoma. Which of the following
statements concerning carcinoma of the esophagus is true?

ITEM A Alcohol has not been implicated as a precipitating factor


ITEM B Squamous carcinoma is the most common type at the cardioesophageal junction
ITEM C It has a higher incidence in females
ITEM D It occurs more commonly
ITEM E The standard of care is radiation herapy

Correct Answer D

Domain: Recall [] Interpretation [] Problem solving []

Reference(s) Bailey & Love


Prepared by
College Frontier Medical & Dental College, Abbottabad

Name:

Signature: –––––––––––––––––––––––––––––––

Date:
Note: Diagrams/Charts/Graphs if any should attach along this

Program MBBS

Subject General Surgery

Prof Final

TOS ID

Description Urogenital

stem A young male presented in emergency department having pain in loin radiating to groin.
He is also having microscopic hematuria. What is likely cause

Item A Renal cell carcinoma

Item B Bladder tumor

Item C Renal stone

Item D BPH

Item E Prostate Ca

Correct answer C

Domain; Recall Interpretation√ Problem solving

Reference; Bailey and love Edition 26 page

Prepared by;

College; Frontier Medical and Dental College, Abbottabad.

Name;

Signature

Date;
Program MBBS

Subject General Surgery

Prof Final

TOS ID

Description Urogenital

stem What percentage of renal stones are radiolucent

Item A 10 %

Item B 20 %

Item C 50%

Item D 70%

Item E 90%

Correct answer A

Domain; Recall√ Interpretation Problem solving


Reference; Bailey and love Edition 26 page

Prepared by;

College; Frontier Medical and Dental College, Abbottabad.

Name;

Signature

Date;

Program MBBS

Subject General Surgery

Prof Final

TOS ID

Description Urogenital

stem Which investigation is not helpful in detecting renal stones

Item A Urine R/E

Item B X-Ray KUB

Item C Ultrasound

Item D CT Scan

Item E Urine culture


Correct answer E

Domain; Recall√ Interpretation Problem solving

Reference; Bailey and love Edition 26 page

Prepared by;

College; Frontier Medical and Dental College, Abbottabad.

Name;

Signature

Date;

Program MBBS

Subject General Surgery

Prof Final

TOS ID
Description Urogenital

Stem A young male presented in emergency department having pain in loin radiating to
groin. Ultra sound showed 2 cm stone in middle pole and no obstruction. What is
right treatment option for him

Item A Pyelolithotomy

Item B Nephrolithotomy

Item C ESWL

Item D Laparoscopy

Item E No treatment

Correct answer C

Domain; Recall Interpretation Problem solving √

Reference; Bailey and love Edition 26 page

Prepared by;

College; Frontier Medical and Dental College, Abbottabad.

Name;

Signature

Date;
Program MBBS

Subject General Surgery

Prof Final

TOS ID

Description Urogenital

Stem A 25 year man is admitted with acute left testicular pain for 02 days dysurea & running
fever. Pain relieves on elevating testis. What is diagnosis

Item A Torsion

Item B Epididimo-Orchitis

Item C Sebaceous cyst

Item D Testicular tumor

Item E Hematocele

Correct answer B

Domain; Recall Interpretation √ Problem solving

Reference; Bailey and love Edition 26 page

Prepared by;

College; Frontier Medical and Dental College, Abbottabad.

Name;

Signature
Date;

Program MBBS

Subject General Surgery

Prof Final

TOS ID

Description Urogenital

Stem A 60 year old male presented with LUTS for 2-3 months. Examination shows enlarged
prostate, rubbery and mobile mucosa. What is likely diagnosis

Item A CA prostate

Item B Urethral stricture

Item C BPH

Item D Detrusor instability

Item E Weak pelvic floor

Correct answer C

Domain; Recall Interpretation√ Problem solving

Reference; Bailey and love Edition 26 page


Prepared by;

College; Frontier Medical and Dental College, Abbottabad.

Name;

Signature

Date;

Program MBBS

Subject General Surgery

Prof Final

TOS ID

Description Urogenital

Stem A 30 year old male presented with difficulty micturation, poor stream for 2-3 months.
He had road traffic accident 6 months ago for which he remained hospitalized &
catheterized for one month. What is likely cause

Item A BPH

Item B Marion’s disease


Item C Urethral stricture

Item D Ca Bladder

Item E Bladder stone

Correct answer C

Domain; Recall Interpretation Problem solving

Reference; Bailey and love Edition 26 page

Prepared by;

College; Frontier Medical and Dental College, Abbottabad.

Name;

Signature

Date;

BAHRIA UNIVERSITY ISLAMABAD

MULTIPLE CHOICES QUESTION

PROGRAM MBBS

SUBJECT SURGERY

PROF FINAL

TOS ID 17.3

Description Urology

STEM A young female presented with lower urinary tract symptoms for last five days.
She is running fever with chills, and generalized body aches. What will be your
investigations of first choice?
ITEM A Blood culture and sensitivity

ITEM B X-Ray KUB

ITEM C Ultrasonography for KUB

ITEM D Intravenous Urography

ITEM E Urine R/E

Correct Answer E

Domain Recall √ Interpretation Problem solving

Reference(s) BAILEY AND LOVES 26 EDITION

Prepared by

College Frontier Medical & Dental College Abbottabad

Name PROF DR JOHAR ALI

Signature……………………………………………………………………
BAHRIA UNIVERSITY ISLAMABAD

MULTIPLE CHOICES QUESTION

PROGRAM MBBS

SUBJECT SURGERY

PROF FINAL

TOS ID 17.3

Description Urology

STEM A diagnosed case of UTI was treated with multiple antibiotics by general
practioner. She came to OPD with the same complaint. What will be your next
step?

ITEM A Urgent CT Scan of kidney

ITEM B Abdominal x-ray

ITEM C Blood complete picture

ITEM D Urinalysis

ITEM E Mid Stream Urine for culture and sensitivity

Correct Answer E

Domain Recall Interpretation Problem solving

Reference(s) BAILEY AND LOVES 26 EDITION

Prepared by

College Frontier Medical & Dental College Abbottabad

Name PROF DR JOHAR ALI

Signature……………………………………………………………………
BAHRIA UNIVERSITY ISLAMABAD

MULTIPLE CHOICES QUESTION

PROGRAM MBBS

SUBJECT SURGERY

PROF FINAL

TOS ID 17.3

Description Urology

STEM A child of 10 years was repeatedly gets admission in urology ward for recurrent
Urinary Tract Infections. What is the most common underlying pathology?

ITEM A Poor personal Hygiene

ITEM B Bottle feeding

ITEM C Contaminated water intake

ITEM D Congenital anomalies of kidney uretar and bladder

ITEM E Poor intake of water

Correct Answer E

Domain Recall Interpretation Problem solving

Reference(s) BAILEY AND LOVES 26 EDITION

Prepared by

College Frontier Medical & Dental College Abbottabad

Name PROF DR JOHAR ALI

Signature……………………………………………………………………
BAHRIA UNIVERSITY ISLAMABAD

MULTIPLE CHOICES QUESTION

PROGRAM MBBS

SUBJECT SURGERY

PROF FINAL

TOS ID 17.3

Description Urology

STEM What is the most common Uropathogen isolated during culture and sensitivity
of clean catch mid stream urine?

ITEM A Streptococcus Aureas

ITEM B E.Coli

ITEM C Staph Aureas

ITEM D Pseudomonas Aeunginosa

ITEM E Bacteriodes

Correct Answer B

Domain Recall Interpretation Problem solving

Reference(s) BAILEY AND LOVES 26 EDITION

Prepared by

College Frontier Medical & Dental College Abbottabad

Name PROF DR JOHAR ALI


Signature……………………………………………………………………
BAHRIA UNIVERSITY ISLAMABAD

MULTIPLE CHOICES QUESTION

PROGRAM MBBS

SUBJECT SURGERY

PROF FINAL

TOS ID 17.3

Description Urology

STEM A 40 years patient presented with haematoma and pain Rt. Flank urinalysis
shows numerous RBCs and pus cells. What is your next investigation of choice?

ITEM A Blood urea/sugar

ITEM B X-Ray KUB

ITEM C Urine culture and sensitivity

ITEM D Radioisotope scanning

ITEM E Intra venous Pyelography

Correct Answer

Domain Recall Interpretation Problem solving

Reference(s) BAILEY AND LOVES 26 EDITION

Prepared by

College Frontier Medical & Dental College Abbottabad


Name PROF DR JOHAR ALI

Signature……………………………………………………………………
BAHRIA UNIVERSITY ISLAMABAD

MULTIPLE CHOICES QUESTION

PROGRAM MBBS

SUBJECT SURGERY

PROF FINAL

TOS ID 17.3

Description Urology

STEM A female of middle age presented with severe flank pain, that radiate to left
grown and labium. What is the most likely underlying pathology ?

ITEM A Kidney stones

ITEM B Lower ureteric calculus

ITEM C Bladder stone

ITEM D Renal pelvis calculus

ITEM E Urinary tract infection

Correct Answer

Domain Recall Interpretation√ Problem solving

Reference(s) BAILEY AND LOVES 26 EDITION page 1272

Prepared by

College Frontier Medical & Dental College Abbottabad

Name PROF DR JOHAR ALI


Signature……………………………………………………………………
BAHRIA UNIVERSITY ISLAMABAD

MULTIPLE CHOICES QUESTION

PROGRAM MBBS

SUBJECT SURGERY

PROF FINAL

TOS ID 17.3

Description Urology

STEM A boy of 8 years presented with recurrent UTI for the last year. Examination of
abdomen has shown a non tender mass in the right iliac fossa. X-ray KUB
reveals a radiopaque shadow at the level of transverse process of lumbar 5
vertebras. What is your diagnosis?

ITEM A Acute appendicitis

ITEM B Undescended testis

ITEM C Pelvic kidney

ITEM D Horse shoe kidney

ITEM E Appendicular mass

Correct Answer

Domain Recall Interpretation Problem solving

Reference(s) BAILEY AND LOVES 26 EDITION Page 1283

Prepared by

College Frontier Medical & Dental College Abbottabad


Name PROF DR JOHAR ALI

Signature……………………………………………………………………
BAHRIA UNIVERSITY ISLAMABAD

MULTIPLE CHOICES QUESTION

PROGRAM MBBS

SUBJECT SURGERY

PROF FINAL

TOS ID 17.3

Description Urology

STEM A patient was investigated for recurrent UTI. On IVU renal calyces one reversed
and lower poles are pointing forwards midline. What is possible radiological
diagnosis?

ITEM A Normal kidneys

ITEM B Unilateral fused kidney

ITEM C Duplication of renal pelvis

ITEM D Horse shoe kidney

ITEM E Duplex of renal pelvis.

Correct Answer D

Domain Recall Interpretation Problem solving

Reference(s) BAILEY AND LOVES 26 EDITION Page 1283

Prepared by

College Frontier Medical & Dental College Abbottabad

Name PROF DR JOHAR ALI


Signature……………………………………………………………………
BAHRIA UNIVERSITY ISLAMABAD

MULTIPLE CHOICES QUESTION

PROGRAM MBBS

SUBJECT SURGERY

PROF FINAL

TOS ID 17.3

Description Urology

STEM A lady of 45 years was presenting with un-controlled hypertension and loin pain
for last two years. Her urinalysis reveals RBCs and pus cells. Her serum critine is
1.2 mg/dl, Hb.7 gm. Abdominal ultrasonography shows multiples in both
kidneys and liver. What is your diagnosis?

ITEM A Bilateral hydro-nephrosis

ITEM B Congenital cystic kidneys

ITEM C Pelvicureterine junction obstruction

ITEM D Chromic renal failure due to hypertension

ITEM E Cystic adenocarcinoma of kidney

Correct Answer B

Domain Recall Interpretation Problem solving

Reference(s) BAILEY AND LOVES 26 EDITION PAGE 1285

Prepared by

College Frontier Medical & Dental College Abbottabad

Name PROF DR JOHAR ALI


Signature……………………………………………………………………
BAHRIA UNIVERSITY ISLAMABAD

MULTIPLE CHOICES QUESTION

PROGRAM MBBS

SUBJECT SURGERY

PROF FINAL

TOS ID 17.3

Description Urology

STEM After sustaining blunt abdominal injury during foot ball match a young athlete
was admitted with haematuria. He was getting conservative treatment. After
started of admission his abdomen was distended and percussion note was
tympanitic. What the cause of this development?

ITEM A Spleenic injury

ITEM B Liver injury

ITEM C Bleeding into lesser sac

ITEM D Retroperitoneal haematoma

ITEM E Bladder perforation

Correct Answer D

Domain Recall Interpretation Problem solving

Reference(s) BAILEY AND LOVES 26 EDITION

Prepared by

College Frontier Medical & Dental College Abbottabad


Name PROF DR JOHAR ALI

Signature……………………………………………………………………
BAHRIA UNIVERSITY ISLAMABAD

MULTIPLE CHOICES QUESTION

PROGRAM MBBS

SUBJECT SURGERY

PROF FINAL

TOS ID 17.3

Description Urology

STEM A patient admitted with blunt renal trauma was treated conservatively in
surgery ward. He was discharged after seven days. He experienced sever
haematuria with in two weeks of discharge. What is the cause of this bleeding?

ITEM A Renal haematoma infection

ITEM B Clot dislogment

ITEM C Hypertension

ITEM D Renal stnes

ITEM E Drugs

Correct Answer A

Domain Recall Interpretation Problem solving

Reference(s) BAILEY AND LOVES 26 EDITION

Prepared by

College Frontier Medical & Dental College Abbottabad

Name PROF DR JOHAR ALI

Signature……………………………………………………………………
BAHRIA UNIVERSITY ISLAMABAD

MULTIPLE CHOICES QUESTION

PROGRAM MBBS

SUBJECT SURGERY

PROF FINAL

TOS ID 17.3

Description Urology

STEM White performing total abdominal hysterectomy Lt ureter was transected


accidentally. What is the best treatment option if injury is identified on the
operation room table?

ITEM A Put large bore pelvic drain and close abdomen

ITEM B Spatulate both ends and do end to end anastomosis

ITEM C Close he cut end and do nephrostomy

ITEM D Bring ureter out of abdominal wall as external diversion

ITEM E Insert retrograde D.J. Stenting

Correct Answer B

Domain Recall Interpretation Problem solving

Reference(s) BAILEY AND LOVES 26 EDITION

Prepared by

College Frontier Medical & Dental College Abbottabad

Name PROF DR JOHAR ALI

Signature……………………………………………………………………
BAHRIA UNIVERSITY ISLAMABAD

MULTIPLE CHOICES QUESTION

PROGRAM MBBS

SUBJECT SURGERY

PROF FINAL

TOS ID 17.3

Description Urology

STEM A large segment of lower end of ureter is last due to fire arm injury in a street
brawl. What s the best option of repair in a haemodynamically stable patient?

ITEM A Boari flap

ITEM B End to end anastomosis

ITEM C Insertion of pigtail catheter

ITEM D Kidney mobilization

ITEM E Illeal conduct formation

Correct Answer A

Domain Recall Interpretation Problem solving

Reference(s) BAILEY AND LOVES 26 EDITION

Prepared by

College Frontier Medical & Dental College Abbottabad

Name PROF DR JOHAR ALI

Signature……………………………………………………………………
BAHRIA UNIVERSITY ISLAMABAD

MULTIPLE CHOICES QUESTION

PROGRAM MBBS

SUBJECT SURGERY

PROF FINAL

TOS ID 17.3

Description Urology

STEM An elderly patient was admitted with raised blood 70 mg/ld urea and cretin 2.5
mg/dl. Abdominal examination reveals hypogastric fullness. Ultrasonography
shows bilateral moderate hydronephrosis. What is the most likely cause in this
patient?

ITEM A Post uretheral valves

ITEM B Ureteric stones

ITEM C Blood stones

ITEM D Bladder outlet obstruction due to enlarged prostate

ITEM E Neurogenic bladder

Correct Answer D

Domain Recall Interpretation Problem solving

Reference(s) BAILEY AND LOVES 26 EDITION

Prepared by

College Frontier Medical & Dental College Abbottabad

Name PROF DR JOHAR ALI


Signature……………………………………………………………………
BAHRIA UNIVERSITY ISLAMABAD

MULTIPLE CHOICES QUESTION

PROGRAM MBBS

SUBJECT SURGERY

PROF FINAL

TOS ID 17.3

Description Urology

STEM Patient with a solitary kidney was admitted in the ward with anurea and
impaired renal functions. Ultranography reveals severe hydrosnephrosis. What
is the best option in management of this patient?

ITEM A Open surgery and removal of stone

ITEM B Extracorporeal shock wave lithotripsy

ITEM C Percutaneous nephrostomy

ITEM D Foly’s catheter insertion

ITEM E I/V fluid and forced diuresis

Correct Answer C

Domain Recall Interpretation Problem solving

Reference(s) BAILEY AND LOVES 26 EDITION

Prepared by

College Frontier Medical & Dental College Abbottabad

Name PROF DR JOHAR ALI

Signature……………………………………………………………………
BAHRIA UNIVERSITY ISLAMABAD

MULTIPLE CHOICES QUESTION

PROGRAM MBBS

SUBJECT SURGERY

PROF FINAL

TOS ID 17.3

Description Urology

STEM An uncircumcised child of 3 years was brought to OPD by his anxious parents
with history of ballooning of penis tip while micturition. What is most likely
etiology

ITEM A Uretheral stricture

ITEM B Uretheral meatal stenosis

ITEM C Phimosis

ITEM D Posterior uretharal valves

ITEM E Paraphimosis

Correct Answer C

Domain Recall Interpretation Problem solving

Reference(s) BAILEY AND LOVES 26 EDITION

Prepared by

College Frontier Medical & Dental College Abbottabad

Name PROF DR JOHAR ALI

Signature……………………………………………………………………
BAHRIA UNIVERSITY ISLAMABAD

MULTIPLE CHOICES QUESTION

PROGRAM MBBS

SUBJECT SURGERY

PROF FINAL

TOS ID 17.3

Description Urology

STEM 15 years old boy was presented with pain in the flanks and intermittent urinary
tract infection. Renal isotope scanning and ultra sonography shows idiopathic
right sided hydronephrosis. What minimal invasive technique will be helpful to
correct this entity.

ITEM A Antegrade double J stenting

ITEM B Ureteroscopy

ITEM C Endoscopic Pyeloysis

ITEM D Laproscopic Pyeloplasty

ITEM E Laser Pyelolysis

Correct Answer C

Domain Recall Interpretation Problem solving

Reference(s) BAILEY AND LOVES 26 EDITION

Prepared by

College Frontier Medical & Dental College Abbottabad

Name PROF DR JOHAR ALI


Signature……………………………………………………………………
BAHRIA UNIVERSITY ISLAMABAD

MULTIPLE CHOICES QUESTION

PROGRAM MBBS

SUBJECT SURGERY

PROF FINAL

TOS ID 17.3

Description Urology

STEM 50 yeas old diabetic female presents with Dysuria and pain left flank, her
Urinalysis shows numerous pus cells and RBCs KUB x-ray reveal a 3x2 cm
irregular radiopaque shadow in the right renal area. What is the best treatment
option?

ITEM A Open surgical removal of stone

ITEM B Extracorporeal shock wave lithotripsy

ITEM C Ureteroscopic Pneumatic lithotripsy

ITEM D Percutaneous nephrolithatomy

ITEM E IV antibiotics analgesics and control of diabetes

Correct Answer D

Domain Recall Interpretation Problem solving

Reference(s) BAILEY AND LOVES 26 EDITION

Prepared by

College Frontier Medical & Dental College Abbottabad

Name PROF DR JOHAR ALI


Signature……………………………………………………………………
BAHRIA UNIVERSITY ISLAMABAD

MULTIPLE CHOICES QUESTION

PROGRAM MBBS

SUBJECT SURGERY

PROF FINAL

TOS ID 17.3

Description Urology

STEM A disabled adult was brought to hospital with flank pains. KUB ex-ray shows
multiple stones in upper and middle zones of kidney measuring 6 mm each.
What will be the best management option?

ITEM A Open surgery nephrolithatomy

ITEM B Extra corporeal shock wave lithotripsy

ITEM C Percutaneous nephrotithatomy

ITEM D Will pass out spontaneously with conservative treatment

ITEM E Thiazide diuretics

Correct Answer D

Domain Recall Interpretation Problem solving

Reference(s) BAILEY AND LOVES 26 EDITION

Prepared by

College Frontier Medical & Dental College Abbottabad

Name PROF DR JOHAR ALI

Signature……………………………………………………………………
Bahria University

Multiple Choices Question

Program MBBS

Subject Surgery

Prof Mbbs

TOS-ID

Description Carcinoma breast

Stem A 42 year aged lady underwent breast conservative surgery for carcinoma of the
breast. She has been discharged from the hospital with advice to come for follow
up yearly. Which of the following tests is recommended for follow up?

Item A FNAC

Item B Mammography

Item C MRI

Item D Tumor marker assay

Item E Ultrasound

Correct Answer Mammography

Domain  Recall Interpretation Problem solving

Preference Bailey & Love 26 edition Chapter , page

Prepared by

College Frontier Medical & Dental College, Abbottabad


Name: Dr Johar Ali

Signature:

Date: 8.8.2016
Bahria University

Multiple Choices Question

Program MBBS

Subject Surgery

Prof Mbbs

TOS-ID

Description Abdominal trauma

Stem

A 20 years old boy was brought to emergency department due to motor car
accident. His systolic blood pressure is 60 mm Hg and abdomen is tender all over.
Which of the following investigation is most appropriate to detect intraabdominal
bleeding in this patient?

Item A Diagnostic peritoneal lavage


Item B Diagnostic laparotomy
Item C FAST
Item D MRI
Item E Spiral CT

Correct Answer a. Diagnostic peritoneal lavage

Domain Recall Interpretation  Problem


solving

Preference Bailey & Love 26 edition Chapter , page


Prepared by

College Frontier Medical & Dental College, Abbottabad

Name: Dr Johar Ali

Signature: Date: 8.8.2016


Bahria University

Multiple Choices Question

Program MBBS

Subject Surgery

Prof Mbbs

TOS-ID

Description

Stem A 30 years male is brought to emergency department due to multiple injuries from
car accident. He is complaining of pain in right hypochondrium. His systolic blood
pressure is 110 mmHg. Which of the following is most suitable investigation to
detect solid organ injuries in this patient?

Item A CT scan
Item B DPL
Item C Digital Xray
Item D FAST
Item E MRI

Correct Answer a. CT scan

Domain Recall Interpretation  Problem


solving

Preference Bailey & Love 26 edition Chapter , page

Prepared by
College Frontier Medical & Dental College, Abbottabad

Name: Dr Ishtiaq Ali Khan

Signature:

Date: 8.8.2016
Bahria University

Multiple Choices Question

Program MBBS

Subject Surgery

Prof Mbbs

TOS-ID

Description Abdominal truama

Stem

A 37 years female presented with discomfort in right hypochondrium. In order to


exclude liver diseases which of the following should be the first line investigation?

Item A Abdominal ultrasound

Item B CT

Item C ERCP

Item D Laproscopic ultrasound

Item E MRI

Correct Answer a. Abdominal ultrasound

Domain  Critical Interpretation Problem solving


thinking

Preference Bailey & Love 26 edition Chapter , page


Prepared by

College Frontier Medical & Dental College, Abbottabad

Name: Dr Johar Ali

Signature:

Date:
Bahria University

Multiple Choices Question

Program MBBS

Subject Surgery

Prof Mbbs

TOS-ID

Description Benign breast diseases

Stem

47 years old perimenopausal woman presented with a 3 week history of green


discharge from the right nipple. O/E right nipple is non-tender, has a slit-like
appearance and is retracted. Which of the following is the most likely diagnosis?

Item A Galactorrhoea

Item B Duct papilloma

Item C Breast carcinoma

Item D Mammary duct ectasia

Item E Periductal mastitis

Correct Answer d. Mammary duct ectasia

Domain Recall  Interpretation Problem solving

Preference Bailey & Love 26 edition Chapter , page


Prepared by

College Frontier Medical & Dental College, Abbottabad

Name: Dr Johar Ali

Signature: Date:
Bahria University

Multiple Choices Question

Program MBBS

Subject Surgery

Prof Mbbs

TOS-ID

Description Abdominal trauma

Stem A teen age boy was stabbed with knife in the abdomen. He was brought to
emergency department and admitted to the surgical department. He is stable
hemodinamically. Which of the following is appropriate treatment for him?
Item A Observation and intravenous antibiotics

Item B Explorative laparotomy

Item C Laparoscopy and intravenous antibiotics

Item D NPO and antibiotics

Item E Intravenous fluids and antibiotics

Correct Answer b. Explorative laparotomy

Domain Recall Interpretation  Problem


solving

Preference Bailey & Love 26 edition Chapter , page

Prepared by
College Frontier Medical & Dental College, Abbottabad

Name: Dr Johar Ali

Signature:

Date:8.8.2016
Bahria University

Multiple Choices Question

Program MBBS

Subject Surgery

Prof Mbbs

TOS-ID

Description

Stem

A 45 year old female presented with unilateral nipple discharge since 3 months.
On examination it was found that discharge is from single duct orifice. Which of
the following is most probable cause in this patient?

Item A Intraductal carcinoma

Item B Inflammatory carcinoma

Item C Lobular carcinoma

Item D Paget’s disease

Item E Mucinous carcinoma

Correct Answer a. Intraductal carcinoma

Domain Recall Interpretation  Problem


solving

Preference Bailey & Love 26 edition Chapter , page


Prepared by

College Frontier Medical & Dental College, Abbottabad

Name: Dr Johar Ali

Signature:

Date: 8.8.2016
Bahria University

Multiple Choices Question

Program MBBS

Subject Surgery

Prof Mbbs

TOS-ID

Description Carcinoma breast

Stem A 41-year-old female presented with a biopsy proven invasive ductal cancer in
the upper outer aspect of her left breast. On examination her left axillary lymph
nodes are palpable. Mammography shows diffuse calcifications throughout the
rest of her breast proven to be DCIS on stereotactic biopsy. Which of the
following is the best treatment option for her?

Item A Lumpectomy

Item B Lumpectomy with axillary clearance

Item C Modified radical mastectomy

Item D Radical mastectomy

Item E Simple mastectomy

Correct Answer c

Domain Recall Interpretation  Problem


solving

Preference Bailey & Love 26 edition Chapter , page


Prepared by

College Frontier Medical & Dental College, Abbottabad

Name: Dr Johar Ali

Signature:

Date:
Bahria University

Multiple Choices Question

Program MBBS

Subject Surgery

Prof Mbbs

TOS-ID

Description Varicose veins

Stem A 30 years old male presented with varicose veins on his right leg. He has been advised
surgery. Which of following is the most common complication of this treatment
option?

Item A Deep vein thrombosis

Item B Femoral nerve injury

Item C Femoral artery injury

Item D Femoral vein injury

Item E Recurrence

Correct Answer e. Recurrence

Domain  Recall Interpretation Problem solving

Preference Bailey & Love 26 edition Chapter , page


Prepared by

College Frontier Medical & Dental College, Abbottabad

Name: Dr Johar Ali

Signature:

Date:
Bahria University

Multiple Choices Question

Program MBBS

Subject Surgery

Prof Mbbs

TOS-ID

Description Lymphatic disorders

Stem A 40 years female presented with painful leg. She is diagnosed as suffering from acute
lymphangitis. Which of the following treatment option will you chose?

Item A Intravenous antibiotics

Item B Immediate lymphangiography

Item C Immediate multiple incisions

Item D Laser coagulation of lymphatic vessels

Item E No treatment is required.

Correct Answer a. Intravenous antibiotic

Domain Recall Interpretation Problem solving

Preference Bailey & Love 26 edition Chapter , page

Prepared by

College Frontier Medical & Dental College, Abbottabad


Name: Dr Johar Ali

Signature:

Date:
Bahria University

Multiple Choices Question

Program MBBS

Subject Surgery

Prof Mbbs

TOS-ID

Description Chest trauma

Stem An 18 years old boy was brought to emergency department with penetrating
chest injuries. Which of the following is most appropriate treatment in this case?

Item A Adequate antiseptic dressing of the wound

Item B Antibiotic and analgesics

Item C Endoscopic management

Item D Resuscitation and drainage

Item E Thoracotomy

Correct Answer d. Resuscitation and drainage

Domain Recall Interpretation Problem solving

Preference Bailey & Love 26 edition Chapter , page


Prepared by

College Frontier Medical & Dental College, Abbottabad

Name: Dr Johar Ali

Signature:

Date: 8.8.2016
Bahria University

Multiple Choices Question

Program MBBS

Subject Surgery

Prof Mbbs

TOS-ID

Description Lung tumors

Stem A 50 years old female presented with persistent cough and weight loss and nonspecific
chest pain. Which of the following is most probable diagnosis?

Item A adenocarcinoma

Item B bronchoalveolar carcinoma

Item C large cell undifferentiated lung tumor

Item D small cell lung tumor

Item E squamous cell carcinoma

Correct Answer a. adenocarcinoma

Domain  Recall Interpretation Problem solving

Preference Bailey & Love 26 edition Chapter , page

Prepared by

College Frontier Medical & Dental College, Abbottabad

Name: Dr Johar Ali


Signature:

Date:

Program MBBS

Subject General Surgery

Prof Final

TOS ID 17.2.1

Description GIT ( Esophagus )

stem A 35 year old female patient complains of dysphagia for last 6 months more for liquids
than solids. Barium swallow shows bird beak deformity. What is likely diagnosis

Item A Zenkers diverticulum

Item B Achalasia

Item C Ca esophagus

Item D Peptic ulcer disease

Item E GORD

Correct answer B

Domain; Recall Interpretation Problem solving

Reference; Bailey and love Edition 26 page 1014

Prepared by;

College; Frontier Medical and Dental College, Abbottabad.

Name; Prof.Dr.Johar Ali

Signature

Date;
Program MBBS

Subject General Surgery

Prof Final

TOS ID 17.2.1

Description GIT ( Esophagus )

Stem Which of the following is considered to be pre malignant lesion

Item A Esophageal candidiasis

Item B Barrets esophagus

Item C Schatzki rings

Item D Moniliasis

Item E Mallory Weiss tear

Correct answer B

Domain; Recall Interpretation Problem solving

Reference; Bailey and love Edition 26 page 1000

Prepared by;

College; Frontier Medical and Dental College, Abbottabad

Name; Prof.Dr.Johar Ali

Signature

Date;
Program MBBS

Subject General Surgery

Prof Final

TOS ID 17.2.1

Description GIT ( Esophagus )

stem A patient presented in surgical OPD having severe cough especially at bed time and
regurgitation of undigested food material. He is also embarrassed by bad breath. What is
likely diagnosis

Item A Ca Esophagus

Item B Pharyngeal pouch

Item C Peptic stricture

Item D Barrets Esophagus

Item E GORD

Correct Answer B

Domain; Recall Interpretation Problem solving

Reference; Bailey and love Edition 26 page 1018

Prepared by;

College; Frontier Medical and Dental College, Abbottabad.

Name; Prof.Dr.Johar Ali

Signature

Date;
Program MBBS

Subject General Surgery

Prof Final

TOS ID 17.2.1

Description GIT ( Esophagus )

stem A 50 year old lady presented to surgical OPD complaining of progressive dysphagia, weight
loss and anemia. On examination patient is pale and emaciated. What is likely diagnosis

Item A GORD

Item B Achalasia

Item C Ca Esophagus

Item D Ca Stomach

Item E PUD

Correct answer C

Domain; Recall Interpretation Problem solving

Reference; Bailey and love Edition 26 page 1004

Prepared by;

College; Frontier Medical and Dental College, Abbottabad.

Name; Prof.Dr.Johar Ali

Signature

Date;
Program MBBS

Subject General Surgery

Prof Final

TOS ID 17.2.1

Description GIT ( Esophagus )

stem A 50 year old lady presented to surgical OPD complaining of progressive dysphagia, weight
loss and anemia. On examination patient is pale and emaciated. Which investigation will
confirm diagnosis

Item A Ultra sound abdomen

Item B Chest X Ray

Item C CT Scan

Item D Endoscopy

Item E Barium Meal

Correct answer D

Domain; Recall Interpretation Problem solving

Reference; Bailey and love Edition 26 page 1007

Prepared by;

College; Frontier Medical and Dental College, Abbottabad.

Name; Prof.Dr.Johar Ali


Signature

Date;
Program MBBS

Subject General Surgery

Prof Final

TOS ID 17.2.1

Description GIT ( Esophagus )

stem A 35 year old female patient complains of dysphagia for last 6 months more for liquids
than solids. Which investigation will confirm diagnosis

Item A Barium Sallow

Item B Barium meal

Item C CT scan

Item D Chest X Ray

Item E Ultra sound chest

Correct answer A

Domain; Recall Interpretation Problem solving

Reference; Bailey and love Edition 26 page 1014

Prepared by;

College; Frontier Medical and Dental College, Abbottabad

Name; Prof.Dr.Johar Ali

Signature

Date;
Program MBBS

Subject General Surgery

Prof Final

TOS ID 17.2.1

Description GIT ( Esophagus )

stem A 35 year old female patient complains of dysphagia for last 6 months more for liquids
than solids. Barium swallow shows bird beak deformity and proximal dilated esophagus.
What is treatment option

Item A Esophagectomy

Item B Heller’s Cardiomyotomy

Item C Endoscopic balloon dilatation

Item D Stenting

Item E Fundoplication

Correct answer C

Domain; Recall Interpretation Problem solving

Reference; Bailey and love Edition 26 page 1015

Prepared by;

College; Frontier Medical and Dental College, Abbottabad

Name; Prof.Dr.Johar Ali

Signature

Date;
Program MBBS

Subject General Surgery

Prof Final

TOS ID 17.2.2

Description GIT ( Stomach & duodenum)

stem A 55 year old male known smoker presented in emergency with history of vomiting and
upper abdominal pain for last few hours. Examination revealed pallor, pulse 110/min, BP
110/70 & tense and tender abdomen. What is likely cause

Item A Acute pancreatitis

Item B Acute Gastritis

Item C Perforated duodenal ulcer

Item D Acute cholecystitis

Item E Acute appendicitis

Correct answer C

Domain; Recall Interpretation Problem solving

Reference; Bailey and love Edition 26 page 1041

Prepared by;

College: Frontier Medical and Dental College, Abbottabad.

Name; Prof.Dr.Johar Ali

Signature
Date;
Program MBBS

Subject General Surgery

Prof Final

TOS ID 17.2.2

Description GIT ( Stomach & duodenum)

stem A 55 year old male known smoker presented in emergency with history of vomiting and
upper abdominal pain for last few hours. Examination revealed pallor, pulse 110/min, BP
110/70 & tense and tender abdomen. Which investigation will help in diagnosis

Item A TLC

Item B X Ray erect abdomen

Item C CXR

Item D Barium swallow

Item E Ultra sound abdomen

Correct answer B

Domain; Recall Interpretation Problem solving

Reference; Bailey and love Edition 26 page 1042

Prepared by;

College: Frontier Medical and Dental College, Abbottabad.

Name; Prof.Dr.Johar Ali

Signature

Date;
Program MBBS

Subject General Surgery

Prof Final

TOS ID 17.2.2

Description GIT ( Stomach & duodenum)

stem A 55 year old female presented in OPD with history of hematamesis and malena for last
few days. She is also using NSAIDs for arthritis for few years. What is likely cause

Item A CA Esophagus

Item B Stress Gastritis

Item C CA Stomach

Item D Erosive Gastritis

Item E Gastric outlet obstruction

Correct answer D

Domain; Recall Interpretation Problem solving

Reference; Bailey and love Edition 26 page 1032

Prepared by;

College; Frontier Medical and Dental College, Abbottabad.

Name; Prof.Dr.Johar Ali

Signature

Date;
Program MBBS

Subject General Surgery

Prof Final

TOS ID 17.2.2

Description GIT ( Stomach & duodenum)

stem A 45 year old male presented in OPD with history of non bilious vomiting 2-3 hours after
meal containing digested food. He has also lost weight in recent days. Examination
revealed dehydration, pallor and palpable mass in epigastrium. What is probable diagnosis

Item A Gastritis

Item B CA esophagus

Item C Ca Stomach

Item D Ca head of pancreas

Item D GORD

Correct answer C

Domain; Recall Interpretation Problem solving

Reference; Bailey and love Edition 26 page 1046

Prepared by;

College: Frontier Medical and Dental College, Abbottabad.

Name; Prof.Dr.Johar Ali


Signature

Date;
Program MBBS

Subject General Surgery

Prof Final

TOS ID 17.2.2

Description GIT ( Stomach & duodenum)

stem A 54 year old male had previous total gastrectomy presents with anemia & tingling in arms
and legs. Peripheral blood smear was done that showed macrocytic anemia. What is likely
cause

Item A Early dumping syndrome

Item B Recurrent ulceration causing bleeding

Item C Malignancy

Item D B12 deficiency

Item E Bleeding varices

Correct answer D

Domain; Recall Interpretation Problem solving

Reference; Bailey and love Edition 26 page 1053

Prepared by;

College; Frontier Medical and Dental College, Abbottabad

Name; Prof.Dr.Johar Ali

Signature

Date;
Program MBBS

Subject General Surgery

Prof Final

TOS ID 17.2.2

Description GIT ( Stomach & duodenum)

stem A 45 year old male had previous surgery for peptic ulcer presents with explosive diarrhoe
that is not responding to routine anti diarrhoe medication. What is likely cause

Item A H-Pylori infection

Item B Malignancy

Item C Dumping syndrome

Item D Post vagotomy diarrhoe

Item E B12 deficiency

Correct answer D

Domain; Recall Interpretation Problem solving

Reference; Bailey and love Edition 26 page 1039

Prepared by;

College; Frontier Medical and Dental College, Abbottabad

Name; Prof.Dr.Johar Ali

Signature

Date;
Program MBBS

Subject General Surgery

Prof Final

TOS ID 17.2.2

Description GIT ( Stomach & duodenum)

Stem Which of the following best explains a longitudinal tear below gastro esophageal junction
induced by repetitive & strenuous vomiting that causes hematamesis

Item A Gastric volvolus

Item B Esophagael varices

Item C Mallory Weiss tear

Item D Gastric ulcer

Item E Gastric polyp

Correct answer C

Domain; Recall Interpretation Problem solving

Reference; Bailey and love Edition 26 page 1043

Prepared by;

College; Frontier Medical and Dental College, Abbottabad

Name; Prof.Dr.Johar Ali

Signature

Date;
Program MBBS

Subject General Surgery

Prof Final

TOS ID 17.2.2

Description GIT ( Stomach & duodenum)

Stem What is common site of peptic ulcer caused by H-pylori

Item A Greater curvature of stomach

Item B 1st part of duodenum

Item C Esophago gastric junction

Item D esophagus

Item E DJ junction

Correct answer B

Domain; Recall Interpretation Problem solving

Reference; Bailey and love Edition 26 page 1032

Prepared by;

College; Frontier Medical and Dental College, Abbottabad

Name; Prof.Dr.Johar Ali

Signature

Date;
Program MBBS

Subject General Surgery

Prof Final

TOS ID 17.2.2

Description GIT ( Stomach & duodenum)

Stem Troisiers sign is enlarged

Item A Left axillary lymph node

Item B Left supra clavicular lymph node

Item C Left inguinal lymph node

Item D Right supra clavicle lymph node

Item E Cervical lymph node

Correct answer B

Domain; Recall Interpretation Problem solving

Reference; Bailey and love Edition 26 page 1048

Prepared by;

College; Frontier Medical and Dental College, Abbottabad

Name; Prof.Dr.Johar Ali

Signature

Date;
Program MBBS

Subject General Surgery

Prof Final

TOS ID 17.2.2

Description GIT ( Stomach & duodenum)

Stem A 45 year old male presented in OPD with history of non bilious vomiting 2-3 hours after
meal containing digested food. He has also lost weight in recent days. Examination
revealed dehydration, pallor and palpable mass in epigastrium and positive succession
splash. Which investigation will help in diagnosis

Item A X- Ray Abdomen

Item B Ultra sound abdomen

Item C CT scan abdomen

Item D Endoscopy

Item E Barium swallow

Correct answer D

Domain; Recall Interpretation Problem solving

Reference; Bailey and love Edition 26 page 1027

Prepared by;

College; Frontier Medical and Dental College, Abbottabad

Name; Prof.Dr.Johar Ali


Signature

Date;
Program MBBS

Subject General Surgery

Prof Final

TOS ID 17.2.3

Description GIT ( Liver )

stem The commonest benign tumor of liver is

Item A Fibroma

Item B Hemangioma

Item C Adenoma

Item D Lymphoma

Item E Hematoma

Correct answer B

Domain; Recall Interpretation Problem solving

Reference; Bailey and love Edition 26 page 1083

Prepared by;

College; Frontier Medical and Dental College, Abbottabad

Name; Prof.Dr.Johar Ali

Signature
Date;
Program MBBS

Subject General Surgery

Prof Final

TOS ID 17.2.3

Description GIT ( Liver)

Stem Budd Chiari Syndrome refers to

Item A Portal vein thrombosis

Item B Bile duct obstruction

Item C Hepatic vein thrombosis

Item D Hepatic artery thrombosis

Item E Mesenteric artery thrombosis

Correct answer C

Domain; Recall Interpretation Problem solving

Reference; Bailey and love Edition 26 page 1077

Prepared by;

College; Frontier Medical and Dental College, Abbottabad

Name; Prof.Dr.Johar Ali

Signature

Date;
Program MBBS

Subject General Surgery

Prof Final

TOS ID 17.2.3

Description GIT ( Liver)

stem A 45 year old diabetic patient complains of anorexia, high grade fever, malaise and right
upper quadrant discomfort. On examination there is tender hepatomegaly. Examination
revealed multiloculated cystic mass in liver. What is likely diagnosis

Item A Amoebic liver abscess

Item B Hydatid cyst

Item C Pyogenic liver abscess

Item D Viral hepatitis

Item E Ascending cholangitis

Correct answer C

Domain; Recall Interpretation Problem solving

Reference; Bailey and love Edition 26 page 1081

Prepared by;

College; Frontier Medical and Dental College, Abbottabad

Name; Prof.Dr.Johar Ali

Signature
Date;
Program MBBS

Subject General Surgery

Prof Final

TOS ID 17.2.3

Description GIT ( Liver)

stem A 50 year old farmer presents in OPD with continuous dull ache in Right hypochondrium
for few days. Examination shows palpable mass arising from right lobe liver. Blood count
shows increased Eosinophils. CT scan shows lesion in right lobe with floating membrane.
What is likely diagnosis

Item A Hepatoma

Item B Hydatid liver cyst

Item C Ascending cholangitis

Item D Hemangioma

Item E Amoebic abscess

Correct answer B

Domain; Recall Interpretation Problem solving

Reference; Bailey and love Edition 26 page 1081

Prepared by;

College; Frontier Medical and Dental College, Abbottabad

Name; Prof.Dr.Johar Ali

Signature

Date;
Program MBBS

Subject General Surgery

Prof Final

TOS ID 17.2.3

Description GIT (Liver)

Stem A 40 year old male presents with persistent upper abdominal pain, malaise and weight
loss. Examination reveals palpable mass in right hypochondrium. Alpha fetoprotein was
raised. What is likely diagnosis

Item A Focal nodular hyperplasia

Item B Hemangioma

Item C Hepatocellular Carcinoma

Item D Hepatic adenoma

Item E Liver cirrhosis

Correct answer C

Domain; Recall Interpretation Problem solving

Reference; Bailey and love Edition 26 page 1085

Prepared by;

College; Frontier Medical and Dental College, Abbottabad

Name; Prof.Dr.Johar Ali

Signature

Date;
Program MBBS

Subject General Surgery

Prof Final

TOS ID 17.2.3

Description GIT (Liver)

stem Charcot’s triad refers to

Item A Fever, Deranged LFTs , Anemia

Item B Fever, Pain right hypochondrium, jaundice

Item C Anemia, hepatomegaly, Jaundice

Item D Anemia, Pain right hypochondrium,


jaundice

Item E Anemia, fever, jaundice

Correct answer B

Domain; Recall Interpretation Problem solving

Reference; Bailey and love Edition 26 page 1080

Prepared by;

College; Frontier Medical and Dental College, Abbottabad

Name; Prof.Dr.Johar Ali

Signature

Date;
Program MBBS

Subject General Surgery

Prof Final

TOS ID 17.2.3

Description GIT ( Liver)

stem A 60 year old male presents with persistent upper abdominal pain, malaise and weight
loss & jaundice. He was operated 4 months back for CA Cecum and right hemicolectomy
was done. U/S shows two solid lesions in right lobe of liver. What is likely diagnosis

Item A Hepatocellular CA

Item B Hydatid cyst

Item C Hepatic adenoma

Item D Liver metastasis

Item E Amoebic liver abscess

Correct answer D

Domain; Recall Interpretation Problem solving

Reference; Bailey and love Edition 26 page 1084

Prepared by;

College; Frontier Medical and Dental College, Abbottabad

Name; Prof.Dr.Johar Ali

Signature

Date;
Program MBBS

Subject General Surgery

Prof Final

TOS ID 17.2.3

Description GIT (Liver)

stem Name the tumor marker of liver carcinoma

Item A CEA

Item B Alpha fetoprotein

Item C HCG

Item D LDH

Item E CA19-9

Correct answer B

Domain; Recall Interpretation Problem solving

Reference; Bailey and love Edition 26 page 1085

Prepared by;

College; Frontier Medical and Dental College, Abbottabad

Name; Prof.Dr.Johar Ali

Signature
Date;
Program MBBS

Subject General Surgery

Prof Final

TOS ID 17.2.3

Description GIT ( Liver)

stem A 50 year old farmer presents in OPD with continuous dull ache in Right hypochondrium
for few days. Examination shows palpable mass arising from right lobe liver. Blood count
shows increased Eosinophils. CT scan shows lesion in right lobe with multiple septae
about 4×5 cm. What is treatment option

Item A Anti Helminthic

Item B PAIR

Item C Open Surgery

Item D Observation

Item E Laparoscopy

Correct answer B

Domain; Recall Interpretation Problem solving

Reference; Bailey and love Edition 26 page 1081

Prepared by;

College; Frontier Medical and Dental College, Abbottabad

NameProf.Dr.Johar Ali;

Signature

Date;
Program MBBS

Subject General Surgery

Prof Final

TOS ID 17.2.4

Description GIT ( Spleen)

stem A 12 year old school student presents in OPD with history of upper abdominal pain for
few days. Examination revealed marked pallor and enlarged spleen. U/Sound abdomen
shows multiple small calculi and splenomegaly. What is likely diagnosis

Item A Idiopathic thrombocytopenic purpura

Item B Hereditary spherocytosis

Item C Splenic Infarction

Item D Infective endocarditis

Item E Malaria

Correct answer B

Domain; Recall Interpretation Problem solving

Reference; Bailey and love Edition 26 page 1092

Prepared by; Prof.Dr.Johar Ali

College; Frontier Medical and Dental College, Abbottabad

Name;

Signature
Date;
Program MBBS

Subject General Surgery

Prof Final

TOS ID 17.2.4

Description GIT ( Spleen)

stem A young cyclist fell down and had bruising over left hypochondrium. He presented in A&E
in state of shock. What is likely cause

Item A Liver injury

Item B Spleen injury

Item C Pancreatic injury

Item D Renal injury

Item E Gut injury

Correct answer B

Domain; Recall Interpretation Problem solving

Reference; Bailey and love Edition 26 page 1090

Prepared by;

College; Frontier Medical and Dental College, Abbottabad

Name; Prof.Dr.Johar Ali

Signature
Date;
Program MBBS

Subject General Surgery

Prof Final

TOS ID 17.2.4

Description GIT ( Spleen)

Stem A young patient had splenectomy for Splenic trauma. He is likely to develop which early
complication after surgery

Item A Reactionary hemorrhage

Item B OPSI

Item C Thrombocytopenia

Item D Leucopenia

Item E Chronic gastric dilatation

Correct answer A

Domain; Recall Interpretation Problem solving

Reference; Bailey and love Edition 26 page 1096

Prepared by;

College; Frontier Medical and Dental College, Abbottabad

Name; Prof.Dr.Johar Ali

Signature

Date;
Program MBBS

Subject General Surgery

Prof Final

TOS ID 17.2.4

Description GIT ( Spleen)

Stem A young patient had splenectomy for Splenic trauma. At time of discharge he was given
different vaccines to prevent which of the following

Item A SIRS

Item B SEPSIS

Item C OPSI

Item D Pancreatitis

Item E Thrombocytopenia

Correct answer C

Domain; Recall Interpretation Problem solving

Reference; Bailey and love Edition 26 page 1096

Prepared by;

College; Frontier Medical and Dental College, Abbottabad

Name; Prof.Dr.Johar Ali

Signature
Date;
Program MBBS

Subject General Surgery

Prof Final

TOS ID 17..4

Description GIT (Liver)

stem A young cyclist fell down and had bruising over left hypochondrium. He presented in A&E
in state of shock. After resuscitation splenectomy was done for grade IV Splenic injury.
On 3rd post operative day surgeon noticed serous fluid discharge from wound. Which
complication has occurred

Item A Gastric injury

Item B Adrenal injury

Item C Pancreatic injury

Item D Reactionary hemorrhage

Item E Wound infection

Correct answer C

Domain; Recall Interpretation Problem solving

Reference; Bailey and love Edition 26 page 1096

Prepared by;

College; Frontier Medical and Dental College, Abbottabad

Name; Prof.Dr.Johar Ali

Signature

Date;
Program MBBS

Subject General Surgery

Prof Final

TOS ID 17.2.5

Description GIT ( Gall Bladder and bile duct )

Stem A 40 year old multiparous fatty female, presented in surgical OPD with history of
recurrent attacks of pain in RHC especially after fatty meals and flatulent dyspepsia.
What is likely cause

Item A Acute cholecystitis

Item B Acute pancreatitis

Item C Acute peritonitis

Item D Biliary colic

Item E Perforated duodenal ulcer

Correct answer D

Domain; Recall Interpretation Problem solving

Reference; Bailey and love Edition 26 page 1107

Prepared by;

College; Frontier Medical and Dental College, Abbottabad

Name; Prof.Dr.Johar Ali

Signature

Date;
Program MBBS

Subject General Surgery

Prof Final

TOS ID 17.2.5

Description GIT ( Gall Bladder and bile duct )

stem A 40 year old multiparous fatty female, presented in surgical OPD with history of pain in
RHC for few hours associated with few episodes of non bilious vomiting. Examination
showed tachycardia and tenderness in RHC. What is likely cause

Item A Acute cholecystitis

Item B Acute pancreatitis

Item C Acute peritonitis

Item D Biliary colic

Item E Perforated duodenal ulcer

Correct answer A

Domain; Recall Interpretation Problem solving

Reference; Bailey and love Edition 26 page 1108

Prepared by;

College; Frontier Medical and Dental College, Abbottabad

Name; Prof.Dr.Johar Ali


Signature

Date;
Program MBBS

Subject General Surgery

Prof Final

TOS ID 17.2.5

Description GIT ( Gall Bladder and bile duct )

Stem A 40 year old multiparous fatty female, presented in surgical OPD with history of pain in
RHC for few hours associated with few episodes of non bilious vomiting. Examination
showed tachycardia and tenderness in RHC. Which investigation will help in diagnosis

Item A CXR

Item B X Ray abdomen

Item C Ultra sound abdomen

Item D TLC

Item E Barium study

Correct answer C

Domain; Recall Interpretation Problem solving

Reference; Bailey and love Edition 26 page 1107

Prepared by;

College; Frontier Medical and Dental College, Abbottabad

Name; Prof.Dr.Johar Ali

Signature
Date;
Program MBBS

Subject General Surgery

Prof Final

TOS ID 17.2.5

Description GIT ( Gall Bladder and bile duct )

stem A 35 year old female presented in OPD for complete medical examination. All
investigations were normal except for multiple small gall stones which were
asymptomatic. What is management options

Item A Open cholecystectomy

Item B Laparoscopic cholecystectomy

Item C Observation

Item D IV antibiotics

Item E IV analgesics

Correct answer C

Domain; Recall Interpretation Problem solving

Reference; Bailey and love Edition 26 page 1108

Prepared by;

College; Frontier Medical and Dental College, Abbottabad

Name; Prof.Dr.Johar Ali

Signature
Date;
Program MBBS

Subject General Surgery

Prof Final

TOS ID 17.2.5

Description GIT ( Gall Bladder and bile duct )

stem A 35 year old female presented in OPD for complete medical examination. Her Random
blood sugar was 400 mg/dl with ++ glucose in urine and multiple small gall stones which
were asymptomatic. What is management options

Item A Open cholecystectomy

Item B Laparoscopic cholecystectomy

Item C Observation

Item D IV antibiotics

Item E IV analgesics

Correct answer B

Domain; Recall Interpretation Problem solving

Reference; Bailey and love Edition 26 page 1108

Prepared by;

College; Frontier Medical and Dental College, Abbottabad

Name; Prof.Dr.Johar Ali

Signature

Date;
Program MBBS

Subject General Surgery

Prof Final

TOS ID 17.2.5

Description GIT ( Gall Bladder and bile duct )

Stem A 40 year old multiparous fatty female, presented in surgical OPD with signs of acute
cholecystitis. She was admitted and managed conservatively. When cholecystectomy
should be offered

Item A Immediate laparotomy

Item B Interval cholecystectomy

Item C After one year

Item D No need

Item E After 5 years

Correct answer B

Domain; Recall Interpretation Problem solving

Reference; Bailey and love Edition 26 page 1108

Prepared by;

College; Frontier Medical and Dental College, Abbottabad

Name; Prof.Dr.Johar Ali

Signature

Date;
Program MBBS

Subject General Surgery

Prof Final

TOS ID 17.2.5

Description GIT ( Gall Bladder and bile duct )

Stem Most common type of gall stones is

Item A Pigment stones

Item B Cholesterol stones

Item C Mixed stones

Item D Brown

Item E Black

Correct answer B

Domain; Recall Interpretation Problem solving

Reference; Bailey and love Edition 26 page 1106

Prepared by;

College; Frontier Medical and Dental College, Abbottabad

Name; Prof.Dr.Johar Ali

Signature

Date;
Program MBBS

Subject General Surgery

Prof Final

TOS ID 17.2.5

Description GIT ( Gall Bladder and bile duct )

Stem A 40 year old multiparous fatty female, presented in surgical OPD with signs of acute
cholecystitis. Which sign helps in clinical diagnosis

Item A Psoas sign

Item B Obturator sign

Item C Murphys sign

Item D Murphys punch

Item E Pointing sign

Correct answer C

Domain; Recall Interpretation Problem solving

Reference; Bailey and love Edition 26 page 1107

Prepared by;

College; Frontier Medical and Dental College, Abbottabad

Name; Prof.Dr.Johar Ali

Signature

Date;
Program MBBS

Subject General Surgery

Prof Final

TOS ID 17.2.5

Description GIT ( Gall Bladder and bile duct )

Stem A patient presented in OPD with H/O jaundice for few days. She is a known case of gall
stones. Surgeon diagnosed her as case of obstructive jaundice. Which investigation will
help

Item A ALT

Item B AST

Item C Serum Bilirubin

Item D Alkaline phosphatase

Item E Serum amylase

Correct answer D

Domain; Recall Interpretation Problem solving

Reference; Bailey and love Edition 26 page 1111

Prepared by;

College; Frontier Medical and Dental College, Abbottabad

Name; Prof.Dr.Johar Ali

Signature

Date;
Program MBBS

Subject General Surgery

Prof Final

TOS ID 17.2.5

Description GIT ( Gall Bladder and bile duct )

Stem A patient presented in OPD with H/O jaundice for few days. U/sound showed dilated
CBD & stone in lower CBD. What is treatment option

Item A Open cholecystectomy

Item B Laparoscopic cholecystectomy

Item C CBD exploration

Item D ERCP

Item E Exploratory Laparotomy

Correct answer D

Domain; Recall Interpretation Problem solving

Reference; Bailey and love Edition 26 page 1111

Prepared by;

College; Frontier Medical and Dental College, Abbottabad

Name; Prof.Dr.Johar Ali


Signature

Date;
Program MBBS

Subject General Surgery

Prof Final

TOS ID 17.2.6

Description GIT ( Pancreas )

Stem Which of the following signs is considered to be positive in acute hemorrhagic


pancreatitis

Item A Grey Turners sign

Item B Boas sign

Item C Trousseaus’ sign

Item D Courvoiser’s sign

Item E Murphy’s sign

Correct answer A

Domain; Recall Interpretation Problem solving

Reference; Bailey and love Edition 26 page 1128

Prepared by;

College; Frontier Medical and Dental College, Abbottabad.

Name; Prof.Dr.Johar Ali

Signature

Date;
Program MBBS

Subject General Surgery

Prof Final

TOS ID 17.2.6

Description GIT ( Pancreas )

Stem A 60 years old male presented in OPD with history of painless jaundice and itching for
last 2-3 weeks. He has also lost some weight. Examination revealed jaundice, pallor and
palpable mass in epigastrium which is hard and fixed and resonant on percussion. What
is likely diagnosis

Item A Acute cholangitis

Item B Ca Head of pancreas

Item C Chronic pancreatitis

Item D Hepatocellular CA

Item E CA Stomach

Correct answer B

Domain; Recall Interpretation Problem solving

Reference; Bailey and love Edition 26 page 1137

Prepared by;

College; Frontier Medical and Dental College, Abbottabad

Name; Prof.Dr.Johar Ali

Signature
Date;
Program MBBS

Subject General Surgery

Prof Final

TOS ID 17.2.6

Description GIT ( Pancreas )

Stem A 45 year old female presented in OPD with upper abdominal pain radiating to back for
last 3 days. She is a known case of gall stones. On examination abdomen is distended and
tender in epigastrium. S/lipase is increased. What is likely diagnosis

Item A Biliary colic

Item B Acute cholecystitis

Item C Acute pancreatitis

Item D Acute hepatitis

Item E Acute gastritis

Correct answer C

Domain; Recall Interpretation Problem solving

Reference; Bailey and love Edition 26 page 1127

Prepared by;

College; Frontier Medical and Dental College, Abbottabad

Name; Prof.Dr.Johar Ali

Signature

Date;
Program MBBS

Subject General Surgery

Prof Final

TOS ID 17.2.6

Description GIT ( Pancreas)

stem A10 year old girl presented in surgical OPD with History of Upper abdominal pain and
swelling for few days. It is also associated with nausea and vomiting. Examination reveals
distended epigastrium with smooth mass with ill defined edges that are resonant on
percussion. What is likely diagnosis

Item A CA head of pancreas

Item B Acute Cholecystitis

Item C CA Stomach

Item D Pancreatic Pseudocyst

Item E Empyema Gall bladder

Correct answer D

Domain; Recall Interpretation √ Problem solving

Reference; Bailey and love Edition 26 page 1133

Prepared by;

College; Frontier Medical and Dental College, Abbottabad

Name; Prof.Dr.Johar Ali

Signature

Date;
Program MBBS

Subject General Surgery

Prof Final

TOS ID 17.2.6

Description GIT ( Pancreas)

stem Ectopic pancreatic tissue can be seen in which of the following

Item A Esophagus

Item B Tonsils

Item C Cecum

Item D Meckels diverticulum

Item E Lesser sac

Correct answer D

Domain; Recall √ Interpretation Problem solving

Reference; Bailey and love Edition 26 page 1125

Prepared by;

College; Frontier Medical and Dental College, Abbottabad

Name; Prof.Dr.Johar Ali

Signature
Date;
Program MBBS

Subject General Surgery

Prof Final

TOS ID 17.2.6

Description GIT (Pancreas)

stem A 58year old female was diagnosed as case of acute pancreatitis. Investigations were
TLC 17000, BSR 250 mg/dl, LDH 720, AST 150, S/Ca 1.5 mmol/L and oxygen saturation 75
%. What is Ranson score

Item A 2

Item B 3

Item C 4

Item D 5

Item E 6

Correct answer D

Domain; Recall Interpretation Problem solving √

Reference; Bailey and love Edition 26 page 1129

Prepared by;

College; Frontier Medical and Dental College, Abbottabad

Name; Prof.Dr.Johar Ali

Signature

Date;
Program MBBS

Subject General Surgery

Prof Final

TOS ID 17.2.6

Description GIT ( Pancreas)

stem A 58year old female was diagnosed as case of acute pancreatitis. Which of the following
criteria is most effective in determining severity index

Item A Glasgow

Item B Ranson

Item C APACHE

Item D POSSUM

Item E Parsonet

Correct answer C

Domain; Recall √ Interpretation Problem solving

Reference; Bailey and love Edition 26 page 1128

Prepared by;

College; Frontier Medical and Dental College, Abbottabad

Name; Prof.Dr.Johar Ali

Signature

Date;
Program MBBS

Subject General Surgery

Prof Final

TOS ID 17.2.8

Description GIT ( Small and large intestine)

Stem A 60 year old female complains of feeling tired while doing routine household work for
last 8 weeks. Examination showed pallor and mass in RIF. Her Hb was 7.5 gm/dl. What is
likely diagnosis

Item A CA Stomach

Item B Appendicular mass

Item C Rectus sheath hematoma

Item D CA Cecum

Item E Mittleschemerz

Correct answer D

Domain; Recall Interpretation √ Problem solving

Reference; Bailey and love Edition 26 page 1166

Prepared by;

College; Frontier Medical and Dental College, Abbottabad

Name; Prof.Dr.Johar Ali

Signature
Date;
Program MBBS

Subject General Surgery

Prof Final

TOS ID 17.2.8

Description GIT ( Small and large intestine)

Stem A young patient presented in OPD having purulent discharge from peri anal region for
last 3 months. He also gave history of fistulotomy 1 year ago. He is also having loose
motions containing blood and mucous. What is most likely cause

Item A Diverticular disease

Item B CA rectum

Item C Vesicorectal fistula

Item D Crohns disease

Item E Gluteal abscess

Correct answer D

Domain; Recall Interpretation √ Problem solving

Reference; Bailey and love Edition 26 page 1152

Prepared by;

College; Frontier Medical and Dental College, Abbottabad

Name; Prof.Dr.Johar Ali

Signature

Date;
Program MBBS

Subject General Surgery

Prof Final

TOS ID 17.2.8

Description GIT ( Small and large intestine)

Stem A 65 year old male presents in surgical OPD complaining of passage of flatus and stools
in urine. What is pathology

Item A Vesicourethral fistula

Item B Enteroenteral fistula

Item C Enterovesical fistula

Item D Enterocutaneous fistula

Item E Vesicocutaneous fistula

Correct answer D

Domain; Recall Interpretation √ Problem solving

Reference; Bailey and love Edition 26 page 1170

Prepared by;

College; Frontier Medical and Dental College, Abbottabad

Name; Prof.Dr.Johar Ali

Signature

Date;
Program MBBS

Subject General Surgery

Prof Final

TOS ID 17.2.8

Description GIT ( Small and large intestine)

Stem Intestinal TB affects which part of gut commonly

Item A Stomach

Item B DJ junction

Item C Ileocecal junction

Item D Transverse colon

Item E Sigmoid colon

Correct answer C

Domain; Recall √ Interpretation Problem solving

Reference; Bailey and love Edition 26 page 1158

Prepared by;

College; Frontier Medical and Dental College, Abbottabad

Name; Prof.Dr.Johar Ali

Signature

Date;
Program MBBS

Subject General Surgery

Prof Final

TOS ID 17.2.12

Description GIT ( Anus and anal canal)

Stem A young patient presented in OPD having purulent discharge from peri anal region for
last 3 months. Examination revealed swelling at 9 O clock positions with signs of
inflammation. What is likely diagnosis

Item A Peri anal fistula

Item B Anal Fissure

Item C Hemorrhoids

Item D Peri anal abscess

Item E Rectal cancer

Correct answer D

Domain; Recall Interpretation √ Problem solving

Reference; Bailey and love Edition 26 page 1259

Prepared by;

College; Frontier Medical and Dental College, Abbottabad

Name; Prof.Dr.Johar Ali

Signature

Date;
Program MBBS

Subject General Surgery

Prof Final

TOS ID 17.2.9

Description GIT ( Intestinal Obstruction)

Stem Which of the following is type of adynamic intestinal obstruction

Item A Adhesions

Item B Paralytic ileus

Item C Worms

Item D Stones

Item E Tumor

Correct answer B

Domain; Recall √ Interpretation Problem solving

Reference; Bailey and love Edition 26 page 1193

Prepared by;

College; Frontier Medical and Dental College, Abbottabad

Name; Prof.Dr.Johar Ali

Signature

Date;
Program MBBS

Subject General Surgery

Prof Final

TOS ID 17.2.9

Description GIT (Intestinal obstruction)

Stem Intusussception most commonly occurs in which age group

Item A < 4 months

Item B 5-10 months

Item C 1-2 year

Item D 2-5 year

Item E > 5 year

Correct answer B

Domain; Recall √ Interpretation Problem solving

Reference; Bailey and love Edition 26 page 114

Prepared by;

College; Frontier Medical and Dental College, Abbottabad

Name; Prof.Dr.Johar Ali

Signature

Date;
Program MBBS

Subject General Surgery

Prof Final

TOS ID 17.2.9

Description GIT ( Intestinal obstruction)

Stem A 12 year old female presents in OPD with abdominal mass and vomiting for last few
days. Examination reveals distended epigastrium with firm mass that is mobile and non
Epigastric hernia tender. What is likely diagnosis

Item A Undigested fruit and vegetables

Item B Hairs

Item C Congenital Hypertrophic Pyloric stenosis

Item D Worms

Item E Annular Pancreas

Correct answer B

Domain; Recall Interpretation √ Problem solving

Reference; Bailey and love Edition 26 page 1183

Prepared by;

College; Frontier Medical and Dental College, Abbottabad

Name; Prof.Dr.Johar Ali


Signature

Date;
Program MBBS

Subject General Surgery

Prof Final

TOS ID 17.2.10

Description GIT ( Appendix)

Stem A 5 year old female was diagnosed as having acute appendicitis. On examination when
surgeon presses LIF patient experiences pain in RIF. What is sign called

Item A Pointing

Item B Rovsings

Item C Psoas

Item D Obturator

Item E Murphy’s

Correct answer B

Domain; Recall Interpretation √ Problem solving

Reference; Bailey and love Edition 26 page 1203

Prepared by;

College; Frontier Medical and Dental College, Abbottabad

Name; Prof.Dr.Johar Ali

Signature

Date;
Program MBBS

Subject General Surgery

Prof Final

TOS ID 17.2.10

Description GIT ( appendix)

stem A 15 year old female presented in OPD having pain in Umbilicus that radiated to RIF. She
is also having nausea and anorexia. TLC was 11500.On examination when surgeon
presses LIF patient experiences pain in RIF and also tenderness and rebound tenderness
in RIF. What is Alvarado score

Item A 2

Item B 3

Item C 5

Item D 7

Item E 8

Correct answer E

Domain; Recall Interpretation Problem solving √

Reference; Bailey and love Edition 26 page 1207

Prepared by;

College; Frontier Medical and Dental College, Abbottabad

Name; Prof.Dr.Johar Ali


Signature

Date;
Program MBBS

Subject General Surgery

Prof Final

TOS ID 17.2.10

Description GIT ( Appendix)

Stem A 15 year old female presented in OPD having pain in Umbilicus that radiated to RIF. Her
TLC was 7500.On examination when surgeon presses LIF patient experiences pain in RIF.
There was also tenderness and rebound tenderness in RIF. What is Alvarado score

Item A 2

Item B 3

Item C 4

Item D 5

Item E 6

Correct answer C

Domain; Recall Interpretation Problem solving √

Reference; Bailey and love Edition 26 page 1207

Prepared by;

College; Frontier Medical and Dental College, Abbottabad

Name; Prof.Dr.Johar Ali

Signature
Date;
Program MBBS

Subject General Surgery

Prof Final

TOS ID 17.2.10

Description GIT ( Appendix)

Stem A 20 year old female presented on surgical OPD having pain in RIF for few hours. Pain
started suddenly and radiates in lower abdomen. Her LPMP was 2 weeks ago.
Examination was unremarkable. What is likely cause

Item A Diverticulitis

Item B Mittleschemerz

Item C Perforated peptic ulcer

Item D Ruptured ectopic pregnancy

Item E CA Cecum

Correct answer B

Domain; Recall Interpretation √ Problem solving

Reference; Bailey and love Edition 26 page 1205

Prepared by;

College; Frontier Medical and Dental College, Abbottabad

Name; Prof.Dr.Johar Ali

Signature

Date;
Program MBBS

Subject General Surgery

Prof Final

TOS ID 17.2.10

Description GIT (Appendix)

Stem A 15 year old female presented in OPD having pain in Umbilicus that radiated to RIF. She
is also having nausea and anorexia. TLC was 11500.On examination when surgeon
presses LIF patient experiences pain in RIF and also tenderness and rebound tenderness
in RIF. What is likely diagnosis

Item A Acute Appendicitis

Item B Right ureteric colic

Item C Ruptured ectopic pregnancy

Item D Mittleschemerz

Item E Perforated peptic ulcer

Correct answer A

Domain; Recall Interpretation √ Problem solving

Reference; Bailey and love Edition 26 page 1203

Prepared by;

College; Frontier Medical and Dental College, Abbottabad

Name; Prof.Dr.Johar Ali

Signature

Date;
Program MBBS

Subject General Surgery

Prof Final

TOS ID 17.2.10

Description GIT (Appendix)

stem A 15 year old female presented in OPD having pain in Umbilicus that radiated to RIF. Her
TLC was 7500.On examination when surgeon presses LIF patient experiences pain in RIF.
There was also tenderness and rebound tenderness in RIF. What is treatment

Item A No treatment

Item B Conservative management

Item C Appendicectomy

Item D Exploratory Laparotomy

Item E Right Hemicolectomy

Correct answer A

Domain; Recall Interpretation √ Problem solving

Reference; Bailey and love Edition 26 page 1207

Prepared by;

College; Frontier Medical and Dental College, Abbottabad

Name; Prof.Dr.Johar Ali

Signature

Date;
Program MBBS

Subject General Surgery

Prof Final

TOS ID 17.2.10

Description GIT (Appendix)

stem An 18 year old female complains of generalized colicky abdominal pain for few hours. She
is anorexic and also has vomited for couple of times. Examination reveals tenderness and
rebound tenderness in RIF. What is treatment option

Item A No treatment

Item B Conservative management

Item C Appendicectomy

Item D Exploratory Laparotomy

Item E Right Hemicolectomy

Correct answer C

Domain; Recall Interpretation Problem solving √

Reference; Bailey and love Edition 26 page 1208

Prepared by;

College; Frontier Medical and Dental College, Abbottabad

Name; Prof.Dr.Johar Ali

Signature

Date;
Program MBBS

Subject General Surgery

Prof Final

TOS ID 17.2.10

Description GIT (Appendix)

stem An 18 year old female complains of generalized colicky abdominal pain for few hours. She
is anorexic and also has vomited for couple of times. Examination reveals tenderness and
rebound tenderness in RIF. What is treatment option

Item A No treatment

Item B Conservative management

Item C Appendicectomy

Item D Exploratory Laparotomy

Item E Right Hemicolectomy

Correct answer C

Domain; Recall Interpretation Problem solving √

Reference; Bailey and love Edition 26 page 1208

Prepared by;

College; Frontier Medical and Dental College, Abbottabad

Name; Prof.Dr.Johar Ali

Signature

Date;
Program MBBS

Subject General Surgery

Prof Final

TOS ID 17.2.10

Description GIT ( Appendix)

stem A 30 year old male was admitted with appendicular mass. After 24 hours of conservative
management he developed high grade fever and increase in size of mass. What has
happened

Item A Carcinoid tumor

Item B CA Cecum

Item C Appendicular Abscess

Item D Generalized Peritonitis

Item E Maloney gangrene

Correct answer C

Domain; Recall Interpretation Problem solving√

Reference; Bailey and love Edition 26 page 1211

Prepared by;

College; Frontier Medical and Dental College, Abbottabad

Name; Prof.Dr.Johar Ali

Signature
Date;
Program MBBS

Subject General Surgery

Prof Final

TOS ID 17.2.11

Description GIT ( Rectum)

Stem A 65 year old male complains of early morning diarrhea for last 4 months. He is also
having tenesmus and bleeding per rectum. He has also lost some weight. What is likely
cause

Item A 2nd degree Hemorrhoids

Item B Anal Fissure

Item C CA Rectum

Item D CA Cecum

Item E Diverticular disease

Correct answer C

Domain; Recall Interpretation √ Problem solving

Reference; Bailey and love Edition 26 page 1227

Prepared by;

College; Frontier Medical and Dental College, Abbottabad

Name; Prof.Dr.Johar Ali

Signature

Date;
Program MBBS

Subject General Surgery

Prof Final

TOS ID 17.2.11

Description GIT (Rectum)

Stem A 62 year old male complains of early morning diarrhea for last 4 months. He is also
having tenesmus and bleeding per rectum. He has also lost some weight. Which anterior
examination will help in diagnosis

Item A DPL

Item B Anoscopy

Item C Thoracoscopy

Item D Sigmoidoscopy

Item E Gastro-duodenoscopy

Correct answer D

Domain; Recall Interpretation Problem solving √

Reference; Bailey and love Edition 26 page 1228

Prepared by;

College; Frontier Medical and Dental College, Abbottabad

Name; Prof.Dr.Johar Ali

Signature

Date;
Program MBBS

Subject General Surgery

Prof Final

TOS ID 17.2.11

Description GIT (Rectum)

Stem A 62 year old male complains of early morning diarrhea for last 4 months. He is also
having tenesmus and bleeding per rectum. He has also lost some weight. Sigmoidoscopy
showed growth in upper part of rectum. Which investigation will stage the disease

Item A U/Sound abdomen

Item B CXR (PA view)

Item C CT Scan Abdomen and pelvis

Item D Angiography

Item E MRCP

Correct answer C

Domain; Recall Interpretation Problem solving√

Reference; Bailey and love Edition 26 page 1228

Prepared by;

College; Frontier Medical and Dental College, Abbottabad

Name; Prof.Dr.Johar Ali

Signature

Date;
Program MBBS

Subject General Surgery

Prof Final

TOS ID 17.2.11

Description GIT (Rectum)

stem A mother brought her 4 year old boy with history of fresh bleeding per rectum and
something coming out of rectum. What is likely cause

Item A Anal Fissure

Item B Hemorrhoids

Item C Rectal polyp

Item D CA Rectum

Item E Proctitis

Correct answer C

Domain; Recall Interpretation √ Problem solving

Reference; Bailey and love Edition 26 page 1224

Prepared by;

College; Frontier Medical and Dental College, Abbottabad

Name; Prof.Dr.Johar Ali

Signature

Date;
Program MBBS

Subject General Surgery

Prof Final

TOS ID 17.2.11

Description GIT (Anus and Anal canal)

Stem A 28 year old male presented with severe anal pain during defecation. He has also streaks
of fresh blood per rectum and history of chronic constipation. What is likely diagnosis

Item A Hemorrhoids

Item B Proctitis

Item C Anal Fissure

Item D Diverticulitis

Item E Ischemic Colitis

Correct answer C

Domain; Recall Interpretation √ Problem solving

Reference; Bailey and love Edition 26 page 1248

Prepared by;

College; Frontier Medical and Dental College, Abbottabad

Name; Prof.Dr.Johar Ali

Signature

Date;
Program MBBS

Subject General Surgery

Prof Final

TOS ID 17.2.11

Description GIT (Anus and Anal canal)

Stem A 45 year old female presented in surgical OPD complaining of bleeding per rectum with
something coming out of rectum that needs manual reduction. She is also having chronic
constipation. What is likely cause

Item A Anal fissure

Item B Hemorrhoids

Item C CA Rectum

Item D Colonic polyp

Item E Anal wart

Correct answer B

Domain; Recall Interpretation √ Problem solving

Reference; Bailey and love Edition 26 page 1251

Prepared by;

College; Frontier Medical and Dental College, Abbottabad

Name; Prof.Dr.Johar Ali

Signature

Date;
Program MBBS

Subject General Surgery

Prof Final

TOS ID 17.2.11

Description GIT (Anus and Anal canal)

Stem A 45 year old female presented in surgical OPD complaining of bleeding per rectum with
something coming out of rectum that needs manual reduction. She is also having chronic
constipation. What is degree of hemorrhoids

Item A Ist Degree

Item B 2nd degree

Item C 3rd degree

Item D 4th degree

Item E 5th degree

Correct answer C

Domain; Recall Interpretation √ Problem solving

Reference; Bailey and love Edition 26 page 1252

Prepared by;

College; Frontier Medical and Dental College, Abbottabad

Name; Prof.Dr.Johar Ali

Signature

Date;
Program MBBS

Subject General Surgery

Prof Final

TOS ID 17.2.11

Description GIT (Anus and Anal canal)

Stem A 45 year old female presented in surgical OPD complaining of bleeding per rectum with
something coming out of rectum that needs manual reduction. She is also having chronic
constipation. What is treatment option

Item A Conservative management

Item B Inj Sclerotherapy

Item C Rubber band ligation

Item D Hemorrhoidectomy

Item E Thiersch stitch

Correct answer D

Domain; Recall Interpretation Problem solving √

Reference; Bailey and love Edition 26 page 1253

Prepared by;

College; Frontier Medical and Dental College, Abbottabad

Name; Prof.Dr.Johar Ali

Signature

Date;
Program MBBS

Subject General Surgery

Prof Final

TOS ID 17.2.11

Description GIT (Anus and Anal canal)

Stem A 45 year old female presented in surgical OPD complaining of bleeding per rectum with
something coming out of rectum that reduces spontaneously. She is also having chronic
constipation. What is treatment option

Item A Conservative management

Item B Inj Sclerotherapy

Item C Rubber band ligation

Item D Hemorrhoidectomy

Item E Thiersch stitch

Correct answer C

Domain; Recall Interpretation Problem solving √

Reference; Bailey and love Edition 26 page 1253

Prepared by;

College; Frontier Medical and Dental College, Abbottabad

Name; Prof.Dr.Johar Ali

Signature

Date;
Program MBBS

Subject General Surgery

Prof Final

TOS ID 17.2.11

Description GIT (Anus and Anal canal)

Stem A 45 year old female presented in surgical OPD complaining of bleeding per rectum and
nothing coming out of rectum. She is also having chronic constipation. Proctoscopy
showed 1st degree hemorrhoids. What is treatment option

Item A Lords operation

Item B Inj Sclerotherapy

Item C Rubber band ligation

Item D Hemorrhoidectomy

Item E Thiersch stitch

Correct answer B

Domain; Recall Interpretation Problem solving √

Reference; Bailey and love Edition 26 page 1252

Prepared by;

College; Frontier Medical and Dental College, Abbottabad

Name; Prof.Dr.Johar Ali

Signature

Date;
Program MBBS

Subject General Surgery

Prof Final

TOS ID 17.2.11

Description GIT (Anus and Anal canal)

Stem A 28 year old male presented with severe anal pain during defecation. He has also streaks
of fresh blood per rectum and history of chronic constipation. What is treatment option

Item A Lords Operation

Item B Inj Sclerotherapy

Item C Lateral internal sphincterotomy

Item D Hemorrhoidectomy

Item E Thiersch stitch

Correct answer C

Domain; Recall Interpretation Problem solving √

Reference; Bailey and love Edition 26 page 1249

Prepared by;

College; Frontier Medical and Dental College, Abbottabad

Name; Prof.Dr.Johar Ali

Signature

Date;
Program MBBS

Subject General Surgery

Prof Final

TOS ID 17.2.11

Description GIT (Anus and Anal canal)

Stem A 45 year old female presented in surgical OPD complaining of bleeding per rectum with
something coming out of rectum that needs manual reduction. She is also having chronic
constipation. What is treatment option

Item A Conservative management

Item B Inj Sclerotherapy

Item C Rubber band ligation

Item D Hemorrhoidectomy

Item E Thiersch stitch

Correct answer D

Domain; Recall Interpretation Problem solving √

Reference; Bailey and love Edition 26 page 1253

Prepared by;

College; Frontier Medical and Dental College, Abbottabad

Name; Prof.Dr.Johar Ali

Signature

Date;
Program MBBS

Subject General Surgery

Prof Final

TOS ID 17.2.11

Description GIT (Anus and Anal canal)

Stem A 28 year old male presented with severe anal pain during defecation. She was operated
for anal fissure. Post operative she developed flatus incontinence. Which procedure was
done

Item A Lateral internal sphincterotomy

Item B Inj Sclerotherapy

Item C Rubber band ligation

Item D Hemorrhoidectomy

Item E Thiersch stitch

Correct answer A

Domain; Recall Interpretation Problem solving √

Reference; Bailey and love Edition 26 page 1249

Prepared by;

College; Frontier Medical and Dental College, Abbottabad

Name; Prof.Dr.Johar Ali

Signature

Date;
Program MBBS

Subject General Surgery

Prof Final

TOS ID 17.2.11

Description GIT (Anus and Anal canal)

Stem A 45 year old female presented in surgical OPD complaining of bleeding per rectum with
something coming out of rectum that needs manual reduction. She is also having chronic
constipation. Which of the following will help in diagnosis

Item A Ultra sound pelvis

Item B Sinogram

Item C Colonoscope

Item D Proctoscope

Item E MRI

Correct answer D

Domain; Recall Interpretation Problem solving √

Reference; Bailey and love Edition 26 page 1240

Prepared by;

College; Frontier Medical and Dental College, Abbottabad

Name; Prof.Dr.Johar Ali

Signature

Date;
BAHRIA UNIVERSITY ISLAMABAD

MULTIPLE CHOICE QUESTIONS


PROGRAM MBBS
General Surgery
SUBJECT
'PROF Final Professional
TOS _ID 17.5.1

Description Thyroid gland and the thyroglossal tract


STEM:
The parafollicular ‘C’ cells of the thyroid gland arise from the
ITEM A Ultimobranchial body
ITEM B Neural crest
ITEM C Thyroglossal duct
ITEM D Fourth pharyngeal pouch
ITEM E

. Correct Answer: B

Domain: Recall Interpretation Problem Solving

Reference(s):

PREPPARED BY: College: Frontier Medical & Dental College, Abbottabad

Name: Prof. Dr. Gul Muhammad

Signature:

Date: 22.6.2013

Note:
Diagrams/Charts/Graphs if any should attach along this.
BAHRIA UNIVERSITY ISLAMABAD

MULTIPLE CHOICE QUESTIONS


PROGRAM MBBS
General Surgery
SUBJECT
'PROF Final Professional
TOS _ID 17.5.1

Description Thyroid gland and the thyroglossal tract


STEM: A patient known to be hypothyroid presents with an altered mental state and
hypothermia. Which of the following is of least value in the management of
this condition?
ITEM A Intravenous T3
ITEM B Slow rewarming
ITEM C Propranolol
ITEM D Hydrocortisone
ITEM E

.
Correct Answer: C

Domain: Recall Interpretation Problem Solving

Reference(s):

PREPPARED BY: College: Frontier Medical & Dental College, Abbottabad

Name: Prof. Dr. Gul Muhammad

Signature:

Date: 22.6.2013

Note:
Diagrams/Charts/Graphs if any should attach along this.
BAHRIA UNIVERSITY ISLAMABAD

MULTIPLE CHOICE QUESTIONS


PROGRAM MBBS
General Surgery
SUBJECT
'PROF Final Professional
TOS _ID 17.5.1

Description Thyroid gland and the thyroglossal tract


STEM:
Within hours of surgery for thyrotoxicosis a patient develops respiratory
distress. Which of the following is the most likely cause?
ITEM A Laryngeal edema following tension hematoma
ITEM B Unilateral laryngeal nerve palsy
ITEM C Bilateral laryngeal nerve plasy
ITEM D
ITEM E

. Correct Answer: C

Domain: Recall Interpretation Problem Solving

Reference(s):

PREPPARED BY: College: Frontier Medical & Dental College, Abbottabad

Name: Prof. Dr. Gul Muhammad

Signature:

Date: 22.6.2013

Note:
Diagrams/Charts/Graphs if any should attach along this.
BAHRIA UNIVERSITY ISLAMABAD

MULTIPLE CHOICE QUESTIONS


PROGRAM MBBS
General Surgery
SUBJECT
'PROF Final Professional
TOS _ID 17.5.1

Description Thyroid gland and the thyroglossal tract


STEM:
As compared to secondary thyrotoxicosis, which of the following features is
NOT true for primary thyrotoxicosis?
ITEM A Hyperthyroidism is more severe
ITEM B Cardiac failure is less common
ITEM C Orbital proptosis is more common
ITEM D Pretibial myxedema is less common
ITEM E

. Correct Answer: D

Domain: Recall Interpretation Problem Solving

Reference(s):

PREPPARED BY: College: Frontier Medical & Dental College, Abbottabad

Name: Prof. Dr. Gul Muhammad

Signature:

Date: 22.6.2013

Note:
Diagrams/Charts/Graphs if any should attach along this.
BAHRIA UNIVERSITY ISLAMABAD

MULTIPLE CHOICE QUESTIONS


PROGRAM MBBS
General Surgery
SUBJECT
'PROF Final Professional
TOS _ID 17.5.1

Description Thyroid gland and the thyroglossal tract


STEM: 24 hours after thyroidectomy for diffuse toxic goiter, a patient develops
hyperpyrexia, marked tachycardia, and restlessness. Which of the following
measures is LEAST useful in management
ITEM A Propanolol
ITEM B Iodine
ITEM C Antibiotics
ITEM D Steroids
ITEM E

.
Correct Answer: C

Domain: Recall Interpretation Problem Solving

Reference(s):

PREPPARED BY: College: Frontier Medical & Dental College, Abbottabad

Name: Prof. Dr. Gul Muhammad

Signature:

Date: 22.6.2013

Note:
Diagrams/Charts/Graphs if any should attach along this.
BAHRIA UNIVERSITY ISLAMABAD

MULTIPLE CHOICE QUESTIONS


PROGRAM MBBS
General Surgery
SUBJECT
'PROF Final Professional
TOS _ID 17.5.2

Description Parathyroid and Adrenal Glands


STEM:
The following are all features of primary hyperaldosteronis. Mark the wrong
statement
ITEM A It is common in women than in men
ITEM B Most cases are due to hyperplasia of the zona glomerulosa
ITEM C There is increased potassium excretion in the urine
ITEM D Spironolactone is the first line medical treatment
ITEM E

. Correct Answer: B

Domain: Recall Interpretation Problem Solving

Reference(s):

PREPPARED BY: College: Frontier Medical & Dental College, Abbottabad

Name: Prof. Dr. Gul Muhammad

Signature:

Date: 22.6.2013

Note:
Diagrams/Charts/Graphs if any should attach along this.
BAHRIA UNIVERSITY ISLAMABAD

MULTIPLE CHOICE QUESTIONS


PROGRAM MBBS
General Surgery
SUBJECT
'PROF Final Professional
TOS _ID 17.5.2

Description Parathyroid and Adrenal Glands


STEM:
The most common site for extra-adrenal catecholamine secreting
paragangliomas is
ITEM A Pelvis
ITEM B Abdominal para-aortic region
ITEM C Thorax para-aortic
ITEM D Urinary bladder
ITEM E

. Correct Answer: D

Domain: Recall Interpretation Problem Solving

Reference(s):

PREPPARED BY: College: Frontier Medical & Dental College, Abbottabad

Name: Prof. Dr. Gul Muhammad

Signature:

Date: 22.6.2013

Note:
Diagrams/Charts/Graphs if any should attach along this.
BAHRIA UNIVERSITY ISLAMABAD

MULTIPLE CHOICE QUESTIONS


PROGRAM MBBS
General Surgery
SUBJECT
'PROF Final Professional
TOS _ID 17.5.2

Description Parathyroid and Adrenal Glands


STEM:
Congenital adrenal hyperplasia
ITEM A Is an autosomal dominant disorder
ITEM B Most commonly occurs due to a deficiency of 21-hydroxylase
ITEM C Results in an isolated increase in androgenic cortisol
ITEM D Is associated with low ACTH levels
ITEM E

. Correct Answer: B

Domain: Recall Interpretation Problem Solving

Reference(s):

PREPPARED BY: College: Frontier Medical & Dental College, Abbottabad

Name: Prof. Dr. Gul Muhammad

Signature:

Date: 22.6.2013

Note:
Diagrams/Charts/Graphs if any should attach along this.
BAHRIA UNIVERSITY ISLAMABAD

MULTIPLE CHOICE QUESTIONS


PROGRAM MBBS
General Surgery
SUBJECT
'PROF Final Professional
TOS _ID

Description
STEM:
Prevention of organ failure by preventing multiple organ dysfunction
syndrome (MODS), the targets for intervention are
ITEM A Established microvascular occlusion
ITEM B Tissue hypoxia
ITEM C Cellular dysfunction
ITEM D Abnormal arachidonic acid metabolism
ITEM E Coagulation and complement activation

. Correct Answer: A

Domain: Recall Interpretation Problem Solving

Reference(s): Bailey & Love 24 edition, Page No. 13

PREPPARED BY: College: Frontier Medical & Dental College, Abbottabad

Name: Prof. Dr. Gul Muhammad

Signature:

Date: 22.6.2013

Note:
Diagrams/Charts/Graphs if any should attach along this.
BAHRIA UNIVERSITY ISLAMABAD

MULTIPLE CHOICE QUESTIONS


PROGRAM MBBS
General Surgery
SUBJECT
'PROF Final Professional
TOS _ID

Description Pharynx, Larynx, and Neck


STEM:
A cystic hygroma
ITEM A Usually presents in the neonate
ITEM B Contains sequestered blood
ITEM C Contains fresh blood
ITEM D Usually disappears spontaneously
ITEM E

. Correct Answer: A

Domain: Recall Interpretation Problem Solving

Reference(s):

PREPPARED BY: College: Frontier Medical & Dental College, Abbottabad

Name: Prof. Dr. Gul Muhammad

Signature:

Date: 22.6.2013

Note:
Diagrams/Charts/Graphs if any should attach along this.
BAHRIA UNIVERSITY ISLAMABAD

MULTIPLE CHOICE QUESTIONS


PROGRAM MBBS
General Surgery
SUBJECT
'PROF Final Professional
TOS _ID 17.5.1

Description Throid gland & thyroglassal Tract


STEM:
The parafollicular ‘C’ cells of the thyroid gland arise from the
ITEM A Ultimobranchial body
ITEM B Neural crest
ITEM C Thyroglossal duct
ITEM D Fourth pharyngeal pouch
ITEM E

. Correct Answer: B

Domain: Recall Interpretation Problem Solving

Reference(s):

PREPPARED BY: College: Frontier Medical & Dental College, Abbottabad

Name: Prof. Dr. Gul Muhammad

Signature:

Date: 22.6.2013

Note:
Diagrams/Charts/Graphs if any should attach along this.
BAHRIA UNIVERSITY ISLAMABAD

MULTIPLE CHOICE QUESTIONS


PROGRAM MBBS
General Surgery
SUBJECT
'PROF Final Professional
TOS _ID 17.5.1

Description Thyroid gland & thyroglassal Tract


STEM:
Within hours of surgery for thyrotoxicosis, a patient develops respiratory
distress. Which of the following is the most likely cause?
ITEM A Laryngeal edema following tension hematoma
ITEM B Unilateral laryngeal nerve plasy
ITEM C Bilateral faryngeal nerve plasy
ITEM D Laryngeal spasm due to hypoparathyroidism
ITEM E

. Correct Answer: A

Domain: Recall Interpretation Problem Solving

Reference(s):

PREPPARED BY: College: Frontier Medical & Dental College, Abbottabad

Name: Prof. Dr. Gul Muhammad

Signature:

Date: 22.6.2013

Note:
Diagrams/Charts/Graphs if any should attach along this.
BAHRIA UNIVERSITY ISLAMABAD

MULTIPLE CHOICE QUESTIONS


PROGRAM MBBS
General Surgery
SUBJECT
'PROF Final Professional
TOS _ID 17.5.1

Description Throid gland & thyroglassal Tract


STEM:
A young man presents with a solitary thyroid nodule, which of the following is
of LEAST value
ITEM A Thyroid function tests
ITEM B Ultrasonography
ITEM C Radioisotope scan
ITEM D FNAC
ITEM E

. Correct Answer: C

Domain: Recall Interpretation Problem Solving

Reference(s):

PREPPARED BY: College: Frontier Medical & Dental College, Abbottabad

Name: Prof. Dr. Gul Muhammad

Signature:

Date: 22.6.2013

BAHRIA UNIVERSITY ISLAMABAD

MULTIPLE CHOICE QUESTIONS


PROGRAM MBBS
General Surgery
SUBJECT
'PROF Final Professional
TOS _ID 17.5.3

Description Salivary Glands


STEM:
Chronic parotitis in children is almost pathognomonic of
ITEM A Syphilis
ITEM B Tubrculosis
ITEM C AIDs
ITEM D Scrcoidosis
ITEM E Sjogren’s syndrome

. Correct Answer: C

Domain: Recall Interpretation Problem Solving

Reference(s): Bailey & Love’s page# 654

PREPPARED BY: College: Frontier Medical & Dental College, Abbottabad

Name: Prof. Dr. Gul Muhammad

Signature:

Date: 22.6.2013

Note:
Diagrams/Charts/Graphs if any should attach along this.
BAHRIA UNIVERSITY ISLAMABAD

MULTIPLE CHOICE QUESTIONS


PROGRAM MBBS
General Surgery
SUBJECT
'PROF Final Professional
TOS _ID 17.5.3

Description Salivary Glands


STEM:
Warthin’s tumour refers to
ITEM A Pleomorphic adenoma
ITEM B Adenolymphioma
ITEM C Mucoepidermoid tumour
ITEM D Adenloid cystic carcinoma
ITEM E

. Correct Answer: B

Domain: Recall Interpretation Problem Solving

Reference(s): Bailey & Love’s page# 659

PREPPARED BY: College: Frontier Medical & Dental College, Abbottabad

Name: Prof. Dr. Gul Muhammad

Signature:

Date: 22.6.2013

Note:
Diagrams/Charts/Graphs if any should attach along this.
BAHRIA UNIVERSITY ISLAMABAD

MULTIPLE CHOICE QUESTIONS


PROGRAM MBBS
General Surgery
SUBJECT
'PROF Final Professional
TOS _ID 17.5.3

Description Salivary Glands


STEM:
Sjogren’s syndrome accompanies. Mark the wrong statement
ITEM A Rheumatoid arthritis
ITEM B SLE
ITEM C Chronic active hepatitis
ITEM D Primary biliary cirrhosis
ITEM E

. Correct Answer: C

Domain: Recall Interpretation Problem Solving

Reference(s): Bailey & Love’s page# 669

PREPPARED BY: College: Frontier Medical & Dental College, Abbottabad

Name: Prof. Dr. Gul Muhammad

Signature:

Date: 22.6.2013

Note:
Diagrams/Charts/Graphs if any should attach along this.
BAHRIA UNIVERSITY ISLAMABAD

MULTIPLE CHOICE QUESTIONS


PROGRAM MBBS
General Surgery
SUBJECT
'PROF Final Professional
TOS _ID 17.5.1

Description Thyroid gland & thyroglassal tract


STEM:
Which of the following statements regarding thyroglassal cyst is ture
ITEM A This may be present in any part of the thyroglassal tract
ITEM B Such a cyst occupies the midline except in the region of the thyroid cartilage
where the thyroglassal tract is pushed to one side
ITEM C The selling moves upward on protrusion of the tongue as well as on
swallowing
ITEM D It should be excised because infection is inevitable
ITEM E All of the above

.
Correct Answer: E

Domain: Recall Interpretation Problem Solving

Reference(s): Bailey & Love’s page# 708

PREPPARED BY: College: Frontier Medical & Dental College, Abbottabad

Name: Prof. Dr. Gul Muhammad

Signature:

Date: 22.6.2013

Note:
Diagrams/Charts/Graphs if any should attach along this.
BAHRIA UNIVERSITY ISLAMABAD

MULTIPLE CHOICE QUESTIONS


PROGRAM MBBS
General Surgery
SUBJECT
'PROF Final Professional
TOS _ID 17.5.1

Description Thyroid gland & thyroglassal tract


STEM:
Which of the following statements regarding cretinism is untrue
ITEM A In endemic area goitrous cretinism is common
ITEM B Is due to maternal and foetal iodine deficiency
ITEM C The parents and other children may be normal
ITEM D Hypothyroidism occurs in 1 in 4000 live births
ITEM E Routine biochemical screening of neonates for hypothyroidism is done

. Correct Answer: E

Domain: Recall Interpretation Problem Solving

Reference(s): Bailey & Love’s page#710

PREPPARED BY: College: Frontier Medical & Dental College, Abbottabad

Name: Prof. Dr. Gul Muhammad

Signature:

Date: 22.6.2013

Note:
Diagrams/Charts/Graphs if any should attach along this.
BAHRIA UNIVERSITY ISLAMABAD

MULTIPLE CHOICE QUESTIONS


PROGRAM MBBS
General Surgery
SUBJECT
'PROF Final Professional
TOS _ID 17.5.1

Description Thyroid gland & thyroglassal tract


STEM:
All of the following are symptoms of adult hypothyroidism. Mark the wrong
statement
ITEM A Tiredness
ITEM B Cold intolerance
ITEM C Weight loss
ITEM D Mental lethargy
ITEM E Menstrual disturbances

. Correct Answer: C

Domain: Recall Interpretation Problem Solving

Reference(s): Bailey & Love’s page# 711

PREPPARED BY: College: Frontier Medical & Dental College, Abbottabad

Name: Prof. Dr. Gul Muhammad

Signature:

Date: 22.6.2013

Note:
Diagrams/Charts/Graphs if any should attach along this.
BAHRIA UNIVERSITY ISLAMABAD

MULTIPLE CHOICE QUESTIONS


PROGRAM MBBS
General Surgery
SUBJECT
'PROF Final Professional
TOS _ID 17.5.1

Description Thyroid gland & thyroglassal tract


STEM:
Simple goiter commonly develops as a result of
ITEM A Stimulation of thyroid gland by TSH—as a result of inappropriate secretion
from a microadenoma in anterior pituitary
ITEM B Stimulation of thyroid gland by TSH to chronically low level of circulating
thyroid hormone
ITEM C The most important factor is dietary deficiency of iodine
ITEM D Defective hormone synthesis probably accounts for many sporadic goiter
ITEM E Well known goitrogens like the vegetables of the brassica family

.
Correct Answer: C

Domain: Recall Interpretation Problem Solving

Reference(s): Bailey & Love’s page#712

PREPPARED BY: College: Frontier Medical & Dental College, Abbottabad

Name: Prof. Dr. Gul Muhammad

Signature:

Date: 22.6.2013

Note:
Diagrams/Charts/Graphs if any should attach along this.
BAHRIA UNIVERSITY ISLAMABAD

MULTIPLE CHOICE QUESTIONS


PROGRAM MBBS
General Surgery
SUBJECT
'PROF Final Professional
TOS _ID 17.5.1

Description Thyroid gland & thyroglassal tract


STEM:
All of the following statements regarding de-Quervain’s thyroiditis is true.
Mark the wrong statement
ITEM A This is due to virus infection
ITEM B In a typical subacute presentation there is pain in the neck, fever, malaise and a
firm irregular enlargement of thyroid
ITEM C ESR is raided
ITEM D Thyroid antibodies are absent
ITEM E 1123 uptake is high

.
Correct Answer: E

Domain: Recall Interpretation Problem Solving

Reference(s): Bailey & Love’s page#733

PREPPARED BY: College: Frontier Medical & Dental College, Abbottabad

Name: Prof. Dr. Gul Muhammad

Signature:

Date: 22.6.2013

Note:
Diagrams/Charts/Graphs if any should attach along this.
BAHRIA UNIVERSITY ISLAMABAD

MULTIPLE CHOICE QUESTIONS


PROGRAM MBBS
General Surgery
SUBJECT
'PROF Final Professional
TOS _ID 17.5.2

Description Parathyroid and Adrenals


STEM:
Which of the following statements regarding para-thormone is untrue
ITEM A Stimulates osteoclastic activity
ITEM B Is an 90-amno acid peptide
ITEM C Increases the reabsorption of calcium by the renal tubule
ITEM D Augments the absorption of calcium from the gut
ITEM E Reduces the renal tubular reabsorption of phosphate, thus promoting
phosphaturia

.
Correct Answer: B

Domain: Recall Interpretation Problem Solving

Reference(s): Bailey & Love’s page#734

PREPPARED BY: College: Frontier Medical & Dental College, Abbottabad

Name: Prof. Dr. Gul Muhammad

Signature:

Date: 22.6.2013

Note:
Diagrams/Charts/Graphs if any should attach along this.
BAHRIA UNIVERSITY ISLAMABAD

MULTIPLE CHOICE QUESTIONS


PROGRAM MBBS
General Surgery
SUBJECT
'PROF Final Professional
TOS _ID 17.5.2

Description Parathyroid and Adrenals


STEM:
All of the statements regarding parthyroid tetany are true. Mark the wrong
statements
ITEM A Parathyroid tetany due to hypocalcaemia is a common complications of total
thyroidectomy
ITEM B May also occur after surgery to parathyroids
ITEM C Symptoms usually appear on the 2nd or 3rd post-operative day
ITEM D Tetany in newborn may occur within the first grew days of life if mother has
ITEM E undiagnosed hypoparathyroidism
Permanent hypoparathyroidism is uncommon following radical thyroidectomy
for cancer

.
Correct Answer: A

Domain: Recall Interpretation Problem Solving

Reference(s): Bailey & Love’s page# 735

PREPPARED BY: College: Frontier Medical & Dental College, Abbottabad

Name: Prof. Dr. Gul Muhammad

Signature:

Date: 22.6.2013

Note:
Diagrams/Charts/Graphs if any should attach along this.
BAHRIA UNIVERSITY ISLAMABAD

MULTIPLE CHOICE QUESTIONS


PROGRAM MBBS
General Surgery
SUBJECT
'PROF Final Professional
TOS _ID 17.5.2

Description Parathyroid and Adrenals


STEM:
The characteristic features of Water House-Friderichsen Syndrome are the
following. Mark the wrong statement
ITEM A Massive bilateral adrenal cortical haemorrhage occurs due to fulminating
meningococcal septicemia
ITEM B Most cases occur in infant and young children
ITEM C The onset is catastrophic with rigors, hyperpyrexia, cyanosis and vomiting
ITEM D Death occurs within 48 hours of the onset of symptoms unless correct
treatment is given without delay
ITEM E Pituitary apoplexy may be associated finding

Correct Answer: E

Domain: Recall Interpretation Problem Solving

Reference(s): Bailey & Love’s page#740

PREPPARED BY: College: Frontier Medical & Dental College, Abbottabad

Name: Prof. Dr. Gul Muhammad

Signature:

Date: 22.6.2013

Note:
Diagrams/Charts/Graphs if any should attach along this.
BAHRIA UNIVERSITY ISLAMABAD

MULTIPLE CHOICE QUESTIONS


PROGRAM MBBS
General Surgery
SUBJECT
'PROF Final Professional
TOS _ID 17.5.2

Description Parathyroid and Adrenals


STEM:
All are true of neuroblastoma. Mark the wrong statement
ITEM A Over 50% occur in children below 2 years
ITEM B 50% arise from adrenal gland
ITEM C 50% are malignant
ITEM D 50% show stippled calcification
ITEM E

. Correct Answer: C

Domain: Recall Interpretation Problem Solving

Reference(s): Bailey & Love’s page# 746

PREPPARED BY: College: Frontier Medical & Dental College, Abbottabad

Name: Prof. Dr. Gul Muhammad

Signature:

Date: 22.6.2013

Note:
Diagrams/Charts/Graphs if any should attach along this.
BAHRIA UNIVERSITY ISLAMABAD

MULTIPLE CHOICE QUESTIONS


PROGRAM MBBS
General Surgery
SUBJECT
'PROF Final Professional
TOS _ID 17.7

Description Breast
STEM:
The acronym QUART stands for
ITEM A Quadrantectomy and radiotherapy
ITEM B Quadrantectomy, axillary dissection and radiotherapy
ITEM C Quadrantic resection and chemotherapy
ITEM D None of above
ITEM E

. Correct Answer: B

Domain: Recall Interpretation Problem Solving

Reference(s): Bailey & Love’s page#767

PREPPARED BY: College: Frontier Medical & Dental College, Abbottabad

Name: Prof. Dr. Gul Muhammad

Signature:

Date: 22.6.2013

Note:
Diagrams/Charts/Graphs if any should attach along this.
BAHRIA UNIVERSITY ISLAMABAD

MULTIPLE CHOICE QUESTIONS


PROGRAM MBBS
General Surgery
SUBJECT
'PROF Final Professional
TOS _ID 17.7

Description Breast
STEM:
Indications for chemotherapy in breast cancer include
ITEM A Premenopausal women with positive lymph nodes
ITEM B Postmenopausal women with poor prognosis disease
ITEM C Extensive disease irresptive pre or post menopausal
ITEM D A + C
ITEM E A + B

. Correct Answer: E

Domain: Recall Interpretation Problem Solving

Reference(s): Bailey & Love’s page#768

PREPPARED BY: College: Frontier Medical & Dental College, Abbottabad

Name: Prof. Dr. Gul Muhammad

Signature:

Date: 22.6.2013

Note:
Diagrams/Charts/Graphs if any should attach along this.
BAHRIA UNIVERSITY ISLAMABAD

MULTIPLE CHOICE QUESTIONS


PROGRAM MBBS
General Surgery
SUBJECT
'PROF Final Professional
TOS _ID 17.6

Description Thorax
STEM:
The vertebral region most vulnerable for fracture is
ITEM A Cervical
ITEM B Upper thoracic
ITEM C Lower thoracic
ITEM D Lumbar
ITEM E Sacral

. Correct Answer: A

Domain: Recall Interpretation Problem Solving

Reference(s): Bailey & Love’s page#774

PREPPARED BY: College: Frontier Medical & Dental College, Abbottabad

Name: Prof. Dr. Gul Muhammad

Signature:

Date: 22.6.2013

Note:
Diagrams/Charts/Graphs if any should attach along this.
BAHRIA UNIVERSITY ISLAMABAD

MULTIPLE CHOICE QUESTIONS


PROGRAM MBBS
General Surgery
SUBJECT
'PROF Final Professional
TOS _ID 17.6

Description Thorax
STEM:
A haemothorax should always be drained because
ITEM A Empyema may supervene
ITEM B Fibrothorxa may occur
ITEM C Relieves compression of contralateral lung
ITEM D Helps to reduce bleeding from torn vessels
ITEM E All of the above

. Correct Answer: E

Domain: Recall Interpretation Problem Solving

Reference(s): Bailey & Love’s page#775

PREPPARED BY: College: Frontier Medical & Dental College, Abbottabad

Name: Prof. Dr. Gul Muhammad

Signature:

Date: 22.6.2013

Note:
Diagrams/Charts/Graphs if any should attach along this.
BAHRIA UNIVERSITY ISLAMABAD

MULTIPLE CHOICE QUESTIONS


PROGRAM MBBS
General Surgery
SUBJECT
'PROF Final Professional
TOS _ID 17.6

Description Thorax
STEM:
Transection of aorta should be suspected in presence of
ITEM A Deceleration injury
ITEM B Radiofemoral delay
ITEM C Hoarseness of voice
ITEM D Left sided pleural effusion
ITEM E All of the above

. Correct Answer: E

Domain: Recall Interpretation Problem Solving

Reference(s): Bailey & Love’s page#777

PREPPARED BY: College: Frontier Medical & Dental College, Abbottabad

Name: Prof. Dr. Gul Muhammad

Signature:

Date: 22.6.2013

Note:
Diagrams/Charts/Graphs if any should attach along this.
BAHRIA UNIVERSITY ISLAMABAD

MULTIPLE CHOICE QUESTIONS


PROGRAM MBBS
General Surgery
SUBJECT
'PROF Final Professional
TOS _ID 17.6

Description
STEM:
Clinical presentation of mediastinal mass lesion is with
ITEM A SVC obstruction and facial suffusion
ITEM B Dry cough
ITEM C Hoarseness of voice
ITEM D Elevated hemidiaphragm
ITEM E All of the above

. Correct Answer: E

Domain: Recall Interpretation Problem Solving

Reference(s): Bailey & Love’s page#792

PREPPARED BY: College: Frontier Medical & Dental College, Abbottabad

Name: Prof. Dr. Gul Muhammad

Signature:

Date: 22.6.2013

Note:
Diagrams/Charts/Graphs if any should attach along this.
BAHRIA UNIVERSITY ISLAMABAD

MULTIPLE CHOICE QUESTIONS


PROGRAM MBBS
General Surgery
SUBJECT
'PROF Final Professional
TOS _ID 17.6

Description Thorax
STEM:
Sudden right sided chest pain in post-surgical patient with dyspnoea and loud
P2 indicates
ITEM A Myocardial infarction
ITEM B Plumonary embolism
ITEM C Acute pneumothorax
ITEM D Congestive cardiac failue
ITEM E

. Correct Answer: B

Domain: Recall Interpretation Problem Solving

Reference(s): Bailey & Love’s page#799

PREPPARED BY: College: Frontier Medical & Dental College, Abbottabad

Name: Prof. Dr. Gul Muhammad

Signature:

Date: 22.6.2013

Note:
Diagrams/Charts/Graphs if any should attach along this.
BAHRIA UNIVERSITY ISLAMABAD

MULTIPLE CHOICE QUESTIONS


PROGRAM MBBS
General Surgery
SUBJECT
'PROF Final Professional
TOS _ID 17.9.1

Description Heart
STEM:
Chronic constrictive pericarditis presents with all. Mark the wrong statement
ITEM A Raised CVP
ITEM B Tender hepatomegaly
ITEM C Ascites and oedema feet
ITEM D Marked cardiomegaly
ITEM E

. Correct Answer: D

Domain: Recall Interpretation Problem Solving

Reference(s): Bailey & Love’s page#831

PREPPARED BY: College: Frontier Medical & Dental College, Abbottabad

Name: Prof. Dr. Gul Muhammad

Signature:

Date: 22.6.2013

Note:
Diagrams/Charts/Graphs if any should attach along this.
BAHRIA UNIVERSITY ISLAMABAD

MULTIPLE CHOICE QUESTIONS


PROGRAM MBBS
General Surgery
SUBJECT
'PROF Final Professional
TOS _ID 17.2.1

Description Oesophagus
STEM:
What is not true of Boerhaave’s syndrome
ITEM A Occurs due to vomiting with full stomach
ITEM B There is longitudinal tear in lower oesophagus
ITEM C Tear involves the mucosa and submucosa
ITEM D Mediatinitis and left pleural effusion can occur
ITEM E

. Correct Answer: C

Domain: Recall Interpretation Problem Solving

Reference(s): Baiely & Love’s page#859

PREPPARED BY: College: Frontier Medical & Dental College, Abbottabad

Name: Prof. Dr. Gul Muhammad

Signature:

Date: 22.6.2013

Note:
Diagrams/Charts/Graphs if any should attach along this.
BAHRIA UNIVERSITY ISLAMABAD

MULTIPLE CHOICE QUESTIONS


PROGRAM MBBS
General Surgery
SUBJECT
'PROF Final Professional
TOS _ID 17.2.1

Description Oesophagus
STEM:
In Mallory-Weiss syndrome all are true. Mark the wrong statement
ITEM A Follows severe vomiting
ITEM B Vertical tear
ITEM C Lower oesophageal tear
ITEM D Mild bleeding
ITEM E

. Correct Answer: C

Domain: Recall Interpretation Problem Solving

Reference(s): Bailey & Love’s page#862

PREPPARED BY: College: Frontier Medical & Dental College, Abbottabad

Name: Prof. Dr. Gul Muhammad

Signature:

Date: 22.6.2013

Note:
Diagrams/Charts/Graphs if any should attach along this.
BAHRIA UNIVERSITY ISLAMABAD

MULTIPLE CHOICE QUESTIONS


PROGRAM MBBS
General Surgery
SUBJECT
'PROF Final Professional
TOS _ID 17.2.2

Description Stomach and Duodenum


STEM:
In hypertrophic pyloric stenosis of adult the pathology is
ITEM A Overgrowth of circular muscle
ITEM B Overgrowth of longitudinal muscle
ITEM C Fibrosis of myenteric plexus
ITEM D None of the above
ITEM E

. Correct Answer: C

Domain: Recall Interpretation Problem Solving

Reference(s): Bailey & Love’s Page#899

PREPPARED BY: College: Frontier Medical & Dental College, Abbottabad

Name: Prof. Dr. Gul Muhammad

Signature:

Date: 22.6.2013

Note:
Diagrams/Charts/Graphs if any should attach along this.
BAHRIA UNIVERSITY ISLAMABAD

MULTIPLE CHOICE QUESTIONS


PROGRAM MBBS
General Surgery
SUBJECT
'PROF Final Professional
TOS _ID 17.2.2

Description Stomach and Duodenum


STEM:
Ulcers at which location have greater chance of malignancy
ITEM A Antrum
ITEM B Prepyloric region
ITEM C Fundus
ITEM D Body
ITEM E

. Correct Answer: B

Domain: Recall Interpretation Problem Solving

Reference(s): Bailey & Love’s page#905

PREPPARED BY: College: Frontier Medical & Dental College, Abbottabad

Name: Prof. Dr. Gul Muhammad

Signature:

Date: 22.6.2013

Note:
Diagrams/Charts/Graphs if any should attach along this.
BAHRIA UNIVERSITY ISLAMABAD

MULTIPLE CHOICE QUESTIONS


PROGRAM MBBS
General Surgery
SUBJECT
'PROF Final Professional
TOS _ID 17.2.2

Description Stomach and Duodenum


STEM:
Which peptic ulcer complication manifests with vomiting, abdominal pain and
rigidity
ITEM A Pyloric obstruction
ITEM B Perforation
ITEM C Malignant transformation
ITEM D Haemorrhage
ITEM E

. Correct Answer: B

Domain: Recall Interpretation Problem Solving

Reference(s): Bailey & Love’s page#911

PREPPARED BY: College: Frontier Medical & Dental College, Abbottabad

Name: Prof. Dr. Gul Muhammad

Signature:

Date: 22.6.2013

Note:
Diagrams/Charts/Graphs if any should attach along this.
BAHRIA UNIVERSITY ISLAMABAD

MULTIPLE CHOICE QUESTIONS


PROGRAM MBBS
General Surgery
SUBJECT
'PROF Final Professional
TOS _ID 17.2.3

Description Liver
STEM:
Variceal bleed secondary to portal vein thrombosis is best treated by
ITEM A TIPSS
ITEM B Sclerotherapy
ITEM C Gastroesophageral devascularisation
ITEM D Octreotide
ITEM E

. Correct Answer: C

Domain: Recall Interpretation Problem Solving

Reference(s): Bailey & Love’s page# 942

PREPPARED BY: College: Frontier Medical & Dental College, Abbottabad

Name: Prof. Dr. Gul Muhammad

Signature:

Date: 22.6.2013

Note:
Diagrams/Charts/Graphs if any should attach along this.
BAHRIA UNIVERSITY ISLAMABAD

MULTIPLE CHOICE QUESTIONS


PROGRAM MBBS
General Surgery
SUBJECT
'PROF Final Professional
TOS _ID 17.2.3

Description Liver
STEM:
Primary sclerosing cholangitis is associated with
ITEM A Rheumatoid arthritis
ITEM B Ulcerative colitis
ITEM C Mixed connective tissue disease
ITEM D All of the above
ITEM E

. Correct Answer: B

Domain: Recall Interpretation Problem Solving

Reference(s): Bailey & Love’s page#944

PREPPARED BY: College: Frontier Medical & Dental College, Abbottabad

Name: Prof. Dr. Gul Muhammad

Signature:

Date: 22.6.2013

Note:
Diagrams/Charts/Graphs if any should attach along this.
BAHRIA UNIVERSITY ISLAMABAD

MULTIPLE CHOICE QUESTIONS


PROGRAM MBBS
General Surgery
SUBJECT
'PROF Final Professional
TOS _ID 17.2.3

Description Liver
STEM:
Intrahepatic biliary lakes with stone characterize
ITEM A Hepatic polycystic disease
ITEM B Primatry biliary cirrhosis
ITEM C Caroli’s disease
ITEM D Cholangitis
ITEM E

. Correct Answer: C

Domain: Recall Interpretation Problem Solving

Reference(s): Bailey & Love’s page # 944

PREPPARED BY: College: Frontier Medical & Dental College, Abbottabad

Name: Prof. Dr. Gul Muhammad

Signature:

Date: 22.6.2013

Note:
Diagrams/Charts/Graphs if any should attach along this.
BAHRIA UNIVERSITY ISLAMABAD

MULTIPLE CHOICE QUESTIONS


PROGRAM MBBS
General Surgery
SUBJECT
'PROF Final Professional
TOS _ID 17.2.3

Description Liver
STEM:
The most common complication of hepatic hydrated disease is
ITEM A Jaundice
ITEM B Rupture into peritoneal cavity
ITEM C Suppuration
ITEM D Rupture into biliary channel
ITEM E

. Correct Answer: D

Domain: Recall Interpretation Problem Solving

Reference(s): Bailey & Love’s Page# 946

PREPPARED BY: College: Frontier Medical & Dental College, Abbottabad

Name: Prof. Dr. Gul Muhammad

Signature:

Date: 22.6.2013

Note:
Diagrams/Charts/Graphs if any should attach along this.
BAHRIA UNIVERSITY ISLAMABAD

MULTIPLE CHOICE QUESTIONS


PROGRAM MBBS
General Surgery
SUBJECT
'PROF Final Professional
TOS _ID 17.2.4

Description Spleen
STEM:
Kehr sign in splenic trauma refers to
ITEM A Pain and hyperaesthesia in leftr shoulder
ITEM B Pain and hyperaesthesia in right shoulder
ITEM C Bruising around left 10th and 11th ribs
ITEM D Hiccupand haemoptysis on leg elevation
ITEM E

. Correct Answer: A

Domain: Recall Interpretation Problem Solving

Reference(s): Bailey & Love’s page#955

PREPPARED BY: College: Frontier Medical & Dental College, Abbottabad

Name: Prof. Dr. Gul Muhammad

Signature:

Date: 22.6.2013

Note:
Diagrams/Charts/Graphs if any should attach along this.
BAHRIA UNIVERSITY ISLAMABAD

MULTIPLE CHOICE QUESTIONS


PROGRAM MBBS
General Surgery
SUBJECT
'PROF Final Professional
TOS _ID 17.2.4

Description Spleen
STEM:
A young child having anemia, and gallstones should be investigated for
ITEM A Cystic fibrosis
ITEM B Congenital spherocytosis
ITEM C Malaria
ITEM D Primary sclerosing cholangitis
ITEM E

. Correct Answer: B

Domain: Recall Interpretation Problem Solving

Reference(s): Bailey & Love’s page# 959

PREPPARED BY: College: Frontier Medical & Dental College, Abbottabad

Name: Prof. Dr. Gul Muhammad

Signature:

Date: 22.6.2013

Note:
Diagrams/Charts/Graphs if any should attach along this.
BAHRIA UNIVERSITY ISLAMABAD

MULTIPLE CHOICE QUESTIONS


PROGRAM MBBS
General Surgery
SUBJECT
'PROF Final Professional
TOS _ID 17.2.4

Description Spleen
STEM:
A 2 year old child with anaemia, hepatosplenomegaly and a large head with
slanting eyes is mots likely to have
ITEM A Cirrhosis
ITEM B Spherocytosis
ITEM C Thalassaemia
ITEM D Gaucher’s disease
ITEM E

. Correct Answer: C

Domain: Recall Interpretation Problem Solving

Reference(s): Bailey & Love’s page# 960

PREPPARED BY: College: Frontier Medical & Dental College, Abbottabad

Name: Prof. Dr. Gul Muhammad

Signature:

Date: 22.6.2013

Note:
Diagrams/Charts/Graphs if any should attach along this.
BAHRIA UNIVERSITY ISLAMABAD

MULTIPLE CHOICE QUESTIONS


PROGRAM MBBS
General Surgery
SUBJECT
'PROF Final Professional
TOS _ID 17.2.5

Description Gall bladder & bile duct


STEM:
The primary investigation of choice in gallstone disease is
ITEM A Plain x-Ray
ITEM B OCG
ITEM C Ultrasonography
ITEM D IV cholangiography
ITEM E

. Correct Answer: C

Domain: Recall Interpretation Problem Solving

Reference(s): Bailey & Love’s page#968

PREPPARED BY: College: Frontier Medical & Dental College, Abbottabad

Name: Prof. Dr. Gul Muhammad

Signature:

Date: 22.6.2013

Note:
Diagrams/Charts/Graphs if any should attach along this.
BAHRIA UNIVERSITY ISLAMABAD

MULTIPLE CHOICE QUESTIONS


PROGRAM MBBS
General Surgery
SUBJECT
'PROF Final Professional
TOS _ID 17.2.5

Description Gall bladder & bile duct


STEM:
Which of the following is a possibility if symptoms persist after
cholecystectomy
ITEM A Stone in CBD
ITEM B Long stump of cystic duct remnant
ITEM C Damage to CBD from consequent stricture
ITEM D Each of above
ITEM E

. Correct Answer: D

Domain: Recall Interpretation Problem Solving

Reference(s): Bailey & Love’s page# 981

PREPPARED BY: College: Frontier Medical & Dental College, Abbottabad

Name: Prof. Dr. Gul Muhammad

Signature:

Date: 22.6.2013

Note:
Diagrams/Charts/Graphs if any should attach along this.
BAHRIA UNIVERSITY ISLAMABAD

MULTIPLE CHOICE QUESTIONS


PROGRAM MBBS
General Surgery
SUBJECT
'PROF Final Professional
TOS _ID 17.2.5

Description Gall bladder & bile duct


STEM:
Gallbladder pain persisting for more than 12 hours is due to
ITEM A Stone colic
ITEM B Cholecystitis
ITEM C Perforation of gallbladder
ITEM D Torsion of gallbladder
ITEM E

. Correct Answer: B

Domain: Recall Interpretation Problem Solving

Reference(s): Bailey & Love’s page # 748

PREPPARED BY: College: Frontier Medical & Dental College, Abbottabad

Name: Prof. Dr. Gul Muhammad

Signature:

Date: 22.6.2013

Note:
Diagrams/Charts/Graphs if any should attach along this.
BAHRIA UNIVERSITY ISLAMABAD

MULTIPLE CHOICE QUESTIONS


PROGRAM MBBS
General Surgery
SUBJECT
'PROF Final Professional
TOS _ID 17.2.6

Description Pancreas
STEM:
Clinical manifestations of cystic fibrosis include all. Mark the wrong statement
ITEM A Malabsorption
ITEM B Obstructive lung disease
ITEM C Salty child
ITEM D Mental retardation
ITEM E

. Correct Answer: D

Domain: Recall Interpretation Problem Solving

Reference(s): Bailey & Love’s page # 994

PREPPARED BY: College: Frontier Medical & Dental College, Abbottabad

Name: Prof. Dr. Gul Muhammad

Signature:

Date: 22.6.2013

Note:
Diagrams/Charts/Graphs if any should attach along this.
BAHRIA UNIVERSITY ISLAMABAD

MULTIPLE CHOICE QUESTIONS


PROGRAM MBBS
General Surgery
SUBJECT
'PROF Final Professional
TOS _ID 17.2.6

Description Pancreas
STEM:
The most common feature of acute pancreatitis is
ITEM A Severe acute epigastric pain radiating to back
ITEM B Jaundice
ITEM C Cullen’s sign and Grey Turner’s sign
ITEM D Abdominal guarding and loss of bowel sounds
ITEM E

. Correct Answer: A

Domain: Recall Interpretation Problem Solving

Reference(s): Bailey & Love’s Page # 997

PREPPARED BY: College: Frontier Medical & Dental College, Abbottabad

Name: Prof. Dr. Gul Muhammad

Signature:

Date: 22.6.2013

Note:
Diagrams/Charts/Graphs if any should attach along this.
BAHRIA UNIVERSITY ISLAMABAD

MULTIPLE CHOICE QUESTIONS


PROGRAM MBBS
General Surgery
SUBJECT
'PROF Final Professional
TOS _ID 17.2.7

Description The Peritoneum, Omentum and Mesentery


STEM:
Hippocratic facies are seen in
ITEM A Acute cholecystitis
ITEM B Acute appendicitis
ITEM C Diffuse peritonitis
ITEM D Acute salpingitis
ITEM E

. Correct Answer: C

Domain: Recall Interpretation Problem Solving

Reference(s): Bailey & Love’s page#1009

PREPPARED BY: College: Frontier Medical & Dental College, Abbottabad

Name: Prof. Dr. Gul Muhammad

Signature:

Date: 22.6.2013

Note:
Diagrams/Charts/Graphs if any should attach along this.
BAHRIA UNIVERSITY ISLAMABAD

MULTIPLE CHOICE QUESTIONS


PROGRAM MBBS
General Surgery
SUBJECT
'PROF Final Professional
TOS _ID 17.2.7

Description The Peritoneum, Omentum and Mesentery


STEM:
Abdominal complications of peritonitis include all. Mark the wrong statement
ITEM A Paralytic ileus
ITEM B Portal pyemia
ITEM C Small bowel obstruction
ITEM D Colonic volvulus
ITEM E

. Correct Answer: D

Domain: Recall Interpretation Problem Solving

Reference(s): Bailey & Love’s page# 1011

PREPPARED BY: College: Frontier Medical & Dental College, Abbottabad

Name: Prof. Dr. Gul Muhammad

Signature:

Date: 22.6.2013

Note:
Diagrams/Charts/Graphs if any should attach along this.
BAHRIA UNIVERSITY ISLAMABAD

MULTIPLE CHOICE QUESTIONS


PROGRAM MBBS
General Surgery
SUBJECT
'PROF Final Professional
TOS _ID 17.3.1

Description Small & Large intestine


STEM:
Clinical presentation of Hirschsprung’s disease includes all. Mark the wrong
statement
ITEM A Delayed passage of meconium in newborn
ITEM B Neonatal intestinal obstruction
ITEM C Severe chronic constipation
ITEM D Common to female child and no familial tendency
ITEM E Common to male child and no familial tendency

. Correct Answer: D

Domain: Recall Interpretation Problem Solving

Reference(s): Bailey & Love’s page# 1027

PREPPARED BY: College: Frontier Medical & Dental College, Abbottabad

Name: Prof. Dr. Gul Muhammad

Signature:

Date: 22.6.2013

Note:
Diagrams/Charts/Graphs if any should attach along this.
BAHRIA UNIVERSITY ISLAMABAD

MULTIPLE CHOICE QUESTIONS


PROGRAM MBBS
General Surgery
SUBJECT
'PROF Final Professional
TOS _ID 17.3.1

Description Small & Large intestine


STEM:
The rule of 2 as applicable to Meckel’s diverticulum is all. Mark the wrong
statement
ITEM A 2” long
ITEM B Present in 2% cases
ITEM C 2′ from ileocaecal valve
ITEM D Contains only 2 coats of intestinal wall
ITEM E

. Correct Answer: D

Domain: Recall Interpretation Problem Solving

Reference(s): Bailey & Love’s page# 1032

PREPPARED BY: College: Frontier Medical & Dental College, Abbottabad

Name: Prof. Dr. Gul Muhammad

Signature:

Date: 22.6.2013

Note:
Diagrams/Charts/Graphs if any should attach along this.
BAHRIA UNIVERSITY ISLAMABAD

MULTIPLE CHOICE QUESTIONS


PROGRAM MBBS
General Surgery
SUBJECT
'PROF Final Professional
TOS _ID 17.3.1

Description Small & Large intestine


STEM:
The most frequent symptom of Meckel’s diverticulum is
ITEM A Diverticulitis
ITEM B Bleeding
ITEM C Intussusception
ITEM D Intestinal obstruction
ITEM E

. Correct Answer: B

Domain: Recall Interpretation Problem Solving

Reference(s): Bailey & Love’s page# 1034

PREPPARED BY: College: Frontier Medical & Dental College, Abbottabad

Name: Prof. Dr. Gul Muhammad

Signature:

Date: 22.6.2013

Note:
Diagrams/Charts/Graphs if any should attach along this.
BAHRIA UNIVERSITY ISLAMABAD

MULTIPLE CHOICE QUESTIONS


PROGRAM MBBS
General Surgery
SUBJECT
'PROF Final Professional
TOS _ID 17.3.1

Description Small & Large intestine


STEM:
Intestinal amoebiasis can manifest with which of the following
ITEM A Typhlitis
ITEM B Caecal mass
ITEM C Intestinal obstruction
ITEM D Fibrous stricture
ITEM E All of the above

. Correct Answer: E

Domain: Recall Interpretation Problem Solving

Reference(s): Bailey & Love’s page# 1045

PREPPARED BY: College: Frontier Medical & Dental College, Abbottabad

Name: Prof. Dr. Gul Muhammad

Signature:

Date: 22.6.2013

Note:
Diagrams/Charts/Graphs if any should attach along this.
BAHRIA UNIVERSITY ISLAMABAD

MULTIPLE CHOICE QUESTIONS


PROGRAM MBBS
General Surgery
SUBJECT
'PROF Final Professional
TOS _ID 17.3.1

Description Small & Large intestine


STEM:
Juvenile polyp belongs to which of the following
ITEM A Hamartoma
ITEM B Villlous adenoma
ITEM C Tubulovillous adenoma
ITEM D Tubular adenoma
ITEM E

. Correct Answer: A

Domain: Recall Interpretation Problem Solving

Reference(s): Bailey & Love’s page# 1048

PREPPARED BY: College: Frontier Medical & Dental College, Abbottabad

Name: Prof. Dr. Gul Muhammad

Signature:

Date: 22.6.2013

Note:
Diagrams/Charts/Graphs if any should attach along this.
BAHRIA UNIVERSITY ISLAMABAD

MULTIPLE CHOICE QUESTIONS


PROGRAM MBBS
General Surgery
SUBJECT
'PROF Final Professional
TOS _ID 17.3.1

Description Small & Large intestine


STEM:
When carcinoma is located in pelvic colon the area to be resected is

ITEM A Entire colon


ITEM B Transverse colon to rectum
ITEM C Splenic flexure to rectum
ITEM D Hepatic flexue to rectum
ITEM E

. Correct Answer: C

Domain: Recall Interpretation Problem Solving

Reference(s): Bailey & Love’s page# 1052

PREPPARED BY: College: Frontier Medical & Dental College, Abbottabad

Name: Prof. Dr. Gul Muhammad

Signature:

Date: 22.6.2013

Note:
Diagrams/Charts/Graphs if any should attach along this.
BAHRIA UNIVERSITY ISLAMABAD

MULTIPLE CHOICE QUESTIONS


PROGRAM MBBS
General Surgery
SUBJECT
'PROF Final Professional
TOS _ID 17.3.3

Description Vermiform Appendix


STEM:
What is not true of microsopic antomay of appendix
ITEM A Base of the crypts contain Kultschitzsky cells
ITEM B Crypts are numerous
ITEM C Submucosa contains “abdominal tonsils”
ITEM D The longitudinal muscle coat is due to joining of taenia coli at base of
appendix
ITEM E

.
Correct Answer: B

Domain: Recall Interpretation Problem Solving

Reference(s): Bailey & Love’s page# 1077

PREPPARED BY: College: Frontier Medical & Dental College, Abbottabad

Name: Prof. Dr. Gul Muhammad

Signature:

Date: 22.6.2013

Note:
Diagrams/Charts/Graphs if any should attach along this.
BAHRIA UNIVERSITY ISLAMABAD

MULTIPLE CHOICE QUESTIONS


PROGRAM MBBS
General Surgery
SUBJECT
'PROF Final Professional
TOS _ID 17.3.3

Description Vermiform Appendix


STEM:
In elderly appendicitis need be differentiated from
ITEM A Mesenteric infarction
ITEM B Colon cancer
ITEM C Torsion appendix epiploicoe
ITEM D Diverticulitis
ITEM E Each of the above

. Correct Answer: E

Domain: Recall Interpretation Problem Solving

Reference(s): Bailey & Love’s page# 1082

PREPPARED BY: College: Frontier Medical & Dental College, Abbottabad

Name: Prof. Dr. Gul Muhammad

Signature:

Date: 22.6.2013

Note:
Diagrams/Charts/Graphs if any should attach along this.
BAHRIA UNIVERSITY ISLAMABAD

MULTIPLE CHOICE QUESTIONS


PROGRAM MBBS
General Surgery
SUBJECT
'PROF Final Professional
TOS _ID 17.3.4

Description Rectum
STEM:
Rectal lesions mimicking rectal cancer include
ITEM A Inflammatory stricture
ITEM B Amoeboma
ITEM C Endometrioma
ITEM D Solitary rectal ulcer
ITEM E All of the above

. Correct Answer: E

Domain: Recall Interpretation Problem Solving

Reference(s): Bailey & Love’s page# 1107

PREPPARED BY: College: Frontier Medical & Dental College, Abbottabad

Name: Prof. Dr. Gul Muhammad

Signature:

Date: 22.6.2013

Note:
Diagrams/Charts/Graphs if any should attach along this.
BAHRIA UNIVERSITY ISLAMABAD

MULTIPLE CHOICE QUESTIONS


PROGRAM MBBS
General Surgery
SUBJECT
'PROF Final Professional
TOS _ID 17.3.4

Description Rectum
STEM:
Rectal cancer can be confused with
ITEM A Benign adenoma
ITEM B Inflammatory stricture
ITEM C Amoebic granuloma
ITEM D Endometrioma
ITEM E Each of the above

. Correct Answer: E

Domain: Recall Interpretation Problem Solving

Reference(s): Bailey & Love’s page# 1107

PREPPARED BY: College: Frontier Medical & Dental College, Abbottabad

Name: Prof. Dr. Gul Muhammad

Signature:

Date: 22.6.2013

Note:
Diagrams/Charts/Graphs if any should attach along this.
BAHRIA UNIVERSITY ISLAMABAD

MULTIPLE CHOICE QUESTIONS


PROGRAM MBBS
General Surgery
SUBJECT
'PROF Final Professional
TOS _ID 17.3.4

Description Rectum
STEM:
What is true of local recurrence after surgical treatment of rectal cancer
ITEM A It is a major problem
ITEM B Mainstay of therapy is radiotherapy
ITEM C Most common cause is inadequate removal than micrometastasis
ITEM D CEA estimation can indicate radical surgery
ITEM E All are true

. Correct Answer: E

Domain: Recall Interpretation Problem Solving

Reference(s): Bailey & Love’s page# 1113

PREPPARED BY: College: Frontier Medical & Dental College, Abbottabad

Name: Prof. Dr. Gul Muhammad

Signature:

Date: 22.6.2013

Note:
Diagrams/Charts/Graphs if any should attach along this.
BAHRIA UNIVERSITY ISLAMABAD

MULTIPLE CHOICE QUESTIONS


PROGRAM MBBS
General Surgery
SUBJECT
'PROF Final Professional
TOS _ID 17.3.5

Description Anus & Anal canal


STEM:
Most common cause of anorectal abscess and fistula is
ITEM A Unhygenic perianal skin
ITEM B Anal intercourse
ITEM C Infection of anal glands
ITEM D Perianal eczema
ITEM E

. Correct Answer: C

Domain: Recall Interpretation Problem Solving

Reference(s): Bailey & Love’s page# 1116

PREPPARED BY: College: Frontier Medical & Dental College, Abbottabad

Name: Prof. Dr. Gul Muhammad

Signature:

Date: 22.6.2013

Note:
Diagrams/Charts/Graphs if any should attach along this.
BAHRIA UNIVERSITY ISLAMABAD

MULTIPLE CHOICE QUESTIONS


PROGRAM MBBS
General Surgery
SUBJECT
'PROF Final Professional
TOS _ID 17.3.5

Description Anus & Anal canal


STEM:
The most common malignant tumour of anus is
ITEM A Basaloid carcinoma
ITEM B Mucoepidermoid carcinoma
ITEM C Melanoma
ITEM D Sequamous cell carcinoma
ITEM E

. Correct Answer: D

Domain: Recall Interpretation Problem Solving

Reference(s): Bailey & Love’s page# 1140

PREPPARED BY: College: Frontier Medical & Dental College, Abbottabad

Name: Prof. Dr. Gul Muhammad

Signature:

Date: 22.6.2013

Note:
Diagrams/Charts/Graphs if any should attach along this.
BAHRIA UNIVERSITY ISLAMABAD

MULTIPLE CHOICE QUESTIONS


PROGRAM MBBS
General Surgery
SUBJECT
'PROF Final Professional
TOS _ID 17.3.6

Description Hernias, Umbilicus and Abdominal Wall


STEM:
Which of the following statements is untrue in obstructed hernia
ITEM A Synonymous with incarcerated hernia
ITEM B This is an irreducible hernia containing intestine which is obstructed
ITEM C There is interference to blood supply
ITEM D The symptoms are less severe than in strangulation
ITEM E Often no clear distinction can be made between obstruction and strangulation

. Correct Answer: C

Domain: Recall Interpretation Problem Solving

Reference(s): bailey & Love’s page# 1145

PREPPARED BY: College: Frontier Medical & Dental College, Abbottabad

Name: Prof. Dr. Gul Muhammad

Signature:

Date: 22.6.2013

Note:
Diagrams/Charts/Graphs if any should attach along this.
BAHRIA UNIVERSITY ISLAMABAD

MULTIPLE CHOICE QUESTIONS


PROGRAM MBBS
General Surgery
SUBJECT
'PROF Final Professional
TOS _ID 17.3.6

Description Hernias, Umbilicus and Abdominal Wall


STEM:
All of the following are the causes of ventral hernia. Mark the wrong statement
ITEM A Occurs most often in obese individuals
ITEM B High incidence following operations for peritonitis
ITEM C The placing of a drainage tube through the incision
ITEM D Persistence of post operative cough and post operative abdominal distntion
ITEM E None of the above

. Correct Answer: E

Domain: Recall Interpretation Problem Solving

Reference(s): Bailey & Love’s page# 1159

PREPPARED BY: College: Frontier Medical & Dental College, Abbottabad

Name: Prof. Dr. Gul Muhammad

Signature:

Date: 22.6.2013

Note:
Diagrams/Charts/Graphs if any should attach along this.
BAHRIA UNIVERSITY ISLAMABAD

MULTIPLE CHOICE QUESTIONS


PROGRAM MBBS
General Surgery
SUBJECT
'PROF Final Professional
TOS _ID 17.4.1

Description Kidneys and Ureter Tract


STEM:
Which of the following statements regarding infantile polycystic disease of
kidney is not true
ITEM A Rare condition
ITEM B The kidneys are large and may cause obstruction at birth
ITEM C Most of the children are stillborn
ITEM D Most others die of renal failure in early life
ITEM E The condition is inherited as autosomal dominant

. Correct Answer: E

Domain: Recall Interpretation Problem Solving

Reference(s): Bailey & Love’s page# 1176

PREPPARED BY: College: Frontier Medical & Dental College, Abbottabad

Name: Prof. Dr. Gul Muhammad

Signature:

Date: 22.6.2013

Note:
Diagrams/Charts/Graphs if any should attach along this.
BAHRIA UNIVERSITY ISLAMABAD

MULTIPLE CHOICE QUESTIONS


PROGRAM MBBS
General Surgery
SUBJECT
'PROF Final Professional
TOS _ID 17.4.1

Description Kidneys and Ureter Tract


STEM:
The following are the causes of pre renal uraemia. Mark the wrong statement
ITEM A Hypovolemia
ITEM B Severe blood loss
ITEM C GM-negative sepsis
ITEM D Cardiogenic shock
ITEM E Acute glomerulonephritis

. Correct Answer: E

Domain: Recall Interpretation Problem Solving

Reference(s): Bailey & Love’s page# 1169

PREPPARED BY: College: Frontier Medical & Dental College, Abbottabad

Name: Prof. Dr. Gul Muhammad

Signature:

Date: 22.6.2013

Note:
Diagrams/Charts/Graphs if any should attach along this.
BAHRIA UNIVERSITY ISLAMABAD

MULTIPLE CHOICE QUESTIONS


PROGRAM MBBS
General Surgery
SUBJECT
'PROF Final Professional
TOS _ID 17.4.1

Description Kidneys and Ureter Tract


STEM:
The following are the causes of post-renal uraemia. Mark the wrong statement
ITEM A Renal calculus disease
ITEM B Pelvic malignancy
ITEM C Pelvic surgery
ITEM D Crystalluria
ITEM E Prolonged hypoxia

. Correct Answer: E

Domain: Recall Interpretation Problem Solving

Reference(s): Bailey & Love’s page# 1170

PREPPARED BY: College: Frontier Medical & Dental College, Abbottabad

Name: Prof. Dr. Gul Muhammad

Signature:

Date: 22.6.2013

Note:
Diagrams/Charts/Graphs if any should attach along this.
BAHRIA UNIVERSITY ISLAMABAD

MULTIPLE CHOICE QUESTIONS


PROGRAM MBBS
General Surgery
SUBJECT
'PROF Final Professional
TOS _ID 17.4.2

Description Urinary Bladder


STEM:
What is not true of ectopia vesicae
ITEM A Occurrence is 1 in 50,000 births
ITEM B Absence of anterior abdominal wall and anterior wall of bladder
ITEM C Early rupture of cloacal membrane is responsible
ITEM D Common to male child
ITEM E Common to female child

. Correct Answer: C

Domain: Recall Interpretation Problem Solving

Reference(s): Bailey & Love’s # 1202

PREPPARED BY: College: Frontier Medical & Dental College, Abbottabad

Name: Prof. Dr. Gul Muhammad

Signature:

Date: 22.6.2013

Note:
Diagrams/Charts/Graphs if any should attach along this.
Bahria University

Multiple Choices Question


Program MBBS

Subject Gen surgery

Prof Final year

TOS-ID 17.4.1

Description thyroid

Stem A 45-year-old woman complains to her primary care physician of nervousness,


sweating, tremulousness, and weight loss. The thyroid scan shown here exhibits a
pattern that is most consistent with which of the following disorders?

Item A Hyper secreting adenoma

Item B Graves’ disease

Item C Lateral aberrant thyroid

Item D Papillary carcinoma of thyroid

Item E Medullary carcinoma of thyroid

Correct Answer A

Domain: recall

Preference Bailys and loves 26 edition

Prepared by

College Frontier Medical & Dental College, Abbottabad

Name: prof dr johar ali

Signature:

Date:

Bahria University

Multiple Choices Question


Program MBBS

Subject Gen surgery

Prof Final year

TOS-ID 17.6.2

Description Breast and benign conditions

Stem A 14-year-old black girl has her right breast removed because of a large mass. The
tumor weighs 1400 g and has a bulging, very firm, lobulated surface with a whorl-like
pattern.

Which of the following is the most likely diagnosis?

Item A Cystosarcoma phyllodes

Item B Intraductal carcinoma

Item C Malignant lymphoma

Item D Fibroadenoma

Item E Juvenile hypertrophy

Correct Answer D

Domain: recall

Preference Bailys and loves 26 edition

Prepared by

College Frontier Medical & Dental College, Abbottabad

Name: prof dr johar ali

Signature:

Date:

Bahria University

Multiple Choices Question


Program MBBS

Subject Gen surgery

Prof Final year

TOS-ID 17.6.1

Description Breast and malignant conditions

Stem A 45-year-old woman is found to have suspicious appearing calcifications in the right
breast on a screening mammogram. Stereotactic biopsy of the calcifications shows lobular
carcinoma in situ (LCIS). On examination both breasts are dense without palpable masses.
The neck and bilateral axilla are negative for lymphadenopathy . Which of the following is
the most appropriate management of this patient?
Item A Frequent self-breast examinations and yearly screening mammograms

Item B Chemotherapy

Item C Radiation

Item D Right total mastectomy with sentinel lymph node biopsy

Item E Bilateral modified radical mastectomy

Correct Answer A

Domain: recall

Preference Bailys and loves 26 edition

Prepared by

College Frontier Medical & Dental College, Abbottabad

Name: prof dr johar ali

Signature:

Date

Bahria University

Multiple Choices Question

Program MBBS
Subject Gen surgery

Prof Final year

TOS-ID 17.6.1

Description Breast malignent

Stem A 33-year-old pregnant woman notices a persistent, painless lump in the left breast.
On examination the left breast has a single mobile mass without evidence of skin
changes or lymphadenopathy in the neck or axilla. An ultrasound demonstrates a solid,
1-cm mass in the upper outer quadrant of the breast. A core-needle biopsy shows
invasive ductal carcinoma. The patient is in her first trimester of pregnancy .

Which of the following is the most appropriate management of this patient?

Item A Termination of the pregnancy followed by modified radical mastectomy

Item B Immediate administration of chemotherapy followed by modified radical mastectomy


after delivery of the baby

Item C Administration of radiation in the third trimester followed by modified radical


mastectomy after delivery of the baby

Item D Total mastectomy with sentinel lymph node biopsy

Item E Modified radical mastectomy

Correct Answer E

Domain: problem solving

Preference Bailys and loves 26 edition

Prepared by

College Frontier Medical & Dental College, Abbottabad

Name: prof dr johar alSignature

Bahria University

Multiple Choices Question

Program MBBS
Subject Gen surgery

Prof Final year

TOS-ID 17.6.1

Description Breast malignant diseases

Stem A 40-year-old woman presents with a rash involving the nipple-areola complex for the
last month with associated itching. On physical examination there is crusting and
ulceration of the nipple with surrounding erythema involving the areola and
surrounding skin, no palpable breast masses, and no cervical or axillary
lymphadenopathy, Which of the following is the most appropriate next step in the
management of this patient?

Item A Reexamine the patient in 1 month

Item B Corticosteroid cream to the affected area

Item C Administration of oral antibiotics

Item D Mammogram and biopsy of the affected area

Item E Modified radical mastectomy

Correct Answer D

Domain: interpretation

Preference Bailys and loves 26 edition

Prepared by

College Frontier Medical & Dental College, Abbottabad

Name: prof dr johar ali

Signature:

Date:

Bahria University
Multiple Choices Question
Program MBBS
Subject Gen surgery
Prof FINAL YEAR
TOS-ID 17.4.1

Description THYRIOD
Stem A 58-year-old man presents with tachycardia, fever, confusion, and vomiting. Workup
reveals markedly elevated (triiodothyronine) T 3 and (thyroxine) T4 levels. He is
diagnosed as having a thyroid storm 0R thyroid crisis.
Which of the following is the most appropriate next step in the management of this
patient?

Item A Emergent subtotal thyroidectomy

Item B Emergent total thyroidectomy

Item C Emergent hemodialysis

Item D Administration of fluid, antithyroid drugs, β-blockers, iodine solution, and steroids

Item E Emergent radiation therapy to the neck

Correct Answer D

Domain: PROBLEM SOLVING

Preference Bailys and loves 26 edition

Prepared by

College Frontier Medical & Dental College, Abbottabad

Name:PROF DR JOHAR ALI

Signature:

Bahria University
Multiple Choices Question
Program MBBS
Subject Gen surgery
Prof FINAL YEAR
TOS-ID 17.4

Description thyroid
Stem A 23-year-old woman undergoes total thyroidectomy for carcinoma of the thyroid
gland. On the second postoperative day, she begins to complain of a tingling sensation
in her hands. She appears quite anxious and later complains of muscle cramps.
Which of the following is the most appropriate initial management strategy ?

Item A 10 mL of 10% magnesium sulfate intravenously

Item B Oral vitamin D


Item C 100 μg of thyroxine
Item D Continuous infusion of calcium gluconate
Item E Oral calcium gluconate

Correct Answer

Domain: recall

Preference Bailys and loves 26 edition

Prepared by

College Frontier Medical & Dental College, Abbottabad

Name:PROF DR JOHAR ALI

Signature:

Date:

Bahria University
Multiple Choices Question

Program MBBS
Subject Gen surgery
Prof FINAL YEAR
TOS-ID 17.3.5

Description TESTIS AND SCROTUM


Stem A 19-year-old college student presents with a testicular mass, and after treatment he
returns for regular follow-up visits. Which of the following is the most useful serum
marker for detecting recurrent disease after treatment of nonseminomatous testicular
cancer?
Item A Carcinoembryonic antigen (CEA)
Item B Human chorionic gonadotropin (hCG)

Item C Prostate-specific antigen (PSA)


Item D CA125
Item E p53 oncogene

Correct Answer B

Domain: recall

Preference Bailys and loves 26 edition

Prepared by

College Frontier Medical & Dental College, Abbottabad

Name:PROF DR JOHAR ALI

Signature:

Bahria University
Multiple Choices Question

Program MBBS
Subject Gen surgery
Prof FINAL YEAR
TOS-ID

Description
Stem Incisional biopsy of a breast mass in a 35-year-old woman demonstrates cystosarcoma
phyllodes at the time of frozen section. Which of the following is the most appropriate
management strategy for this lesion?

Item A Wide local excision with a rim of normal tissue

Item B Lumpectomy and axillary lymphadenectomy

Item C Modified radical mastectomy

Item D Excision and postoperative radiotherapy


Item E Excision, postoperative radiotherapy, and systemic chemotherapy

Correct Answer A
Domain: recall

Preference Bailys and loves 26 edition

Prepared by

College Frontier Medical & Dental College, Abbottabad

Name:PROF DR JOHAR ALI

Signature:

Date:

Bahria University
Multiple Choices Question

Program MBBS
Subject Gen surgery
Prof FINAL YEAR
TOS-ID 17.4.1

Description Thyroid
Stem A 36-year-old woman, 20 weeks pregnant, presents with a 1.5-cm right thyroid mass.
FNA is consistent with a papillary neoplasm. The mass is cold on scan and solid on
ultrasound.
Which of the following methods of treatment is contraindicated in this patient?
Item A Right thyroid lobectomy
Item B Subtotal thyroidectomy
Item C Total thyroidectomy
Item D Total thyroidectomy with lymph node dissection
Item E 131I radioactive ablation of the thyroid gland

Correct Answer E
Domain: problem solving

Preference Bailys and loves 26 edition

Prepared by

College Frontier Medical & Dental College, Abbottabad

Name:PROF DR JOHAR ALI

Signature:

Date:

Bahria University
Multiple Choices Question

Program MBBS
Subject Gen surgery
Prof FINAL YEAR
TOS-ID 17.4.1

Description thyroid
Stem A 63-year-old woman notices lumps on both sides of her neck. A fine-needle aspirate is
nondiagnostic, and she undergoes total thyroidectomy. Final pathology reveals a 2-cm
Hürthle cell carcinoma.
Which of the following is the most appropriate postsurgical management of this
patient?
Item A No further therapy is indicated.
Item B Chemotherapy.
Item C External beam radiotherapy .
Item D Radioiodine ablation.
Item E Chemotherapy, external beam radiotherapy, and radioiodine ablation.

Correct Answer D

Domain: PROBLEM SOLVING

Preference Bailys and loves 26 edition

Prepared by

College Frontier Medical & Dental College, Abbottabad

Name:PROF DR JOHAR ALI

Signature:

Date

Bahria University
Multiple Choices Question

Program MBBS
Subject Gen surgery
Prof FINAL YEAR
TOS-ID 17.4.1

Description THYROID
Stem A 51-year-old man presents with a 2-cm left thyroid nodule. Thyroid scan shows a cold
lesion. FNA cytology demonstrates follicular cells.
Which of the following is the most appropriate initial treatment of this patient?

Item A External beam radiation to the neck.


Item B Multidrug chemotherapy.
Item C TSH suppression by thyroid hormone.
Item D Prophylactic neck dissection is indicated along with a total thyroidectomy
Item E Thyroid lobectomy.
Correct Answer E

Domain: PROBLEM SOLVING

Preference Bailys and loves 26 edition

Prepared by

College Frontier Medical & Dental College, Abbottabad

Name:PROF DR JOHAR ALI

Signature:

Date:

Bahria University
Multiple Choices Question

Program MBBS
Subject Gen surgery
Prof Final year
TOS-ID 17.6.1

Description Breast malignant disease


Stem A 36-year-old woman whose mother has just undergone treatment for breast cancer is
asking about how this affects her and what can be done to lessen her chances of
having the disease.
Which of the following has the lowest risk factor for breast cancer?

Item A Dietary fat intake


Item B Paternal relative with breast cancer 1 (BRCA1) mutation
Item C Excessive estrogen exposure—early menarche, late menopause, nulliparity
Item D Previous biopsy with atypical hyperplasia

Item E Exposure to ionizing radiation

Correct Answer A

Domain: INTERPRETATION

Preference Bailys and loves 26 edition

Prepared by

College Frontier Medical & Dental College, Abbottabad

Name: prof dr johar ali

Signature:

Date:

Bahria University
Multiple Choices Question

Program MBBS
Subject Gen surgery
Prof Final year
TOS-ID 17.6.1

Description Breast malignant diseases


Stem A 40 years old lady presented with lump breast, size of Tumor is 4.0 cm; clinically
positive, movable axillary ipsilateral lymph nodes; no evidence of metastases. What is
the stage ?
Item A Stage 0
Item B Stage I
Item C Stage II

Item D Stage III


Item E Stage IV
Correct Answer C

Domain: interpretation

Preference Bailys and loves 26 edition

Prepared by

College Frontier Medical & Dental College, Abbottabad

Name: prof dr johar ali

Signature:

Date:

Bahria University
Multiple Choices Question

Program MBBS
Subject Gen surgery
Prof Final year
TOS-ID 17.2.10

Description Vermiform appendix


Stem 43-year-old man presents with signs and symptoms of peritonitis in the right lower
quadrant. The clinical impression and supportive data suggest acute appendicitis. At
exploration, however, a tumor is found to have carcinoid features. Its 2.5cm in size
located at the base of appendix.
What will be your choice of treatment?
Item A Appendectomy
Item B Segmental ileal resection
Item C Cecectomy
Item D Right hemi colectomy
Item E Hepatic wedge resection and appropriate bowel resection
Correct Answer D

Domain:

Preference Bailys and loves 26 edition

Prepared by

College Frontier Medical & Dental College, Abbottabad

Name: prof dr johar ali

Signature:

Date:

Bahria University
Multiple Choices Question

Program MBBS
Subject Gen surgery
Prof Final year
TOS-ID 17.6.1

Description Breast Malignant disease


Stem A 49-year-old woman has a 6-cm palpable mass that is biopsy -proven ductal
adenocarcinoma. She undergoes neoadjuvant chemotherapy which reduces the
tumor to 3 cm in size. However, she has palpable axillary lymph nodes; FNA
demonstrates adenocarcinoma. She desires breast conservation therapy if possible.
What option will be most appropriate in this case?
Item A Wide local excision
Item B Wide local excision with adjuvant radiation therapy
Item C Wide local excision with axillary lymph node dissection and radiation therapy
Item D Simple mastectomy (without axillary lymph node dissection)
Item E M odified radical mastectomy (simple mastectomy with in-continuity axillary lymph
node dissection
Correct Answer c

Domain: PROBLEM SOLVING

Preference Bailys and loves 26 edition

Prepared by

College Frontier Medical & Dental College, Abbottabad

Name: prof dr johar ali

Signature:

Date:

Bahria University
Multiple Choices Question

Program MBBS
Subject Gen surgery
Prof Final year
TOS-ID 17.6.

Description BREAST BENIGN DISEASES


Stem A 30-year-old woman presents with multiple bilateral breast swellings which cause her
discomfort, particularly just before her periods. They have been present for several
years but appear to be getting worse.

What is the most likely diagnosis?


Item A Peau d’orange
Item B Fibrocystic disease
Item C Fibroadenoma
Item D Fat necrosis
Item E Duct ectasia

Correct Answer C

Domain: recall

Preference Bailys and loves 26 edition

Prepared by

College Frontier Medical & Dental College, Abbottabad

Name: prof dr johar ali

Signature:

Date:

Bahria University
Multiple Choices Question

Program MBBS
Subject Gen surgery
Prof Final year
TOS-ID 17,2,2

Description stomach
Stem A 74-year-old woman is admitted with upper gastrointestinal (GI) bleeding. She is
started on H 2 blockers, but experiences another bleeding episode. Endoscopy
documents diffuse gastric ulcerations. Omeprazole is added to the H2 antagonists as a
therapeutic approach to the management of acute gastric and duodenal ulcers. Which
of the following is the mechanism of action of omeprazole

Item A Blockage of the breakdown of mucosa-damaging metabolites of nonsteroidal anti-


inflammatory drugs (NSAIDs)

Item B Provision of a direct cytoprotective effect

Item C Buffering of gastric acids

Item D Inhibition of parietal cell hydrogen potassium ATPase (adenosine triphosphatase

Item E Inhibition of gastrin release and parietal cell acid production

Correct Answer D

Domain: recall

Preference Bailys and loves 26 edition

Prepared by

College Frontier Medical & Dental College, Abbottabad

Name: prof dr johar ali

Signature:

Date:

Bahria University
Multiple Choices Question

Program MBBS

Subject Gen surgery

Prof Final year

TOS-ID 17.2.2
Description stomach

Stem A 41-year-old man complains of regurgitation of saliva and of undigested food. An


esophagram reveals a dilated esophagus and a bird’s-beak deformity. Manometry
shows a hypertensive lower esophageal sphincter with failure to relax with deglutition.
Which of the following is the safest and most effective treatment of this condition?

Item A Medical treatment with sublingual nitroglycerin, nitrates, or calcium-channel blockers

Item B Repeated bougie dilations

Item C Injections of botulinum toxin directly into the lower esophageal sphincter

Item D Dilation with a volume-limited, pressure-control balloon

Item E Laproscopic esophagomyotomy

Correct Answer E

Domain: problem solving

Preference Bailys and loves 26 edition

Prepared by

College Frontier Medical & Dental College, Abbottabad

Name: prof dr johar ali

Signature:

Date:

Bahria University
Multiple Choices Question

Program MBBS
Subject Gen surgery
Prof Final year
TOS-ID 17.2.6
Description pancreas
Stem A 39-year-old previously healthy male is hospitalized for 2 weeks with epigastric pain
radiating to his back, nausea, and vomiting. Initial laboratory values revealed an
elevated amylase level consistent with acute pancreatitis. Five weeks following
discharge, he complains of early satiety, epigastric pain, and fevers. On presentation,
his temperature is 38.9°C (102°F) and his heart rate is 120 beats per minute; his white
blood cell (WBC) count is 24,000/mm3 and his amylase level is normal. He undergoes a
CT scan demonstrating a 6 cm by 6 cm rim-enhancing fluid collection in the body of the
pancreas. Which of the following would be the most definitive management of the
fluid collection?
Item A Antibiotic therapy alone
Item B CT-guided aspiration with repeat imaging in 2 to 3 days
Item C Antibiotics and CT-guided aspiration with repeat imaging in 2 to 3 days
Item D Antibiotics and percutaneous catheter drainage
Item E Surgical internal drainage of the fluid collection with a cyst-gastrostomy or Roux-en-Y
cyst-jejunostomy

Correct Answer D

Domain: PROBLEM SOLVING

Preference Bailys and loves 26 edition

Prepared by

College Frontier Medical & Dental College, Abbottabad

Name: prof dr johar ali

Signature:

Date:

Bahria University
Multiple Choices Question

Program MBBS
Subject Gen surgery
Prof Final year
TOS-ID 17.2.5

Description Gall bladder &bile duct


Stem A 56-year-old previously healthy physician notices that his eyes are yellow and he has
been losing weight. On physical examination the patient has jaundice and scleral
icterus with a benign abdomen. Transcutaneous ultrasound of the abdomen
demonstrates biliary ductal dilation without gallstones.
Which of the following is the most appropriate next step in the workup of this patient?

Item A Esophagogastroduodenoscopy (EGD)


Item B Endoscopic retrograde cholangiopancreatography (ERCP)
Item C Acute abdominal series
Item D Computed tomography (CT) scan
Item E Positron emission tomography (PET) scan

Correct Answer D

Domain: PROBLEM SOLVING

Preference Bailys and loves 26 edition

Prepared by

College Frontier Medical & Dental College, Abbottabad

Name: prof dr johar ali

Signature:

Date:

Bahria University
Multiple Choices Question

Program MBBS
Subject Gen surgery
Prof Final year
TOS-ID 17.2.2

Description stomach
Stem A 45-year-old woman with history of heavy nonsteroidal anti-inflammatory drug
ingestion presents with acute abdominal pain. She undergoes exploratory laparotomy
30 hours after onset of symptoms and is found to have a perforated duodenal ulcer.
Which of the following is the procedure of choice to treat her perforation?

Item A Simple closure with omental patch


Item B Truncal vagotomy and pyloroplasty
Item C Truncal vagotomy and antrectomy
Item D Highly selective vagotomy with omental patch
Item E Hemigastrectomy

Correct Answer A

Domain: PROBLEM SOLVING

Preference Bailys and loves 26 edition

Prepared by

College Frontier Medical & Dental College, Abbottabad

Name: prof dr johar ali

Signature:

Date:

Bahria University
Multiple Choices Question

Program MBBS
Subject Gen surgery
Prof Final year
TOS-ID 17.2.2
Description stomach
Stem A 55-year-old man complains of chronic intermittent epigastric pain. A gastroscopy
demonstrates a 2-cm prepyloric ulcer. Biopsy of the ulcer yields no malignant tissue.
After a 6-week trial of medical therapy, the ulcer is unchanged.
Which of the following is the best next step in his management?
Item A Repeat trial of medical therapy
Item B Local excision of the ulcer
Item C Highly selective vagotomy
Item D Partial gastrectomy with vagotomy and Billroth I reconstruction
Item E Vagotomy and pyloroplasty

Correct Answer D

Domain: problem solving

Preference Bailys and loves 26 edition

Prepared by

College Frontier Medical & Dental College, Abbottabad

Name: prof dr johar ali

Signature:

Date:

Bahria University
Multiple Choices Question

Program MBBS
Subject Gen surgery
Prof Final year
TOS-ID 17.2.8

Description Small and large intestine


Stem A 45-year-old man was discovered to have a hepatic flexure colon cancer during a
colonoscopy for anemia requiring transfusions. Upon exploration of his abdomen in
the operating room, an unexpected discontinuous 3-cm metastasis is discovered in the
edge of the right lobe of the liver. Preoperatively, the patient was counseled of this
possibility and the surgical options.
Which of the following is the most appropriate management of this patient?

Item A A diverting ileostomy should be performed and further imaging obtained


Item B Right hemicolectomy
Item C Right hemicolectomy with local resection of the liver metastasis
Item D Closure of the abdomen followed by chemotherapy
Item E Right hemicolectomy with postoperative radiation therapy to the liver

Correct Answer C

Domain: PROBLEM SOLVING

Preference Bailys and loves 26 edition

Prepared by

College Frontier Medical & Dental College, Abbottabad

Name: prof dr johar ali

Signature:

Date:

Bahria University
Multiple Choices Question

Program MBBS
Subject Gen surgery
Prof Final year
TOS-ID 17.2.13

Description Hernia, umbilicus, abdominal wall


Stem A 22-year-old college student notices a bulge in his right groin. It is accentuated with
coughing, but is easily reducible. Which of the following hernias follows the path of the
spermatic cord within the cremaster muscles
Item A Femoral
Item B Direct inguinal
Item C Indirect inguinal
Item D Spigelian
Item E Interparietal

Correct Answer C

Domain: recall

Preference Bailys and loves 26 edition

Prepared by

College Frontier Medical & Dental College, Abbottabad

Name: prof dr johar ali

Signature:

Date:

Bahria University
Multiple Choices Question

Program MBBS
Subject Gen surgery
Prof Final year
TOS-ID 17.2.7

Description GALL BLADDER AND BILE DUCT


Stem An 80-year-old man with history of symptomatic cholelithiasis presents with signs and
symptoms of a small-bowel obstruction. Which of the following findings would provide
the most help in ascertaining the diagnosis?

Item A Coffee-grounds aspirate from the stomach


Item B Pneumobilia
Item C A leukocyte count of 40,000/mL
Item D Gas under right dome of diapharam
Item E A palpable mass in the pelvis

Correct Answer B

Domain: recall

Preference Bailys and loves 26 edition

Prepared by

College Frontier Medical & Dental College, Abbottabad

Name: prof dr johar ali

Signature:

Date:

Bahria University
Multiple Choices Question

Program MBBS
Subject Gen surgery
Prof Final year
TOS-ID 17.2.13

Description Hernia
Stem A 22-year-old woman is seen in a surgery clinic for a bulge in the right groin. She
denies pain and is able to make the bulge disappear by lying down and putting steady
pressure on the bulge. She has never experienced nausea or vomiting. On examination
she has a reducible hernia below the inguinal ligament. Which of the following is the
most appropriate management of this patient?
Item A Observation for now and follow-up in surgery clinic in 6 months
Item B Observation for now and follow-up in surgery clinic if she develops further symptoms
Item C Elective surgical repair of hernia
Item D Emergent surgical repair of hernia
Item E Emergent surgical repair of hernia with exploratory laparotomy to evaluate the small
bowel

Correct Answer C

Domain: PROBLEM SOLVING

Preference Bailys and loves 26 edition

Prepared by

College Frontier Medical & Dental College, Abbottabad

Name: prof dr johar ali

Signature:

Date:

Bahria University
Multiple Choices Question

Program MBBS
Subject Gen surgery
Prof Final year
TOS-ID 17.2.10

Description Vermiform appendix


Stem A 20-year-old woman develops pain in the right lower quadrant . The pain progresses
and the patient presents to the emergency room later that day with a low-grade fever,
a WBC count of 13,000/mm3 and complaints of anorexia and nausea as well as
persistent, sharp pain of the right lower quadrant. On examination, she is tender in the
right lower quadrant with muscular spasm .
Which of the following is the most likely diagnosis?
Item A Acute appendicitis
Item B Cecal carcinoma
Item C Hematoma of the rectus sheath
Item D Torsion of an ovarian cyst
Item E Cholecystitis

Correct Answer A

Domain: INTERPRETATION

Preference Bailys and loves 26 edition

Prepared by

College Frontier Medical & Dental College, Abbottabad

Name: prof dr johar ali

Signature:

Date

Bahria University
Multiple Choices Question

Program MBBS
Subject Gen surgery
Prof Final year
TOS-ID 16.6

Description PILONIDAL ABCESS


Stem A 22-year-old woman presents with a painful fluctuant mass in the midline between
the gluteal folds. She denies pain on rectal examination.
Which of the following is the most likely diagnosis?
Item A Pilonidal abscess
Item B Perianal abscess
Item C Perirectal abscess
Item D Fistula-in-ano
Item E Anal fissure

Correct Answer A
Domain: RECALL

Preference Bailys and loves 26 edition

Prepared by

College Frontier Medical & Dental College, Abbottabad

Name: prof dr johar ali

Signature:

Date:

Bahria University
Multiple Choices Question

Program MBBS
Subject Gen surgery
Prof Final year
TOS-ID 17.2.3

Description LIVER
Stem A 39-year-old woman with no significant past medical history and whose only
medication is oral contraceptive pills (OCP) presents to the emergency room with right
upper quadrant pain. CT scan demonstrates a 6-cm hepatic adenoma in the right lobe
of the liver.
Which of the following describes the definitive treatment of this lesion?
Item A Cessation of oral contraceptives and serial CT scans
Item B Intra-arterial embolization of the hepatic adenoma
Item C Embolization of the right portal vein
Item D Resection of the hepatic adenoma
Item E Systemic chemotherapy
Correct Answer D

Domain: PROBLEM SOLVING

Preference Bailys and loves 26 edition

Prepared by

College Frontier Medical & Dental College, Abbottabad

Name: prof dr johar ali

Signature:

Date

Bahria University
Multiple Choices Question

Program MBBS
Subject Gen surgery
Prof Final year
TOS-ID 17.2.3

Description LIVER
Stem A 57-year-old previously alcoholic man with a history of chronic pancreatitis presents
with hematemesis. Endoscopy reveals isolated gastric varices in the absence of
esophageal varices. Ultrasound examination demonstrates normal portal flow but a
thrombosed splenic vein. He undergoes banding, which is initially successful, but he
subsequently rebleeds during the same hospitalization. Attempts to control the
bleeding endoscopically are unsuccessful.
Which of the following is the most appropriate next step in management?

Item A Transjugular intrahepatic portosystemic shunt


Item B Surgical portocaval shunt
Item C Surgical mesocaval shunt
Item D Splenectomy
Item E Placement of a Sengstaken-Blakemore tube

Correct Answer D

Domain: recall

Preference Bailys and loves 26 edition

Prepared by

College Frontier Medical & Dental College, Abbottabad

Name: prof dr johar ali

Signature:

Date:

Bahria University
Multiple Choices Question

Program MBBS
Subject Gen surgery
Prof Final year
TOS-ID 17.2.1

Description OESOPHAGUS
Stem A 62-year-old man has been noticing progressive difficulty swallowing, first solid food
and now liquids as well. A barium study shows a ragged narrowing just below the
carinal level. Endoscopic biopsy confirms squamous cell carcinoma.
Which of the following provides the most accurate information regarding the T stage
of an esophageal carcinoma?
Item A Computed tomography
Item B Positron emission tomography
Item C M agnetic resonance imaging
Item D Endoscopic ultrasound
Item E Bronchoscopy

Correct Answer D

Domain: PROBLEM SOLVING

Preference Bailys and loves 26 edition

Prepared by

College Frontier Medical & Dental College, Abbottabad

Name: prof dr johar ali

Signature:

Date:

Bahria University
Multiple Choices Question

Program MBBS
Subject Gen surgery
Prof Final year
TOS-ID 17.5.1

Description Chest trauma


Stem An 18-year-old woman is transported to your trauma unit after sustaining a side-
impact collision on her side of the car. She is hypotensive and in respiratory distress,
and has distended neck veins. Diagnosis of flail chest is entertained, what will be the
lifesaving intervension?

Item A Endotracheal intubation


Item B Cricothyroidotomy
Item C Subxiphoid window
Item D Tube thoracostomy

Item E Occlusive dressing

Correct Answer A

Domain: PROBLEM SOLVING

Preference Bailys and loves 26 edition

Prepared by

College Frontier Medical & Dental College, Abbottabad

Name: prof dr johar ali

Signature:

Date

Bahria University
Multiple Choices Question

Program MBBS
Subject Gen surgery
Prof Final year
TOS-ID 17.2.6

Description PANCREASE
Stem A 70-year-old woman presents to the emergency room complaining of severe
epigastric pain radiating to her back, nausea, and vomiting. CT scan of the abdomen
demonstrates inflammation and edema of the pancreas. A right upper quadrant
ultrasound demonstrates the presence of gallstones in the gallbladder.
Which of the following is an important prognostic sign in acute pancreatitis according
to Ranson’s criteria?
Item A Amylase level
Item B Age
Item C Total bilirubin level
Item D Albumin level
Item E Lipase level

Correct Answer B

Domain: INTERPRETATION

Preference Bailys and loves 26 edition

Prepared by

College Frontier Medical & Dental College, Abbottabad

Name: prof dr johar ali

Signature:

Date:

Bahria University
Multiple Choices Question

Program MBBS
Subject Gen surgery
Prof Final year
TOS-ID 17.2.6

Description PANCREASE
Stem A 65-year-old man who is extremely obese reports weakness, sweating, tachycardia,
confusion, and headache whenever he fasts for more than a few hours. He has prompt
relief of symptoms when he eats. Labarotory examination reveals an inappropriately
high level of serum insulin during the episodes of fasting.
Which of the following is the most appropriate treatment for this condition?

Item A Diet modification to include frequent meals


Item B Long-acting somatostatin analogue octreotide
Item C Simple excision of the tumor
Item D Total pancreatectomy
Item E Chemotherapy and radiation

Correct Answer C

Domain: PROBLEM SOLVING

Preference Bailys and loves 26 edition

Prepared by

College Frontier Medical & Dental College, Abbottabad

Name: prof dr johar ali

Signature:

Date:

Bahria University
Multiple Choices Question

Program MBBS
Subject Gen surgery
Prof Final year
TOS-ID 17.2.12

Description ANUS AND ANAL CANAL


Stem A 57-year-old woman sees blood on the toilet paper. Her doctor notes the presence of
an excoriated bleeding 2.8-cm mass at the anus. Biopsy confirms the clinical suspicion
of anal cancer. Which of the following is the best initial management strategy?

Item A Abdominoperineal resection


Item B Wide local resection with bilateral inguinal node dissection
Item C Local radiation therapy
Item D Systemic chemotherapy
Item E Combined radiation therapy and chemotherapy

Correct Answer E

Domain: PROBLEM SOLVING

Preference Bailys and loves 26 edition

Prepared by

College Frontier Medical & Dental College, Abbottabad

Name: prof dr johar ali

Signature:

Date:

Bahria University
Multiple Choices Question

Program MBBS
Subject Gen surgery
Prof Final year
TOS-ID 17.2.9

Description LARGE INTESTINAL OBSTRUCTION


Stem An 80-year-old man is admitted to the hospital complaining of nausea, abdominal pain,
distention, and spurious diarrhea. A cautiously performed transanal contrast study
reveals an apple-core configuration in the recto sigmoid area.
What is your most likely diagnosis?
Item A Carcinoma rectosigmoid junction
Item B Ulcerative collitis
Item C Chrons disease
Item D Familial adenomatous polyposis
Item E Congenital magacolon

Correct Answer A

Domain: INTERPRETATION

Preference Bailys and loves 26 edition

Prepared by

College Frontier Medical & Dental College, Abbottabad

Name: prof dr johar ali

Signature:

Date:

Bahria University
Multiple Choices Question

Program MBBS
Subject Gen surgery
Prof Final year
TOS-ID 17.2.10

Description VERMIFORM APPENDIX


Stem A 28-year-old woman who is 15 weeks pregnant has new onset of nausea, vomiting,
and right-sided abdominal pain. She has been free of nausea since early in her first
trimester. The pain has become worse over the past 6 hours.
Which of the following is the most common nonobstetric surgical disease of the
abdomen during pregnancy
Item A Appendicitis
Item B Cholecystitis
Item C Pancreatitis
Item D Intestinal obstruction
Item E Acute fatty liver of pregnancy

Correct Answer A

Domain: recall

Preference Bailys and loves 26 edition

Prepared by

College Frontier Medical & Dental College, Abbottabad

Name: prof dr johar ali

Signature:

Date:

Bahria University
Multiple Choices Question

Program MBBS
Subject Gen surgery
Prof Final year
TOS-ID 17.2.8

Description SMALL AND LARGE INTESTINE


Stem A 24-year-old man presents to the emergency room with abdominal pain and fever. CT
scan of the abdomen reveals inflammation of the colon. He is referred to a
gastroenterologist to be evaluated for inflammatory bowel disease (Crohn disease
versus ulcerative colitis).
Which of the following indications for surgery is more prevalent in patients with Crohn
disease?

Item A Toxic megacolon


Item B M assive bleeding
Item C Fistulas between the colon and segments of intestine, bladder, vagina, urethra, and
skin
Item D Intractable disease
Item E Dysplasia or carcinoma

Correct Answer C

Domain: INTERPRETATION

Preference Bailys and loves 26 edition

Prepared by

College Frontier Medical & Dental College, Abbottabad

Name: prof dr johar ali

Signature:

Date

Bahria University
Multiple Choices Question

Program MBBS
Subject Gen surgery
Prof Final year
TOS-ID 17.2.6

Description PANCREASE CARCINOMA


Stem A 54-year-old man complains that his eyes are yellow. His bilirubin is elevated. His
physical examination is unremarkable. A CT of the abdomen shows a small mass in the
head of the pancreas encasing the superior mesenteric artery. Cytology from the ERCP
is positive for cancer.
Which of the following is the most appropriate treatment for this patient?

Item A Pancreaticoduodenectomy
Item B Pancreaticoduodenectomy with reconstruction of the superior mesenteric artery
Item C Total pancreatectomy

Item D Total pancreatectomy with reconstruction of the superior mesenteric artery


Item E Chemoradiation therapy

Correct Answer E

Domain: recall

Preference Bailys and loves 26 edition

Prepared by

College Frontier Medical & Dental College, Abbottabad

Name: prof dr johar ali

Signature:

Date:

Bahria University
Multiple Choices Question

Program MBBS
Subject Gen surgery
Prof Final year
TOS-ID 17.2.1O

Description VERMIFORM APPENDIX


Stem A 32-year-old woman undergoes an uncomplicated appendectomy for acute
appendicitis. The pathology report notes the presence of a 1-cm carcinoid tumor in the
tip of the appendix.
Which of the following is the most appropriate management of this patient?
Item A Right hemicolectomy
Item B Right hemicolectomy and chemotherapy
Item C Chemotherapy only
Item D Radiation only
Item E No further treatment

Correct Answer E

Domain: recall

Preference Bailys and loves 26 edition

Prepared by

College Frontier Medical & Dental College, Abbottabad

Name: prof dr johar ali

Signature:

Date:

Bahria University
Multiple Choices Question

Program MBBS
Subject Gen surgery
Prof Final year
TOS-ID 17.2.3

Description Pancrease gastrinoma


Stem . A 52-year-old man with a family history of multiple endocrine neoplasia type 1
(M EN1) has an elevated gastrin level and is suspected to have a gastrinoma.
Which of the following is the most likely location for his tumor?
Item A Fundus of the stomach
Item B Antrum of the stomach
Item C Within the triangle formed by the junction of the second and third portions of the
duodenum, the junction of the neck and body of the pancreas, and the junction of the
cystic and common bile duct
Item D Tail of the pancreas
Item E Within the triangle formed by the inferior edge of the liver, the cystic duct, and the
common hepatic duct

Correct Answer C

Domain: recall

Preference Bailys and loves 26 edition

Prepared by

College Frontier Medical & Dental College, Abbottabad

Name: prof dr johar ali

Signature:

Date:

Bahria University
Multiple Choices Question

Program MBBS
Subject Gen surgery
Prof Final year
TOS-ID 17.2.3

Description LIVER
Stem A 45-year-old woman has an incidental finding of a liver mass on a CT scan. M agnetic
resonance imaging (M RI) is suggestive of a hemangioma.
Which of the following is the most appropriate management strategy for this patient?
Item A Observation
Item B Discontinuation of oral contraceptive pills
Item C Percutaneous biopsy of the lesion to confirm the diagnosis
Item D Resection of the hemangioma
Item E Liver transplantation

Correct Answer A

Domain: INTERPRETATION

Preference Bailys and loves 26 edition

Prepared by

College Frontier Medical & Dental College, Abbottabad

Name: prof dr johar ali

Signature:

Date:

Bahria University
Multiple Choices Question

Program MBBS
Subject Gen surgery
Prof Final year
TOS-ID 17.2.8

Description SMALL AND LARGE INTESTINE


Stem A 65-year-old man presents to his primary care physician with complaints of
intermittent constipation and is found to have microcytic anemia. Colonoscopy reveals
a fungating mass in the proximal sigmoid colon with no other synchronous lesions.
Biopsy of the mass confirms adenocarcinoma .
what will be the right surgical option?

Item A Left hemicolectomy


Item B Right hemicolectomy
Item C Subtotal colectomy
Item D Total colectomy
Item E Hepatic resection.

Correct Answer A

Domain: PROBLEM SOLVING

Preference Bailys and loves 26 edition

Prepared by

College Frontier Medical & Dental College, Abbottabad

Name: prof dr johar ali

Signature:

Date:

Bahria University
Multiple Choices Question

Program MBBS
Subject Gen surgery
Prof Final year
TOS-ID 17.2.5

Description Gall bladder and bile duct


Stem A 62-year-old man presents with right upper quadrant abdominal pain and jaundice.
He is afebrile with normal vital signs. On laboratory findings he has elevated levels of
bilirubin and alkaline phosphatase. Ultrasound demonstrates gallstones, normal
gallbladder wall thickness, no pericholecystic fluid, and a common bile duct of 1.0 cm.
What is your diagnosis?
Item A Symptomatic cholelithiasis
Item B Acute cholecystitis
Item C Gallstone pancreatitis
Item D Choledocholithiasis
Item E Cholangitis

Correct Answer D

Domain: INTERPRETATION

Preference Bailys and loves 26 edition

Prepared by

College Frontier Medical & Dental College, Abbottabad

Name: prof dr johar ali

Signature:

Date:

Bahria University
Multiple Choices Question

Program MBBS
Subject Gen surgery
Prof Final year
TOS-ID 17.2,6

Description pancreatitis
Stem A 23-year-old woman presents with epigastric abdominal pain and nausea. She is
afebrile with normal vital signs. On laboratory results she has no leukocytosis with
normal levels of bilirubin and alkaline phosphatase. The amylase and lipase are
elevated. Ultrasound demonstrates gallstones, normal gallbladder wall thickness, no
pericholecystic fluid, and a common bile duct of 3 mm.

What is your diagnosis?

Item A pancreatitis
Item B cholecyistitis
Item C cholangitis
Item D Symptomatic cholelithiasis
Item E Hepatitis

Correct Answer A

Domain: INTERPRETATION

Preference Bailys and loves 26 edition

Prepared by

College Frontier Medical & Dental College, Abbottabad

Name: prof dr johar ali

Signature:

Date:

Bahria University
Multiple Choices Question

Program MBBS
Subject Gen surgery
Prof FINAL YEAR
TOS-ID 17.3.5

Description TESTIS AND SCROTUM


Stem A 19-year-old college student presents with a testicular mass, and after treatment he
returns for regular follow-up visits. Which of the following is the most useful serum
marker for detecting recurrent disease after treatment of nonseminomatous testicular
cancer?
Item A Carcinoembryonic antigen (CEA)
Item B Human chorionic gonadotropin (hCG)

Item C Prostate-specific antigen (PSA)


Item D CA125
Item E p53 oncogene

Correct Answer B

Domain: recall

Preference Bailys and loves 26 edition

Prepared by

College Frontier Medical & Dental College, Abbottabad

Name:PROF DR JOHAR ALI

Signature:

Bahria University
Multiple Choices Question

Program MBBS
Subject Gen surgery
Prof FINAL YEAR
TOS-ID

Description
Stem Incisional biopsy of a breast mass in a 35-year-old woman demonstrates cystosarcoma
phyllodes at the time of frozen section. Which of the following is the most appropriate
management strategy for this lesion?

Item A Wide local excision with a rim of normal tissue

Item B Lumpectomy and axillary lymphadenectomy

Item C Modified radical mastectomy

Item D Excision and postoperative radiotherapy

Item E Excision, postoperative radiotherapy, and systemic chemotherapy


Correct Answer A
Domain: recall

Preference Bailys and loves 26 edition

Prepared by

College Frontier Medical & Dental College, Abbottabad

Name:PROF DR JOHAR ALI

Signature:

Date:

Bahria University
Multiple Choices Question

Program MBBS
Subject Gen surgery
Prof FINAL YEAR
TOS-ID 17.4.1

Description Thyroid
Stem A 36-year-old woman, 20 weeks pregnant, presents with a 1.5-cm right thyroid mass.
FNA is consistent with a papillary neoplasm. The mass is cold on scan and solid on
ultrasound.
Which of the following methods of treatment is contraindicated in this patient?
Item A Right thyroid lobectomy
Item B Subtotal thyroidectomy
Item C Total thyroidectomy
Item D Total thyroidectomy with lymph node dissection
Item E 131I radioactive ablation of the thyroid gland

Correct Answer E

Domain: problem solving

Preference Bailys and loves 26 edition


Prepared by

College Frontier Medical & Dental College, Abbottabad

Name:PROF DR JOHAR ALI

Signature:

Date:

Bahria University
Multiple Choices Question

Program MBBS
Subject Gen surgery
Prof FINAL YEAR
TOS-ID 17.4.1

Description thyroid
Stem A 63-year-old woman notices lumps on both sides of her neck. A fine-needle aspirate is
nondiagnostic, and she undergoes total thyroidectomy. Final pathology reveals a 2-cm
Hürthle cell carcinoma.
Which of the following is the most appropriate postsurgical management of this
patient?
Item A No further therapy is indicated.
Item B Chemotherapy.
Item C External beam radiotherapy .
Item D Radioiodine ablation.
Item E Chemotherapy, external beam radiotherapy, and radioiodine ablation.
Correct Answer D

Domain: PROBLEM SOLVING

Preference Bailys and loves 26 edition

Prepared by

College Frontier Medical & Dental College, Abbottabad

Name:PROF DR JOHAR ALI

Signature:

Date

Bahria University
Multiple Choices Question

Program MBBS
Subject Gen surgery
Prof FINAL YEAR
TOS-ID 17.4.1

Description THYROID
Stem A 51-year-old man presents with a 2-cm left thyroid nodule. Thyroid scan shows a cold
lesion. FNA cytology demonstrates follicular cells.
Which of the following is the most appropriate initial treatment of this patient?

Item A External beam radiation to the neck.


Item B Multidrug chemotherapy.
Item C TSH suppression by thyroid hormone.
Item D Prophylactic neck dissection is indicated along with a total thyroidectomy
Item E Thyroid lobectomy.

Correct Answer E

Domain: PROBLEM SOLVING

Preference Bailys and loves 26 edition


Prepared by

College Frontier Medical & Dental College, Abbottabad

Name:PROF DR JOHAR ALI

Signature:

Date:

Bahria University
Multiple Choices Question

Program MBBS
Subject Gen surgery
Prof Final year
TOS-ID 17.6.1

Description Breast malignant disease


Stem A 36-year-old woman whose mother has just undergone treatment for breast cancer is
asking about how this affects her and what can be done to lessen her chances of
having the disease.
Which of the following has the lowest risk factor for breast cancer?

Item A Dietary fat intake


Item B Paternal relative with breast cancer 1 (BRCA1) mutation
Item C Excessive estrogen exposure—early menarche, late menopause, nulliparity
Item D Previous biopsy with atypical hyperplasia

Item E Exposure to ionizing radiation

Correct Answer A
Domain: INTERPRETATION

Preference Bailys and loves 26 edition

Prepared by

College Frontier Medical & Dental College, Abbottabad

Name: prof dr johar ali

Signature:

Date:

Bahria University
Multiple Choices Question

Program MBBS
Subject Gen surgery
Prof Final year
TOS-ID 17.6.1

Description Breast malignant diseases


Stem A 40 years old lady presented with lump breast, size of Tumor is 4.0 cm; clinically
positive, movable axillary ipsilateral lymph nodes; no evidence of metastases. What is
the stage ?
Item A Stage 0
Item B Stage I
Item C Stage II

Item D Stage III


Item E Stage IV

Correct Answer C

Domain: interpretation

Preference Bailys and loves 26 edition

Prepared by
College Frontier Medical & Dental College, Abbottabad

Name: prof dr johar ali

Signature:

Date:

Bahria University
Multiple Choices Question

Program MBBS
Subject Gen surgery
Prof Final year
TOS-ID 17.2.10

Description Vermiform appendix


Stem 43-year-old man presents with signs and symptoms of peritonitis in the right lower
quadrant. The clinical impression and supportive data suggest acute appendicitis. At
exploration, however, a tumor is found to have carcinoid features. Its 2.5cm in size
located at the base of appendix.
What will be your choice of treatment?
Item A Appendectomy
Item B Segmental ileal resection
Item C Cecectomy
Item D Right hemi colectomy
Item E Hepatic wedge resection and appropriate bowel resection

Correct Answer D

Domain:

Preference Bailys and loves 26 edition

Prepared by
College Frontier Medical & Dental College, Abbottabad

Name: prof dr johar ali

Signature:

Date:

Bahria University
Multiple Choices Question

Program MBBS
Subject Gen surgery
Prof Final year
TOS-ID 17.6.1

Description Breast Malignant disease


Stem A 49-year-old woman has a 6-cm palpable mass that is biopsy -proven ductal
adenocarcinoma. She undergoes neoadjuvant chemotherapy which reduces the
tumor to 3 cm in size. However, she has palpable axillary lymph nodes; FNA
demonstrates adenocarcinoma. She desires breast conservation therapy if possible.
What option will be most appropriate in this case?
Item A Wide local excision
Item B Wide local excision with adjuvant radiation therapy
Item C Wide local excision with axillary lymph node dissection and radiation therapy
Item D Simple mastectomy (without axillary lymph node dissection)
Item E M odified radical mastectomy (simple mastectomy with in-continuity axillary lymph
node dissection

Correct Answer c

Domain: PROBLEM SOLVING

Preference Bailys and loves 26 edition


Prepared by

College Frontier Medical & Dental College, Abbottabad

Name: prof dr johar ali

Signature:

Date:

Bahria University
Multiple Choices Question

Program MBBS
Subject Gen surgery
Prof Final year
TOS-ID 17.6.

Description BREAST BENIGN DISEASES


Stem A 30-year-old woman presents with multiple bilateral breast swellings which cause her
discomfort, particularly just before her periods. They have been present for several
years but appear to be getting worse.

What is the most likely diagnosis?

Item A Peau d’orange


Item B Fibrocystic disease
Item C Fibroadenoma
Item D Fat necrosis
Item E Duct ectasia

Correct Answer C
Domain: recall

Preference Bailys and loves 26 edition

Prepared by

College Frontier Medical & Dental College, Abbottabad

Name: prof dr johar ali

Signature:

Date:

Bahria University
Multiple Choices Question

Program MBBS
Subject Gen surgery
Prof Final year
TOS-ID 17,2,2

Description stomach
Stem A 74-year-old woman is admitted with upper gastrointestinal (GI) bleeding. She is
started on H 2 blockers, but experiences another bleeding episode. Endoscopy
documents diffuse gastric ulcerations. Omeprazole is added to the H2 antagonists as a
therapeutic approach to the management of acute gastric and duodenal ulcers. Which
of the following is the mechanism of action of omeprazole

Item A Blockage of the breakdown of mucosa-damaging metabolites of nonsteroidal anti-


inflammatory drugs (NSAIDs)

Item B Provision of a direct cytoprotective effect


Item C Buffering of gastric acids

Item D Inhibition of parietal cell hydrogen potassium ATPase (adenosine triphosphatase

Item E Inhibition of gastrin release and parietal cell acid production

Correct Answer D

Domain: recall

Preference Bailys and loves 26 edition

Prepared by

College Frontier Medical & Dental College, Abbottabad

Name: prof dr johar ali

Signature:

Date:

Bahria University
Multiple Choices Question

Program MBBS

Subject Gen surgery

Prof Final year

TOS-ID 17.2.2

Description stomach

Stem A 41-year-old man complains of regurgitation of saliva and of undigested food. An


esophagram reveals a dilated esophagus and a bird’s-beak deformity. Manometry
shows a hypertensive lower esophageal sphincter with failure to relax with deglutition.
Which of the following is the safest and most effective treatment of this condition?

Item A Medical treatment with sublingual nitroglycerin, nitrates, or calcium-channel blockers


Item B Repeated bougie dilations

Item C Injections of botulinum toxin directly into the lower esophageal sphincter

Item D Dilation with a volume-limited, pressure-control balloon

Item E Laproscopic esophagomyotomy

Correct Answer E

Domain: problem solving

Preference Bailys and loves 26 edition

Prepared by

College Frontier Medical & Dental College, Abbottabad

Name: prof dr johar ali

Signature:

Date:

Bahria University
Multiple Choices Question

Program MBBS
Subject Gen surgery
Prof Final year
TOS-ID 17.2.6

Description pancreas
Stem A 39-year-old previously healthy male is hospitalized for 2 weeks with epigastric pain
radiating to his back, nausea, and vomiting. Initial laboratory values revealed an
elevated amylase level consistent with acute pancreatitis. Five weeks following
discharge, he complains of early satiety, epigastric pain, and fevers. On presentation,
his temperature is 38.9°C (102°F) and his heart rate is 120 beats per minute; his white
blood cell (WBC) count is 24,000/mm3 and his amylase level is normal. He undergoes a
CT scan demonstrating a 6 cm by 6 cm rim-enhancing fluid collection in the body of the
pancreas. Which of the following would be the most definitive management of the
fluid collection?
Item A Antibiotic therapy alone
Item B CT-guided aspiration with repeat imaging in 2 to 3 days
Item C Antibiotics and CT-guided aspiration with repeat imaging in 2 to 3 days
Item D Antibiotics and percutaneous catheter drainage
Item E Surgical internal drainage of the fluid collection with a cyst-gastrostomy or Roux-en-Y
cyst-jejunostomy

Correct Answer D

Domain: PROBLEM SOLVING

Preference Bailys and loves 26 edition

Prepared by

College Frontier Medical & Dental College, Abbottabad

Name: prof dr johar ali

Signature:

Date:

Bahria University
Multiple Choices Question

Program MBBS
Subject Gen surgery
Prof Final year
TOS-ID 17.2.5

Description Gall bladder &bile duct


Stem A 56-year-old previously healthy physician notices that his eyes are yellow and he has
been losing weight. On physical examination the patient has jaundice and scleral
icterus with a benign abdomen. Transcutaneous ultrasound of the abdomen
demonstrates biliary ductal dilation without gallstones.
Which of the following is the most appropriate next step in the workup of this patient?
Item A Esophagogastroduodenoscopy (EGD)
Item B Endoscopic retrograde cholangiopancreatography (ERCP)
Item C Acute abdominal series
Item D Computed tomography (CT) scan
Item E Positron emission tomography (PET) scan

Correct Answer D

Domain: PROBLEM SOLVING

Preference Bailys and loves 26 edition

Prepared by

College Frontier Medical & Dental College, Abbottabad

Name: prof dr johar ali

Signature:

Date:

Bahria University
Multiple Choices Question

Program MBBS
Subject Gen surgery
Prof Final year
TOS-ID 17.2.2

Description stomach
Stem A 45-year-old woman with history of heavy nonsteroidal anti-inflammatory drug
ingestion presents with acute abdominal pain. She undergoes exploratory laparotomy
30 hours after onset of symptoms and is found to have a perforated duodenal ulcer.
Which of the following is the procedure of choice to treat her perforation?

Item A Simple closure with omental patch


Item B Truncal vagotomy and pyloroplasty
Item C Truncal vagotomy and antrectomy
Item D Highly selective vagotomy with omental patch
Item E Hemigastrectomy

Correct Answer A

Domain: PROBLEM SOLVING

Preference Bailys and loves 26 edition

Prepared by

College Frontier Medical & Dental College, Abbottabad

Name: prof dr johar ali

Signature:

Date:

Bahria University
Multiple Choices Question

Program MBBS
Subject Gen surgery
Prof Final year
TOS-ID 17.2.2

Description stomach
Stem A 55-year-old man complains of chronic intermittent epigastric pain. A gastroscopy
demonstrates a 2-cm prepyloric ulcer. Biopsy of the ulcer yields no malignant tissue.
After a 6-week trial of medical therapy, the ulcer is unchanged.
Which of the following is the best next step in his management?
Item A Repeat trial of medical therapy
Item B Local excision of the ulcer
Item C Highly selective vagotomy
Item D Partial gastrectomy with vagotomy and Billroth I reconstruction
Item E Vagotomy and pyloroplasty

Correct Answer D

Domain: problem solving

Preference Bailys and loves 26 edition

Prepared by

College Frontier Medical & Dental College, Abbottabad

Name: prof dr johar ali

Signature:

Date:

Bahria University
Multiple Choices Question

Program MBBS
Subject Gen surgery
Prof Final year
TOS-ID 17.2.8

Description Small and large intestine


Stem A 45-year-old man was discovered to have a hepatic flexure colon cancer during a
colonoscopy for anemia requiring transfusions. Upon exploration of his abdomen in
the operating room, an unexpected discontinuous 3-cm metastasis is discovered in the
edge of the right lobe of the liver. Preoperatively, the patient was counseled of this
possibility and the surgical options.
Which of the following is the most appropriate management of this patient?

Item A A diverting ileostomy should be performed and further imaging obtained


Item B Right hemicolectomy
Item C Right hemicolectomy with local resection of the liver metastasis
Item D Closure of the abdomen followed by chemotherapy
Item E Right hemicolectomy with postoperative radiation therapy to the liver

Correct Answer C

Domain: PROBLEM SOLVING

Preference Bailys and loves 26 edition

Prepared by

College Frontier Medical & Dental College, Abbottabad

Name: prof dr johar ali

Signature:

Date:

Bahria University
Multiple Choices Question

Program MBBS
Subject Gen surgery
Prof Final year
TOS-ID 17.2.13

Description Hernia, umbilicus, abdominal wall


Stem A 22-year-old college student notices a bulge in his right groin. It is accentuated with
coughing, but is easily reducible. Which of the following hernias follows the path of the
spermatic cord within the cremaster muscles
Item A Femoral
Item B Direct inguinal
Item C Indirect inguinal
Item D Spigelian
Item E Interparietal
Correct Answer C

Domain: recall

Preference Bailys and loves 26 edition

Prepared by

College Frontier Medical & Dental College, Abbottabad

Name: prof dr johar ali

Signature:

Date:

Bahria University
Multiple Choices Question

Program MBBS
Subject Gen surgery
Prof Final year
TOS-ID 17.2.7

Description GALL BLADDER AND BILE DUCT


Stem An 80-year-old man with history of symptomatic cholelithiasis presents with signs and
symptoms of a small-bowel obstruction. Which of the following findings would provide
the most help in ascertaining the diagnosis?
Item A Coffee-grounds aspirate from the stomach
Item B Pneumobilia
Item C A leukocyte count of 40,000/mL
Item D Gas under right dome of diapharam
Item E A palpable mass in the pelvis

Correct Answer B

Domain: recall

Preference Bailys and loves 26 edition

Prepared by

College Frontier Medical & Dental College, Abbottabad

Name: prof dr johar ali

Signature:

Date:

Bahria University
Multiple Choices Question

Program MBBS
Subject Gen surgery
Prof Final year
TOS-ID 17.2.13

Description Hernia
Stem A 22-year-old woman is seen in a surgery clinic for a bulge in the right groin. She
denies pain and is able to make the bulge disappear by lying down and putting steady
pressure on the bulge. She has never experienced nausea or vomiting. On examination
she has a reducible hernia below the inguinal ligament. Which of the following is the
most appropriate management of this patient?
Item A Observation for now and follow-up in surgery clinic in 6 months
Item B Observation for now and follow-up in surgery clinic if she develops further symptoms
Item C Elective surgical repair of hernia
Item D Emergent surgical repair of hernia
Item E Emergent surgical repair of hernia with exploratory laparotomy to evaluate the small
bowel

Correct Answer C

Domain: PROBLEM SOLVING

Preference Bailys and loves 26 edition

Prepared by

College Frontier Medical & Dental College, Abbottabad

Name: prof dr johar ali

Signature:

Date:

Bahria University
Multiple Choices Question

Program MBBS
Subject Gen surgery
Prof Final year
TOS-ID 17.2.10

Description Vermiform appendix


Stem A 20-year-old woman develops pain in the right lower quadrant . The pain progresses
and the patient presents to the emergency room later that day with a low-grade fever,
a WBC count of 13,000/mm3 and complaints of anorexia and nausea as well as
persistent, sharp pain of the right lower quadrant. On examination, she is tender in the
right lower quadrant with muscular spasm .
Which of the following is the most likely diagnosis?
Item A Acute appendicitis
Item B Cecal carcinoma
Item C Hematoma of the rectus sheath
Item D Torsion of an ovarian cyst
Item E Cholecystitis

Correct Answer A

Domain: INTERPRETATION

Preference Bailys and loves 26 edition

Prepared by

College Frontier Medical & Dental College, Abbottabad

Name: prof dr johar ali

Signature:

Date

Bahria University
Multiple Choices Question

Program MBBS
Subject Gen surgery
Prof Final year
TOS-ID 16.6

Description PILONIDAL ABCESS


Stem A 22-year-old woman presents with a painful fluctuant mass in the midline between
the gluteal folds. She denies pain on rectal examination.
Which of the following is the most likely diagnosis?
Item A Pilonidal abscess
Item B Perianal abscess
Item C Perirectal abscess
Item D Fistula-in-ano
Item E Anal fissure

Correct Answer A

Domain: RECALL

Preference Bailys and loves 26 edition

Prepared by
College Frontier Medical & Dental College, Abbottabad

Name: prof dr johar ali

Signature:

Date:

Bahria University
Multiple Choices Question

Program MBBS
Subject Gen surgery
Prof Final year
TOS-ID 17.2.3

Description LIVER
Stem A 39-year-old woman with no significant past medical history and whose only
medication is oral contraceptive pills (OCP) presents to the emergency room with right
upper quadrant pain. CT scan demonstrates a 6-cm hepatic adenoma in the right lobe
of the liver.
Which of the following describes the definitive treatment of this lesion?
Item A Cessation of oral contraceptives and serial CT scans
Item B Intra-arterial embolization of the hepatic adenoma
Item C Embolization of the right portal vein
Item D Resection of the hepatic adenoma
Item E Systemic chemotherapy

Correct Answer D

Domain: PROBLEM SOLVING

Preference Bailys and loves 26 edition


Prepared by

College Frontier Medical & Dental College, Abbottabad

Name: prof dr johar ali

Signature:

Date

Bahria University
Multiple Choices Question

Program MBBS
Subject Gen surgery
Prof Final year
TOS-ID 17.2.3

Description LIVER
Stem A 57-year-old previously alcoholic man with a history of chronic pancreatitis presents
with hematemesis. Endoscopy reveals isolated gastric varices in the absence of
esophageal varices. Ultrasound examination demonstrates normal portal flow but a
thrombosed splenic vein. He undergoes banding, which is initially successful, but he
subsequently rebleeds during the same hospitalization. Attempts to control the
bleeding endoscopically are unsuccessful.
Which of the following is the most appropriate next step in management?

Item A Transjugular intrahepatic portosystemic shunt


Item B Surgical portocaval shunt
Item C Surgical mesocaval shunt
Item D Splenectomy
Item E Placement of a Sengstaken-Blakemore tube

Correct Answer D

Domain: recall
Preference Bailys and loves 26 edition

Prepared by

College Frontier Medical & Dental College, Abbottabad

Name: prof dr johar ali

Signature:

Date:

Bahria University
Multiple Choices Question

Program MBBS
Subject Gen surgery
Prof Final year
TOS-ID 17.2.1

Description OESOPHAGUS
Stem A 62-year-old man has been noticing progressive difficulty swallowing, first solid food
and now liquids as well. A barium study shows a ragged narrowing just below the
carinal level. Endoscopic biopsy confirms squamous cell carcinoma.
Which of the following provides the most accurate information regarding the T stage
of an esophageal carcinoma?
Item A Computed tomography
Item B Positron emission tomography
Item C M agnetic resonance imaging
Item D Endoscopic ultrasound
Item E Bronchoscopy

Correct Answer D
Domain: PROBLEM SOLVING

Preference Bailys and loves 26 edition

Prepared by

College Frontier Medical & Dental College, Abbottabad

Name: prof dr johar ali

Signature:

Date:

Bahria University
Multiple Choices Question

Program MBBS
Subject Gen surgery
Prof Final year
TOS-ID 17.5.1

Description Chest trauma


Stem An 18-year-old woman is transported to your trauma unit after sustaining a side-
impact collision on her side of the car. She is hypotensive and in respiratory distress,
and has distended neck veins. Diagnosis of flail chest is entertained, what will be the
lifesaving intervension?

Item A Endotracheal intubation


Item B Cricothyroidotomy
Item C Subxiphoid window
Item D Tube thoracostomy

Item E Occlusive dressing

Correct Answer A

Domain: PROBLEM SOLVING

Preference Bailys and loves 26 edition


Prepared by

College Frontier Medical & Dental College, Abbottabad

Name: prof dr johar ali

Signature:

Date

Bahria University
Multiple Choices Question

Program MBBS
Subject Gen surgery
Prof Final year
TOS-ID 17.2.6

Description PANCREASE
Stem A 70-year-old woman presents to the emergency room complaining of severe
epigastric pain radiating to her back, nausea, and vomiting. CT scan of the abdomen
demonstrates inflammation and edema of the pancreas. A right upper quadrant
ultrasound demonstrates the presence of gallstones in the gallbladder.
Which of the following is an important prognostic sign in acute pancreatitis according
to Ranson’s criteria?

Item A Amylase level


Item B Age
Item C Total bilirubin level
Item D Albumin level
Item E Lipase level

Correct Answer B

Domain: INTERPRETATION
Preference Bailys and loves 26 edition

Prepared by

College Frontier Medical & Dental College, Abbottabad

Name: prof dr johar ali

Signature:

Date:

Bahria University
Multiple Choices Question

Program MBBS
Subject Gen surgery
Prof Final year
TOS-ID 17.2.6

Description PANCREASE
Stem A 65-year-old man who is extremely obese reports weakness, sweating, tachycardia,
confusion, and headache whenever he fasts for more than a few hours. He has prompt
relief of symptoms when he eats. Labarotory examination reveals an inappropriately
high level of serum insulin during the episodes of fasting.
Which of the following is the most appropriate treatment for this condition?

Item A Diet modification to include frequent meals


Item B Long-acting somatostatin analogue octreotide
Item C Simple excision of the tumor
Item D Total pancreatectomy
Item E Chemotherapy and radiation

Correct Answer C
Domain: PROBLEM SOLVING

Preference Bailys and loves 26 edition

Prepared by

College Frontier Medical & Dental College, Abbottabad

Name: prof dr johar ali

Signature:

Date:

Bahria University
Multiple Choices Question

Program MBBS
Subject Gen surgery
Prof Final year
TOS-ID 17.2.12

Description ANUS AND ANAL CANAL


Stem A 57-year-old woman sees blood on the toilet paper. Her doctor notes the presence of
an excoriated bleeding 2.8-cm mass at the anus. Biopsy confirms the clinical suspicion
of anal cancer. Which of the following is the best initial management strategy?

Item A Abdominoperineal resection


Item B Wide local resection with bilateral inguinal node dissection
Item C Local radiation therapy
Item D Systemic chemotherapy
Item E Combined radiation therapy and chemotherapy

Correct Answer E

Domain: PROBLEM SOLVING

Preference Bailys and loves 26 edition


Prepared by

College Frontier Medical & Dental College, Abbottabad

Name: prof dr johar ali

Signature:

Date:

Bahria University
Multiple Choices Question

Program MBBS
Subject Gen surgery
Prof Final year
TOS-ID 17.2.9

Description LARGE INTESTINAL OBSTRUCTION


Stem An 80-year-old man is admitted to the hospital complaining of nausea, abdominal pain,
distention, and spurious diarrhea. A cautiously performed transanal contrast study
reveals an apple-core configuration in the recto sigmoid area.
What is your most likely diagnosis?
Item A Carcinoma rectosigmoid junction
Item B Ulcerative collitis
Item C Chrons disease
Item D Familial adenomatous polyposis
Item E Congenital magacolon

Correct Answer A

Domain: INTERPRETATION

Preference Bailys and loves 26 edition


Prepared by

College Frontier Medical & Dental College, Abbottabad

Name: prof dr johar ali

Signature:

Date:

Bahria University
Multiple Choices Question

Program MBBS
Subject Gen surgery
Prof Final year
TOS-ID 17.2.10

Description VERMIFORM APPENDIX


Stem A 28-year-old woman who is 15 weeks pregnant has new onset of nausea, vomiting,
and right-sided abdominal pain. She has been free of nausea since early in her first
trimester. The pain has become worse over the past 6 hours.
Which of the following is the most common nonobstetric surgical disease of the
abdomen during pregnancy
Item A Appendicitis
Item B Cholecystitis
Item C Pancreatitis
Item D Intestinal obstruction
Item E Acute fatty liver of pregnancy

Correct Answer A

Domain: recall
Preference Bailys and loves 26 edition

Prepared by

College Frontier Medical & Dental College, Abbottabad

Name: prof dr johar ali

Signature:

Date:

Bahria University
Multiple Choices Question

Program MBBS
Subject Gen surgery
Prof Final year
TOS-ID 17.2.8

Description SMALL AND LARGE INTESTINE


Stem A 24-year-old man presents to the emergency room with abdominal pain and fever. CT
scan of the abdomen reveals inflammation of the colon. He is referred to a
gastroenterologist to be evaluated for inflammatory bowel disease (Crohn disease
versus ulcerative colitis).
Which of the following indications for surgery is more prevalent in patients with Crohn
disease?

Item A Toxic megacolon


Item B M assive bleeding
Item C Fistulas between the colon and segments of intestine, bladder, vagina, urethra, and
skin
Item D Intractable disease
Item E Dysplasia or carcinoma
Correct Answer C

Domain: INTERPRETATION

Preference Bailys and loves 26 edition

Prepared by

College Frontier Medical & Dental College, Abbottabad

Name: prof dr johar ali

Signature:

Date

Bahria University
Multiple Choices Question

Program MBBS
Subject Gen surgery
Prof Final year
TOS-ID 17.2.6

Description PANCREASE CARCINOMA


Stem A 54-year-old man complains that his eyes are yellow. His bilirubin is elevated. His
physical examination is unremarkable. A CT of the abdomen shows a small mass in the
head of the pancreas encasing the superior mesenteric artery. Cytology from the ERCP
is positive for cancer.
Which of the following is the most appropriate treatment for this patient?

Item A Pancreaticoduodenectomy
Item B Pancreaticoduodenectomy with reconstruction of the superior mesenteric artery
Item C Total pancreatectomy

Item D Total pancreatectomy with reconstruction of the superior mesenteric artery


Item E Chemoradiation therapy

Correct Answer E
Domain: recall

Preference Bailys and loves 26 edition

Prepared by

College Frontier Medical & Dental College, Abbottabad

Name: prof dr johar ali

Signature:

Date:

Bahria University
Multiple Choices Question

Program MBBS
Subject Gen surgery
Prof Final year
TOS-ID 17.2.1O

Description VERMIFORM APPENDIX


Stem A 32-year-old woman undergoes an uncomplicated appendectomy for acute
appendicitis. The pathology report notes the presence of a 1-cm carcinoid tumor in the
tip of the appendix.
Which of the following is the most appropriate management of this patient?
Item A Right hemicolectomy
Item B Right hemicolectomy and chemotherapy
Item C Chemotherapy only
Item D Radiation only
Item E No further treatment

Correct Answer E

Domain: recall

Preference Bailys and loves 26 edition

Prepared by
College Frontier Medical & Dental College, Abbottabad

Name: prof dr johar ali

Signature:

Date:

Bahria University
Multiple Choices Question

Program MBBS
Subject Gen surgery
Prof Final year
TOS-ID 17.2.3

Description Pancrease gastrinoma


Stem . A 52-year-old man with a family history of multiple endocrine neoplasia type 1
(M EN1) has an elevated gastrin level and is suspected to have a gastrinoma.
Which of the following is the most likely location for his tumor?
Item A Fundus of the stomach
Item B Antrum of the stomach
Item C Within the triangle formed by the junction of the second and third portions of the
duodenum, the junction of the neck and body of the pancreas, and the junction of the
cystic and common bile duct
Item D Tail of the pancreas
Item E Within the triangle formed by the inferior edge of the liver, the cystic duct, and the
common hepatic duct

Correct Answer C

Domain: recall

Preference Bailys and loves 26 edition


Prepared by

College Frontier Medical & Dental College, Abbottabad

Name: prof dr johar ali

Signature:

Date:

Bahria University
Multiple Choices Question

Program MBBS
Subject Gen surgery
Prof Final year
TOS-ID 17.2.3

Description LIVER
Stem A 45-year-old woman has an incidental finding of a liver mass on a CT scan. M agnetic
resonance imaging (M RI) is suggestive of a hemangioma.
Which of the following is the most appropriate management strategy for this patient?
Item A Observation
Item B Discontinuation of oral contraceptive pills
Item C Percutaneous biopsy of the lesion to confirm the diagnosis
Item D Resection of the hemangioma
Item E Liver transplantation

Correct Answer A

Domain: INTERPRETATION

Preference Bailys and loves 26 edition

Prepared by

College Frontier Medical & Dental College, Abbottabad


Name: prof dr johar ali

Signature:

Date:

Bahria University
Multiple Choices Question

Program MBBS
Subject Gen surgery
Prof Final year
TOS-ID 17.2.8

Description SMALL AND LARGE INTESTINE


Stem A 65-year-old man presents to his primary care physician with complaints of
intermittent constipation and is found to have microcytic anemia. Colonoscopy reveals
a fungating mass in the proximal sigmoid colon with no other synchronous lesions.
Biopsy of the mass confirms adenocarcinoma .
what will be the right surgical option?

Item A Left hemicolectomy


Item B Right hemicolectomy
Item C Subtotal colectomy
Item D Total colectomy
Item E Hepatic resection.

Correct Answer A

Domain: PROBLEM SOLVING

Preference Bailys and loves 26 edition


Prepared by

College Frontier Medical & Dental College, Abbottabad

Name: prof dr johar ali

Signature:

Date:

Bahria University
Multiple Choices Question

Program MBBS
Subject Gen surgery
Prof Final year
TOS-ID 17.2.5

Description Gall bladder and bile duct


Stem A 62-year-old man presents with right upper quadrant abdominal pain and jaundice.
He is afebrile with normal vital signs. On laboratory findings he has elevated levels of
bilirubin and alkaline phosphatase. Ultrasound demonstrates gallstones, normal
gallbladder wall thickness, no pericholecystic fluid, and a common bile duct of 1.0 cm.
What is your diagnosis?
Item A Symptomatic cholelithiasis
Item B Acute cholecystitis
Item C Gallstone pancreatitis
Item D Choledocholithiasis
Item E Cholangitis

Correct Answer D

Domain: INTERPRETATION
Preference Bailys and loves 26 edition

Prepared by

College Frontier Medical & Dental College, Abbottabad

Name: prof dr johar ali

Signature:

Date:

Bahria University
Multiple Choices Question

Program MBBS
Subject Gen surgery
Prof Final year
TOS-ID 17.2,6

Description pancreatitis
Stem A 23-year-old woman presents with epigastric abdominal pain and nausea. She is
afebrile with normal vital signs. On laboratory results she has no leukocytosis with
normal levels of bilirubin and alkaline phosphatase. The amylase and lipase are
elevated. Ultrasound demonstrates gallstones, normal gallbladder wall thickness, no
pericholecystic fluid, and a common bile duct of 3 mm.

What is your diagnosis?

Item A pancreatitis
Item B cholecyistitis
Item C cholangitis
Item D Symptomatic cholelithiasis
Item E Hepatitis

Correct Answer A
Domain: INTERPRETATION

Preference Bailys and loves 26 edition

Prepared by

College Frontier Medical & Dental College, Abbottabad

Name: prof dr johar ali

Signature:

Date:

Uro mcq August 2013


16.6.3
1 A colicky pain with sharp exacerbations against a constant back ground is a feature of :
A.medical renal diseases
B.renal cell carcinoma
C.renal tuberculosis
D.ureteric calculous
E. urinary tract infection

Interpretation

Bailey and loves 24 edition page no 1295.

Prof Dr johar ali


FMDC Abbottabad

2 Pain from lower ureteric calculous refers to.select the right option:
A. Referred to epigasteric region
B. umbilical region
C. Iliac fossa
D.groin ,scrotum and labium
E.lower back

Recall

Bailey and loves 24 edition page no 1295.

Prof Dr johar ali


FMDC Abbottabad
3. An adult of 45 years with a solitary kidney has ureteric calculous at the lower end near urinary
bladder . He has anuria pain flank on affected site and high grade fever . Ultrasonography
reveals moderate to severe proximal dilatation of renal tract. His blood urea and serum
Creatinine are high.
What will be your management of choice?
A.Iv antibiotic
B.Intravenous normal saline and iv antibiotics
C.Immediate urinary bladder catheterization
D open ureterolithotomy
E iv antibiotics and percutaneous nephrostomy .

Problem solving

Bailey and loves 26 edition page no 1294.


Prof Dr johar ali
FMDC Abbottabad

4.
A child of 4 years brought to you with history of recurrent urinary Tract infection . His parents has urine
culture and sensitivity reports with them. Child has been prescribed variety of antibiotics of varying
spectrums.
What will be your next investigation?
A x ray KUB
B .urine r/e
C. Renal isotope scan DMSA
D.Iv urography
E. Antegrade pyelography.

Problem solving

Bailey and loves edition 26 page no 1298.


Prof Dr johar ali
FMDC Abbottabad.

5. In a patient with Chronic pyelonephritis select the incorrect option:


A a common cause of end stage renal failure
B associated with ureteric reflux
C may be symptomatically silient
D leads to progressive renal scarring
E it's almost always has some congenital anomaly in the renal tract.

Recall
Bailey and loves 26 edition page 1300

Prof Dr johar ali


FMDC Abbottabad
6 perinephric abscess in a diabetic patient is a surgical emergency.
What is best management option:
A broad spectrum anti biotics
B ureteric stenting
C incision and drainage
D nephrostomy
E leprotomy from midline and debridement of necrotic tissues.

Problem solving

Baily and loves 26 edition page 1301.

Prof Dr johar ali


FMDC Abbottabad

FRONTIER MEDICAL & DENTAL COLLEGE, ABBOTTABAD

FINAL PROFESSIONAL EXAMINATION 2014

SURGERY-B

PAPER-1

MCQs

Total Marks: 60 Time Allowed 60 minutes

Note: Encircle the correct answer

1. A 48 year old woman develops constipation postoperatively and self medicates with Milk of
Magnesia. She presents to clinic, at which time her serum electrolytes are checked, and she is noted to
have an elevated serum magnesium level. Which of the following represents the earliest clinical
indication of hypomagnesaemia?

a. Loss of deep tendon reflexes


b. Flaccid paralysis
c. Respiratory arrest
d. Hypotension
e. Stupor
2. Five days after an uneventful cholecystectomy, an asymptomatic middle aged woman is found to
have a serum sodium level of 125 mEq/L. Which of the following is the most appropriate management
strategy for this patient?

a. Administration of hypertonic saline solution


b. Restriction of free water
c. Plasma ultra-filtration
d. Hemodialysis
e. Aggressive diuresis with furosemide

3. A patient with a nonobstructing carcinoma of the sigmoid colon is being prepared for elective
resection. Which of the following reduces the risk of postoperative infectious complications?

a. A single preoperative parenteral dose of antibiotic effective against aerobes and anaerobes
b. Avoidance of oral antibiotics to prevent emergence of Clostridium difficile
c. Postoperative administration for 48 hours of parenteral antibiotics effective against aerobes and
anaerobes
d. Postoperative administration of parenteral antibiotics effective against aerobes and anaerobs
until the patient s intravenous lines and all other drains are removed
e. Redosing of antibiotics in the operating room if the case lasts for more than 2 hours

4. A 52 year old man with gastric outlet obstruction secondary toa duodenal ulcer presents with
hypochloremic, hypokalemic metabolic alkalosis. Which of the following is the most appropriate therapy
for this patient?

a. Infusion of 0.9% NaC1 with supplemental KC1 until clinical signs of volume depletion are
eliminated
b. Infusion of isotonic (0.15N) HC1 via a central venous catheter
c. Clamping the nasogastric tube to prevent further acid losses
d. Administration of acetazolamide to promote renal excretion of bicarbonate
e. Intubation and controlled hypoventilation on a volume cycled ventilator to further increase Pco2
5. A 65 year old man undergoes a low anterior resection for rectal cancer. On the fifth day in hospital,
his physical examination shows a temperature of 39º (102ºF), blood pressure of 150/90 mm Hg, pulse of
110 beats per minute and regular, and respiratory rate of 28 breaths per minute. A computed
tomography (CT) scan of the abdomen reveals an abscess in the pelvis. Which of the following most
accurately describes his present condition?

a. Systemic inflammatory response syndrome (SIRS)


b. Sepsis
c. Severe sepsis
d. Septic shock
e. Severe septic shock

6. A victim of blunt abdominal trauma undergoes a partial hepatectomy. During surgery, he receives
twelve units of packed red blood cells. I n the recovery room, he is noted to be bleeding from
intravenous puncture sites and the surgical incision. Which of the following statements regarding the
coagulopathy is most likely true?

a. The patient has an unknown primary bleeding disorder


b. The coagulopathy is secondary to the partial hepatectomy
c. The coagulopathy is secondary to dilutional thrombocytopenia and deficiency of clotting
factory from the massive blood transfusion
d. The treatment is oral vitamin K
e. The treatment is intravenous vitamin K

7. On postoperative day 5 , an otherwise healthy 55 year old man recovering from a partial hepatectomy
is noted to have a fever of 38.6ºC (101. 5ºF). Which of the following is the most common nosocomial
infection postoperatively?

a. Wound infection
b. Pneumonia
c. Urinary tract infection
d. Intra-abdominal abscess
e. Intravenous catheter related infection

8. A 26 year old male is resuscitated with packed red blood cells following a motor vehicle collision
complicated by a fractured pelvis and resultant hemorrhage. A few hours later the patient becomes
hypotensive with a normal central venous pressure (CVP), oliguric, and febrile. Upon examination, the
patient is noted to have profuse oozing of blood from his intravenous (V) sites. Which of the following is
the most likely diagnosis?
a. Hypovolemic shock
b. Acute adrenal insufficiency
c. Gram negative bacteremia
d. Transfusion reaction
e. Ureteral obstruction

9. A 23 year old woman undergoes total thyroidectomy for carcinoma of the thyroid gland. On the
second postoperative day, she begins to complain of a tingling sensation I n her hands. She appears
quite anxious and later complains of muscle cramps. Which of the following is the most appropriate
initial management strategy?

a. 10 mL of 10% magnesium sulfate intravenously


b. Oral vitamin D
c. 100ug oral Synthroid
d. Continuous infusion of calcium gluconate
e. Oral calcium gluconate

10. A 42 year old man sustains a gunshot wound to the abdomen and is in shock. Multiple units of
packed red blood cells are transfused in an effort to resuscitate him. He complains of numbness around
his mouth and displays carpopedal spasm and a positive Chvostek sign. An electrocardiogram
demonstrates a prolonged QT interval. Which of the following is the most appropriate treatment?

a. Intravenous bicarbonate
b. Intravenous potassium
c. Intravenous calcium
d. Intravenous digoxin
e. Intravenous parathyroid hormone

11. Pelvic abscess. All are true Except

a. Lies extra-peritoneally
b. May be a complication of abdominal surgery
c. Often presents with diarrhea
d. Can be readily diagnosed by clinical examination
e. Should be treated with antibiotics alone

12. Paralytic ileus. All are true Except


a. Is associated with electrolyte imbalance
b. May be associated with mechanical intestinal obstruction
c. Requires treatment with nasogastric suction and intravenous fluids
d. Is associated with retroperitoneal haematoma
e. Will sometimes respond to atropine

13. Cancer of the tongue. All are true Except

a. Is usually an adenocarcinoma
b. Is more common in males
c. Most commonly presents as an indolent ulcer
d. Metastasizes via the lymphatic at an early stage of the disease
e. Is best treated by surgery and radiotherapy

14. The symptoms of peptic oesophagitis. All are true Except

a. Include intermittent dysphagia


b. Are often associated with those of anaemia
c. Include retrosternal discomfort
d. Are more likely to occur when the patient is in the upright position
e. Can occur in childhood

15. Peptic oesophagitis. All are False Except

a. Is effectively demonstrated by a barium swallow and meal


b. Is always associated with a hiatus hernia
c. Can be readily confirmed by oesophagoscopy
d. Is associated with the production of higher than normal amounts of gastric acid
e. Is a condition confined to adults
16. The successful medical management of pepticoesophagitis. All are true Except

a. Should include anti-cholinergic drugs


b. Includes weight reduction
c. Includes readily confirmed by oesophagoscopy
d. Is associated with the production of higher than normal amounts of gastric acid
e. Is a condition confined to adults

17. Cancer of the esophagus. All are true Except

a. Usually presents with intermittent dysphagia


b. Has its highest incidence in the fifth decade
c. Is reliably diagnosed by barium swallow
d. Should be assessed endoscopically when surgical resection is contemplated
e. May require Bronchoscopic assessment

18. Vomiting against a closed glottis. All are true Except

a. Cause a spontaneous rupture of the esophagus


b. Be followed by severe chest pain and circulatory collapse
c. Be effectively managed conservatively
d. Causes a laceration of the mucosa of the oesophagogastric junction
e. Cause gastrointestinal bleeding

19. The diagnosis of an inguinal hernia. All are false Except

a. In an infant often depends on the history given by its mother


b. In the adult is most easily made with the patient in the sitting position
c. Depends on the hernial sac or cough impulse being felt below the inguinal ligament
d. Is supported by the presence of a transilluminable scrotal swelling
e. Is more frequently male in females than in males

20. Inguinal hernia in children, All are true Except

a. Are usually of the indirect type


b. Are congenital in origin
c. Will regress spontaneously
d. Frequently obstruct
e. May be associated with a hydrocele

21. Intestinal hernia are related to, all are true Except

a. Wound infections
b. Anaemia and malnutrition
c. Obesity
d. The use of absorbable suture materials
e. Failure of surgical technique

22. A discharge from the umbilicus, all are true Except

a. May indicate a patent vitello-intestinal duct


b. May indicate an anomaly of the urachus
c. At the time of menstruation may indicate endometriosis
d. In the neonate is of no immediate clinical significance
e. Appearing for the fist time in the adult usually indicates poor personal hygiene

23. Car seat belts (lap diagonal) when properly adjusted, all are false Except

a. Prevent injuries to t he abdominal viscera


b. May cause small bowel injuries
c. Do not reduce the incidence of head injuries amongst car passengers involved in a road tract
accident
d. Protect the cervical spine during sudden acceleration
e. Often do no prevent thoracic injuries

24. Indication of a serious intra-abdominal injury in a comatose patient may be gained by, all are true
Except

a. Abdominal paracentesis
b. The observation of pattern bruising o n the abdominal wall
c. Falling haemoglobin values
d. The presence of diarrhea
e. A falling blood pressure

25. Penetrating wounds of the abdomen, all are false except

a. Can be adequately explored an their depth determined by a probe


b. Frequently result in acquired abdominal wall hernia
c. May be managed by careful observation, laparotomy being indicated only if signs of
peritonitis or blood loss occur
d. Demand urgent laparotomy
e. Can be treated conservatively if the weapon is less than 3 cm long

26. In the acute abdomen vomiting, all are false Except

a. Occurring soon after the onset of colicky pain often indicates pathology outside the
gastrointestinal tract
b. Of fluid containing no bile is characteristic of small bowel obstruction
c. Of faceulent fluid usually indicates a gastro colic fistula
d. Is a common early accompaniment of gastro duodenal perforation
e. Which is projectile and of large volume often indicates an obstruction at the cardia

27. Appendicitis is, all are false Except

a. More common in females


b. Distributed evenly throughout the world’s population
c. More likely to occur if the appendix is in the retrocaecal position
d. Commonly the result of appendicular obstruction
e. Frequently recurrent

28. Appendicectomy should be undertaken, all are true Except

a. After laparotomy reveals a diagnosis of mesenteric adenitis


b. Immediately in adult patients within appendix abscess
c. In patients with a gangrenous appendix
d. In patients with chronic appendicitis
e. It is inflammation is the result of neighbouring Crohn’s disease
29. Emergency treatment of a mechanical obstruction of the large bowel may include, all are true Except

a. Enemata
b. Radioloical examination by contrast enema
c. Exteriorization of the lesion with colostomy
d. A transverse colostomy
e. An ileo-transverse colostomy

30. Strangulation of the bowel, al are true Except

a. Commonly complicates closed loop obstruction


b. Is difficult to distinguish from simple intestinal obstruction
c. Is accompanied by bleeding into the affect bowel frequently causes peritonitis
d. May result in gram-negative septicaemia
e. Frequently causes peritonitis
31. Acute pancreatitis, all are true Except

a. Is frequently associated with gallstones


b. Often simulates perforated peptic ulcer in its presentation
c. Often presents with the signs of hypovolaemia
d. Can readily be distinguished from other causes of acute abdominal pain by the presence of a
raised serum amylase
e. Frequently has a raised concentration of urinary amylase

32. The treatment of acute pancreatitis, all are false Except

a. Should include a laparotomy in the majority of cases


b. Should routinely include the administration of calcium
c. Should routinely include the administration of antibiotics
d. Is more effective when peritoneal lavage is included

33. Acute superior mesenteric artery occlusion. All are true Except
a. Characteristically presents with sudden pain and tenderness of increasing intensity
b. Is frequently due to embolism
c. Is frequently accompanied by overt or occult blood loss in the stools
d. Frequently produces peritonitis
e. Can usually be diagnosed on plain abdominal radiographs

34. A ruptured Ectopic pregnancy, all are true Except

a. Usually occurs in the first month of pregnancy


b. Usually presents with severe lower abdominal pain
c. Frequently presents with Hypovolemic shock
d. Can usually be diagnosed by pelvic examination
e. May demonstrate a lower abdominal mass

35. Uncomplicated benign gastric ulcers, all are true Except

a. Occur most commonly on the greater curve of the stomach


b. Should initially be treated medically
c. Commonly reoccur after medical treatment
d. Require endoscopic surveillance during treatment
e. Should receive surgical treatment if healing has not occurred after a 8 weeks of medical
treatment

36. There is an associated of gastric cancer with, all are true Except

a. Achlorhydria of the stomach


b. Atrophic gastritis
c. Adenomatous gastric polyps
d. Duodenal ulceration
e. A history of gastric cancer in first degree relatives

37. Duodenal diverticulae, all are false Except

a. Most commonly arise from the concavity of the second part


b. Are frequently the site of infection, hemorrhage or ulceration
c. May be associated with obstructive jaundice
d. May follow a severe cause of pancreatitis
e. Are usually asymptomatic
38. The treatment of duodenal ulcers in young patients.
a. Can include antacids
b. Can include a strict dietary regime
c. Should include anticholinergic drugs
d. Demands the long term use of H12 blockers, such as cimetidine or ranitidine
e. Should be by early surgery

39. Chronic duodenal ulcers may be treated by m all are true Except

a. Histamine H12 receptor antagonists


b. Sub-total gastrectomy
c. Vagotomy and gastric drainage
d. Highly selective Vagotomy (HSV)
e. Non-steroidal anti-inflammatory drugs (NSAIDs)

40. Acute hemorrhage from the upper gastrointestinal tract, All are false Except

a. Requires an urgent barium meal examination


b. Requires an urgent gastsroscopy
c. Should be treated surgically as soon as h diagnosis is made
d. Indicates the presence of a chronic duodenal ulcer
e. Should be treated with H12 blockade

41. The post-gastrectomy syndrome may result in, all are true Except

a. Megalobastic anaemia
b. Steatorrhoea
c. Iron/deficiency anaemia
d. Osteoporosis
e. Renal Calculi

42. Patients with Crohn’s disease often present with, all true Except

a. Colicky abdominal pain


b. Constipation
c. Nutritional deficiencies
d. Rectal bleeding
e. Tender abdominal masses
43. Systemic manifestations of crohn’s disease include, all are true except

a. Arthralia
b. Finger clubbing
c. Growth retardation
d. Alopecia
e. Pyodema gangrenosum
44. Patients with the blind loop syndrome often, al are true except

a. Present with signs of mal-nutrition


b. Present a complication of Crohn’s disease
c. Respond to treatment with oral penicillin
d. Show a reduced excretion of vitamin B12 after an intramuscular injection of the vitamin
e. Can be treated successfully with surgery

45. Un-complicated diverticulae disease of the colon, al are false except

a. Can be most effectively treated with antispasmodics


b. Can be most effectively treated with a high residue diet
c. Frequently requires surgical resection of the sigmoid colon
d. May require long term antibiotic therapy
e. Is best monitored by colonoscopy

46. Ulcerative colitis, al are true except

a. Is more common in males than females


b. Appears most commonly between the ages of 20 and 30 years
c. Usually presents with abdominal discomfort and diarrhea
d. Can usually be diagnosed on sigmoidoscopic examination
e. May have an infective cause

47. Surgical treatment of ulcerative colitis, all are false Except

a. Is usually be subtotal collectomy


b. Is usually undertaken as an urgent measure
c. May reduce the risk of colonic cancer
d. Is often indicated for colitis confined to the recto sigmoid region
e. Is usually a desirable alternative to long term medical treatment

48. Long term medical management of total ulcerative colitis, all are false except

a. A Is usually indicted in patients with total colonic involvement


b. Is usually indicated in children
c. Is effective in the majority of patients
d. Includes intravenous pitressin in acute exacerbations
e. Frequently is curative

49. Crohn’s disease of the rectum, all are true Except

a. Is associated with an anal fissure or fistula in the majority of cases


b. May produce a diffuse granular proctitis
c. Is usually associated with ileal or colonic Crohn’s disease
d. Is characterized by long periods of remission
e. Is best treated conservatively
50. Ischemic colitis, all are true Except

a. Often presents with diarrhea


b. Often presents with rectal bleeding
c. Is commonly situated around the splenic flexure
d. Is often effectively manage by non-surgical means
e. May associated with abdominal aneurysms

51. Acute volvulus of the sigmoid colon. All are true Except

a. Most frequently occurs in Eastern Europe and Africa


b. Is relatively common amongst athletes
c. Usually produces abdominal distension which is most marked o n the left side
d. May be effectively treated without resorting to laparotomy
e. Can be readily diagnosed by abdominal radiographs

52. Hirschsprung’s disease, all are false except

a. Is the result of acquired aganglionosis of the large bowel


b. Is associated with small bowel dysmotility
c. Usually becomes evident in early adult life
d. Can usually be diagnosed on a barium enema
e. Can usually be manage by dietary means

53. Rectal cancer, all are false except

a. Is usually of squamous cell origin


b. Usually meastasizes by lymphatic spread
c. Frequently presents with faucal impaction
d. Often causes intestinal obstruction
e. Frequently requires a barium enema for accurate diagnosis

54. Rectal prolapse, all are true except

a. Is commonly seen in infants


b. Is commonly seen in the elderly
c. Is often associated with poor anal tone
d. May result from severe constipation
e. May in some cases be successfully treated by haemorrhoidectomy

55. Bile , all false except

a. Is secreted by the liver following the stimulus of the ingestion of food


b. Is secreted into the gastrointestinal tract following the stimulus of food ingestion
c. Is secreted at the rate of 200 to 400 ml per day
d. Secretion is reduced after cholecystectomy
e. Contains cholecystokinin
56. A patient with chronic liver failure frequently exhibits, all are true except

a. Tremors
b. Disorders of consciousness
c. Palmer erythema
d. A cold collapsed peripheral circulation
e. Gynaecomastia

57. The diagnosis of chronic liver disease may be helped by serum estimations of, all are true except

a. Alkaline phosphates
b. Creatine phosphokinase
c. Glutamic pyruvic transaminase (SGPT)
d. Clutamic oxaloacetic transaminase (SGOT)
e. Protein

58. What is not true of flail chest

a. Paradoxial chest movement


b. Right to left shunt with systemic desaturation
c. Can occur when multiple ribs are fractured at one place
d. May need endotracheal intubation and positive pressure ventilation

59. The “triangle: of safety for pleural drain is bounded by all Except

a. Anterior axillary fold


b. Posterior axillary fold
c. Mid axillary fold
d. Apex of axilla

60. A haemothorax should always be drained because

a. Empyema may supervene


b. Fibrothorax may occur
c. Relieves compression of contra-lateral lung
d. Helps to reduce bleeding from torn vessels
e. All of the above
…………………………………………
FRONTIER MEDICAL & DENTAL COLLEGE, ABBOTTABAD

FINAL PROFESSIONAL EXAMINATION 2014

SURGERY-B

PAPER-2

MCQs

Total Marks: 60 Time Allowed 60 minutes

Note: Encircle the correct answer

1. Sudden right sided chest pain in a post surgical patient with dyspnoea and loud P2 indicates:

a. Myocardial infarction
b. Pulmonary embolism
c. Acute pneumothorax
d. Congestive cardiac failure

2. In a patient of massive pulmonary embolism the treatment is

a. O2 administration
b. Plenty of IV fluids
c. Thrombolytic agents
d. Embolectomy using caval inflow occlusion but not CP bypass
e. All of the above

3. Chronic constrictive pericarditis presents with all Except

a. Raised CVP
b. Tender hepatomegaly
c. Ascites and oedema feet
d. Marked cardiomegaly

4. What is not trfue of Boerhaave’s syndrome


a. Occurs due to vomting with full stomach
b. There is longitudinal tear in loweroesophagus
c. Tear involves the mucosa and submucosa
d. Mediatinitis and left pleural effusion can occur

5. In Mallory-Weiss syndrome all are true Except

a. Follows severe vomiting


b. Vertical tear
c. Lower oesophageal tear
d. Mild bleeding

6. The carcinoma common in Berret’s oesophagus is

a. quamous cell cancer


b. Adenocarcinoma
c. Transitional cell carcinoma
d. All of the above
7. Symptoms of Para-oesophageal hernia include

a. Intermittent dysphagia
b. Angina like pain
c. Bouts of hiccoughs
d. All of the above

8. What is not true of achalasia

a. Increased incidence of carcinoma


b. Relief with local injection of botulinum toxin
c. Proximal esophagus contains few ganglion cells
d. All are true

9. What is not true of H-Pylori

a. Difficult to grow outside mucous layer of stomach


b. Hydrolyses urea to produce ammonia
c. Can infect normal duodenal mucosa
d. Causes antral gastritis
10. What is true of medical treatment in upper GI bleed

a. H2 antagonist and proton pump inhibitors do not reduce re-bleeding rate


b. Traxenamic acid reduces re-bleeding rate
c. Octreotide is not effective
d. All are true

11. Risk of gastric cancer increase with

a. H.Pylori infection
b. Gastric Polyp
c. Permicious anaemia
d. Gastric drainage surgery
e. Each of the above

12. In hypertrophic pyloric stenosis of adult the pathology is

a. Overgrowth of circular muscle


b. Overgrowth of longitudinal muscle
c. Fibrosis of myenteric plexus
d. None of the above
13. What is Helicobacter pyloric

a. Chalmydiae
b. Gram positive bacillus
c. Spirochaetal bacterium
d. None of the above
14. Ulcers at which location have greater chance of malignancy

a. Antrum
b. Prepyloric region
c. Fundus
d. Body

15. Which peptic ulcer complication manifests with vomiting, abdominal pain and rigidity
a. Pyloric obstruction
b. Perforation
c. Malignant transformation
d. Haemorrhage

16. Recognized complications of gastric surgery for peptic ulcer include

a. Stomal obstruction
b. Duodenal fistula
c. Oaralytic ileus
d. All of the above

17. In Mallory-Weiss syndrome the characteristic features are all except

a. Prolonged vomiting followed by haematemosis


b. Oesophagoscopy reveals tear at cardioesophageal junction
c. Tear is only mucosal and is longit udinal
d. Most cases need Laparotmy and suturing

18. Pyloric stenosis causes what type of metabolic abnormality

a. Metabolic acidosis
b. Metabolic alkalosis
c. Respiratory alkalosis
d. None of the above

19. Krukenberg’s tumor of ovary is common with which form of stomach cancer

a. Cauliflower growth
b. Scirrhous carcinoma
c. Colloid carcinoma
d. Polykpoid carcinoma

20. Variceal bleed secondary to portal vein thrombosis is best treated by


a. TIPSS
b. Sclerothrapy
c. Gastroesophageal devascularisation
d. Octreotide
21. Primary sclerosing cholangitis is associated with

a. A Rheumatoid arthritis
b. Ulcerative colitis
c. Mixed connective tissue disease
d. All of the above

22. Intra-hepatic biliary lakes with stone characterize

a. Hepatic polycystic disease


b. Primary biliary cirrhosis
c. Caroli’s disease
d. Cholangitis

23. What is not true about Hepatititis-A

a. Incubation period 3-5 weeks


b. Gammaglobulin may prevent or modify the disease
c. Fatality is nil
d. Steroids may be given for prolonged Cholestasis

24. The common symptoms and signs of cholangitis are all except

a. Biliary colic
b. Fever and chills
c. Jaundice
d. Obliterated biliary radicles in US

25. About hepatitis C, spot the wrong statement


a. 1% blood donors are HCV positive
b. Acute hepatitis C proceeds to cirrhosis in 20%
c. Chances of hepatic malignancy is negligible
d. Deteriorating HCV encephalopathy demands liver transplantation

26. A 35 year old woman with a history of previous right thyroidectomy for a benign thyroid nodule now
undergoes completion thyroidectomy for a suspicious thyroid mass. Severed hours postoperatively, she
develops progressive swelling under the incision, stridor, and difficulty breathing. Orotracheal intubation
is successful. Which of the following is the most appropriate next step?

a. Fiberoptic laryngoscopy to rule out bilateral vocal cord paralysis


b. Administration of intravenous calcium
c. Administration of broad-spectrum antibiotics and Debridement
d. Wound exploration
e. Administration of high dose steroids and antihistamines

27. A 55 year old woman presents with a 6 cm thyroid mass and palpable cervical lymphadenopathy.
Fine needle aspiration (FNA) of one of the lymph nodes demonstrates the presence of calcified clumps
of sloughed cells. Which of the following best describes the management of this thyroid disorder?

a. The patient should be screened for pancreatic endocrine neoplasms and hyper-calcemia
b. The patient should undergo total thyroidectomy with modified radical neck dissection
c. The patient should undergo total thyroidectomy with frozen section intra-operatively, with
modified radical neck dissection reserved for patients with extra-capsular invasion
d. The patient should undergo right thyroid lobectomy followed by iodine 131(131 1) therapy
e. The patient should undergo right thyroid lobectomy

28. A woman sustains an injury to her chest after striking the steering wheel of her automobile during a
collision. Which of the following statements concerning fat necrosis of the beast is true?

a. Most patients report a history of trauma


b. The lesion is usually non-tender and diffuse
c. It predisposes patients to the development of breast cancer
d. It is difficult to distinguish from breast cancer
e. Excision exacerbates the process

29. A 45 year old woman presents with hypertension, development of facial hair, and a 7 cm suprarenal
mass. Which of the following is the most likely diagnosis?
a. Myelolipoma
b. Cushing disease
c. Adrenocortical carcinoma
d. Pheochromocytoma
e. Carcinoid

30. A 40 year old woman notices a skin rash around her areola. There is an erosive area about the
nipple. There is tenderness, itching, and intermittent bleeding. Which of t he following statements is
true regarding this entity?

a. It usually precedes development of Paget’s disease of bone


b. It presents with nipple areolar eczematous changes
c. It does not involve axillary lymph nodes because it is a manifestation of intra-ductal carcinoma
only
d. It accounts for 10% to 15% of all newly diagnosed breast cancers
e. It is adequately treated with wide excision when it presents as a mass

31. A 52 year old woman is referred to you. She says that her doctor told her she had a condition called
Cushing. She has hypertension, obesity and new skin striae. True statements regarding Cushing disease
and Cushing Syndrome include which of the following?

a. Adrenocortical hyperplasia is the most common cause of Cushing disease


b. Overproduction of adrenocorticotropic hormone (ACTH) is pathognomonic of Cushing Syndrome
c. Clinical manifestations of Cushing disease and Cushing Syndrome are identical
d. Cushing syndrome is caused only by neoplasms of either the pituitary or adrenal glands
e. Cushing disease is incurable

32. Diagnosis of Hydated liver disease is by all except

a. US
b. CT scan
c. Needle aspiration
d. Casoni’s skin test

33. The most common complication of hepatic Hydatid disease is


a. Jaundice
b. Rupture into peritoneal cavity
c. Suppuration
d. Rupture into biliary channel

34. What is not true of hepatoadenomas

a. Common to women taking oral contraceptives


b. Mimic nodular hyperplasia of cirrhosis
c. Have high malignancy potential
d. Frequently multiple

35. To differentiate focal nodular hyperplasia from adenoma, the best study is

a. CT
b. MRI
c. Sulphur colloid scan
d. Rose Bengal scan

36. Normal liver function and impalpable liver but splenomegaly and ascites in child is due to an
obstruction which is

a. Prehepatic
b. Hepatic
c. Post hepatic
d. All of the above
37. Spot the wrong statement about gastro-oesophageal bleed

a. Bleeding may often be severe to cause collapse


b. Endoscopic sclerotherapy can arrest bleeding
c. Prophylactic sclerotherapy in G.E varices obviates chance of bleeding and prolongs survival
d. Tamponade by Shangtaken tube is effective
38. A 60 year old male patient with hepatitis C with a previous history of variceal bleeding is admitted to
the hospital with hematemesis. His blood pressure is 80/60 mm Hg, physical examination reveals
splenomegaly and ascites, and initial hematocrit is 35%. Prior to Endoscopy, which of the following is the
best initial management of the patient?

a. Administration of intravenous octreotide


b. Administration of a ß-blocker 0eg. Propranolol)
c. Measurement of prothrombin time and transfusion with cryoglobulin if elevated
d. Empiric transfusion of platelets given splenomegaly
e. Gastric and esophageal balloon tampondade (Sengstaken-Blakemor tube)

39. A 22 year old college student notices a bulge in his right groin. It is accentuated with coughing, but is
easily reducible. Which of the following hernias follows the path of the spermatic cord within the
cremaster muscle?

a. Femoral
b. Direct inguinal
c. Indirect inguinal
d. Spigelian
e. Interparietal

40. A 22 year old woman presents with a painful fluctuant mass in the midline between the gluteal folds.
She denies pain on rectal examination. Which of the following is the most likely diagnosis?

a. Pilonidal abscess
b. Perianal abscess
c. Perirectal abscess
d. Fistula-in-ano
e. Arial fissure

41. A 39 year old woman with no significant past medical history and whose only medication is oral
contraceptive pills (OCP) presents to the emergency room with right upper quadrant pain. CT scan
demonstrates a 6 cm hepatic adenoma in the right lobe of the liver. Which of the following describes the
definitive treatment of this lesion?

a. Cessation of oral contraceptives and serial CT scans


b. Intra-arterial embolization of the hepatic adenoma
c. Embolization of the right portal vein
d. Resection of the hepatic adenoma
e. Systemic chemotherapy

42. During an appendectomy for acute appendicitis, a 4 cm mass is found in the mid-portion of the
appendix. Frozen section reveals this lesion to be a carcinoid tumor. Which of the following statements
is true?

a. No further surgery is indicated


b. A right hemicolectomy should be performed
c. There is about a 50% chance that this patient will develop the Carcinoid syndrome
d. Carcinoid tumors arise from islet cells
e. Carcinoid syndrome can occur only in the presence of liver metastases

43. A 71 year old woman is diagnosed with small cell lung cancer. Which of the following statements
regarding small cell lung carcinoma is true?

a. It represents about 80% of all lung cancers


b. Most are located peripherally
c. It rarely spreads to mediastinal lymph nodes
d. It is slow growing and rarely metastasizes
e. Most are treated with chemotherapy and radiation instead of surgery
44. A previously healthy 20 year old male is admitted to the hospital with acute onset of left sided chest
pain. Electrocardiographic findings are normal, but CXR shows a 40% left pneumothorax. Appropriate
treatment consists of which of the following procedures?

a. Observation
b. Barium swallow
c. Thoracotomy
d. Tube thoracostomy
e. Thoracostomy and intubation

45. Kehr sing in splenic trauma refers to

a. Pain and hyperaesthesia in left shoulder


b. Pain and hyperaesthesia in right should
c. Bruising around left 10th and 11th ribs
d. Hiccup and haemoptysis o n leg elevation
46. Radiographic signs of splenic rupture include

a. Obliteration of psoas shadow


b. Indentation of gastgric air bubble
c. Elevation of left hemidiaphragm
d. All of the above

47. Splenomegaly along with lymphadenopathy characterize all the following Except

a. Hodgkin’s disease
b. Acute keukemia
c. Cirrhosis
d. Felty’s syndrome
48. Thrombocytopenia can be due to all except

a. Aplastic anaemia
b. DIC
c. Hypersplenism
d. Posts splenectomy

49. A young child having anaemia, and gallstones should be investigated for

a. Cystic fibrosis
b. Congenital spherocytosis
c. Malaria
d. Primary Sclerosing cholongitis

50. A 2 year old child with anaemia, hepatosplenomegaly and a large head with slanting eyes is most
likely to have

a. Cirrhosis
b. Spherocytosis
c. Thalassaemia
d. Gaucher’s disease
51. The primary investigation of coice in gallstone disease is

a. Plain x-ray
b. ECG
c. Ultrasongraphy
d. IV cholangiography

52. Differential diagnosis of biliary atresia includes all except

a. Choledochal cyst
b. Gilbert syndrome
c. Inspissated bile syndrome
d. Alfah1 antitrypsin deficiency

53. Which of the following is a possibility if symptoms persist after cholecystectomy

a. Stone in CBD
b. Long stump of cystic duct remnant
c. Damage to CBD from consequent stricture
d. Each of the above

54. Constituents of Saint’s triad are all except

a. Gallstones
b. Colonic diverticulosis
c. Hiatus hernia
d. Crohn’s disease
55. Charcot’s triad of cholangitis include all except

a. Upper abdominal pain


b. Jaundice intermittent persistent
c. Projectile effortless vomiting
d. Fever with rigor
56. Gallbladder painpersisting for more than 12 hours is due to

a. Stone colic
b. Cholecystitis
c. Perforation of gallbladder
d. Torsion of gallbladder

57. ERCP findings suggestive of chronic pancreatitis include

a. Dilated main duct with stones and stricture


b. Pancreatic cysts
c. Chain of lakes appearance
d. All of the above
58. Clinical manifestations of cystic fibrosis include all except

a. Mal-absorption
b. Obstructive lung disease
c. Salty child
d. Mental retardation

59. Pancreatic pseudo cyst is usually drained to

a. Abdominal wall
b. Stomach
c. Duodenum
d. Jejunum

60. The most common feature of acute pancreatitis is

a. Severe acute epigastric pain radiating to back


b. Jaundice
c. Cullen’s sign and Grey Turner’s sign
d. Abdominal guarding and loss of bowel sounds

..........................
FRONTIER MEDICAL & DENTAL COLLEGE, ABBOTTABAD

FINAL PROFESSIONAL EXAMINATION 2014

SURGERY-B

PAPER-3

MCQs

Total Marks: 60 Time Allowed 60 minutes

Note: Encircle the correct answer

1. A 55 years old male had a chest trauma after a mine collapse. He was brought to the emergency room
in a state of shock. Examination revealed BP of 80 mmHg, Pulse 110, shallow breathing with respirate of
35/min, X-ray chest showed fractures at both ends of ribs 307 and a diffuse opacity involving the right
lung. He was administered Oxygen through venti mask. ABGs showed PO2 of 50 mmHg. PO2 of 52 and
pH of 7.3. He was given I.V analgesia. The next step in the management of this patient is :

a. Oropharyngeal airway
b. Tracheostomy
c. Chest intubation
d. Immediate thoracotomy
e. Mechanical ventilation

2. Which of the following is TRUE regarding wound dehiscence?

a. Usually occurs before the fifth post-operative day


b. Frequently recurs after repair
c. Is often fatal
d. Is more common in Grid iron incision
e. Increased incidence in patients on steroid therapy

3. Which of the following is TRUE regarding aspiration pneumonia

a. Most commonly affects the left upper lobe


b. Is more frequent following elective surgery
c. Is more likely if cricoid pressure is used during intubation
d. Is aggravated by gastric acidity
e. Is less common in patients with reflux oesophagitis

4. Post-operative atelectasis is more common I n all of the following EXCEPT

a. Smokers
b. Patients undergoing lower abdominal surgery
c. Patients with acute intestinal obstruction
d. During first 24 post-operative hours
e. Over sedated patients
5. A 20 years young healthy man weighing 79 kg was involved in a road traffic accident and had multiple
lacerations all over the body and a compound fracture of right femur. His clothes were soaked with
blood. On examination his pulse was 130 per min, BP is 80/60 mm Hg, Respiratory rate 32/min and was
mentally confused. Based on these findings the approximate blood loss in this patient is :

a. Less than 500 ml


b. Less than 750 ml
c. 750 to 1500 ml
d. 1500 to 2000 ml
e. More than 2000 ml

6. The commonest causative organism for gas gangrene is

a. Anaerobic streptococci
b. Clostridium Welchii (Perfringens)
c. Clostridium difficle
d. Pseudomonas aeruginosa
e. Staphylococcal aureus

7. Which of the following is the most essential step for the prevention of gas gangrene in a
contaminated wound?

a. A administration of antibiotics
b. Fasciotomy
c. Amputation of the limb
d. Wound Debridement
e. Fluid and electrolytes replacement
8. Which of the following statements is true regarding Tetanus?

a. Tetanus toxin mainly affects the muscle fibres


b. Patient suffering from tetanus have episodes of carpo-pedal spasm
c. The organism causing tetanus produces purulent discharge
d. Tetanus causes death due to respiratory muscle spasm
e. Tetanus immunoglobin is injected after intra-dermal testing

9. The most objective, practical monitoring of adequacy of volume resuscitation I n cases of


hypovolaemic is done by

a. Serum potassium level


b. Serum creatinine level
c. Blood pressure
d. Hourly urine output
e. Arterial blood gases

10. All of the following are associated with Buerger’s disease (Thromboagnitis obliterans) EXCEPT

a. Arteritis
b. Superficial thrombophlebitis
c. Smoking
d. Male Predominance
e. Age 60 and above

11. A 14 years old girl had multiple lumps in the neck for the last three months. On examination she was
found to have multiple matted, non-tender cervical lymph nodes palpable o n left side of the neck.
There were two scar marks o n the right side of neck with past history of discharging sinuses. Bases o n
these findings the most probable cause of lymphadenopathy in this girl is:

a. Lymphoma
b. Syphilis
c. Tuberculosis
d. Infectious mononucleosis
e. Reactive hyperplasia
12. Ideal time to administer prophylactic antibiotic is

a. 24 hours before surgery


b. At the time of induction
c. 4 hours before surgery
d. Immediately after surgery
e. 1 hour after surgery

13. All the following are the risk factors to develop deep vein thrombosis EXCEPT

a. Smoking
b. Obesity
c. Malignancy
d. Antithrombin III deficiency
e. Factor VIII deficiency

14. A 35 year old teacher, a known patient of valvular heart disease, fell down in the washroom. After
sometime he notices that he could move the left lower limb with great difficulty. In the hospital his
examination revealed a pale looking left leg with no sensations or palpable pulses below the inguinal
ligament. Based on these findings the diagnosis in this patient is :

a. Acute arterial thrombosis


b. Acute venous obstruction
c. Acute arterial embolism
d. Acute slipped intervertebral disc
e. Fractured neck of femur

15. All the following are the common causes of postoperative fever EXCEPT

a. Respiratory tract infection


b. Acute gastritis
c. Wound infection
d. Urinary tract infection
e. Deep vein thrombosis
16. Which of the following is TGRUE regarding Barrett’s esopghagus?

a. Un-related to gastro-esophageal reflux


b. Best diagnosed using contrast radiology
c. Best treated by surgical resection
d. A pre-malignant condition
e. Associated with cardiac achalasia

17. All of the following are associated with Gastric carcinoma EXCEPT

a. Atrophic gastritis
b. Pernicious anaemia
c. Polya gastrectomy
d. Aspirin intake
e. Exposure to Nitroso compunds

18. Which of the following is a classical featdure of a perforated peptic ulcer?

a. Succussion splash
b. Cullen’s sign
c. kehr’s sSign
d. Fluid thrill
e. Obliteration of liver dullness

19. Which of the following statements is TRUE regarding gastric carcinoma?

a. It is common in strict vegetarians


b. It is linked to patients with type O-blood group
c. It often presents with a major gastrointestinal haemorrhage
d. It is usually associated with hyperchlorhydria
e. It metastasizes to ovaries in young females

20. Which of the following statements is TRUE regarding ascending Cholangitis?

a. It causes Charcot’s triad


b. Is caused by stone impaction in Hartmann’s pouch
c. Is treated by percutaneous drainage of biliary tree
d. Is cured by cholecystectomy
e. Is usually caused by Gram-positive organisms
21. Which one of the following statements is TRUE regarding patients with acute cholecystitis?
a. Gall stones are always present
b. Diagnosis is usually confirmed by an oral cholecystogram
c. Carcinoma gall bladder is a common complication
d. The presence of jaundice invariably implies a coexisting common bile duct stone
e. Complications are more likely if the patient is a diabetic

22. All of the following statements are True regarding common bile duct stones EXCEPT

a. Most are formed primarily in the common bile duct


b. Most are associated with an elevated serum alkaline phosphatise
c. Small stones are removed endoscopically
d. The gall bladder is usually not palpable
e. They are a common cause of acute pancreatitis

23. An elevated serum amylase is seen in all of the following EXCEPT

a. Acute mesenteric ischemia


b. Ruptured Ectopic pregnancy
c. Acute pancreatitis
d. Perforated peptic ulcer
e. Acute viral hepatitis

24. All of the following are true regarding necrotising pancreatitis EXCEPT

a. Complicates 10% of all episodes of acute pancreatitis


b. Is associated with a poor outcome
c. Is usually sterile
d. Is diagnosed by CT scan
e. Radical surgical Debridement is required

25. Which of the following is TGRUE regarding pancreatic Carcinoma?

a. They are associated with superficial thrombophlebitis


b. Cancer of the tail of the gland is treated by Whipple’s operation
c. Maturity onset diabetes is uncommon
d. Obstructive jaundice is infrequent
e. Periampullary cancers have the worst prognosis

26. All of the following are TRUE regarding injection Sclerotherapy for oesophageal varices EXCEPT

a. Is particularly useful in controlling gastric fundal varices


b. Can cause an oesophageal stricture
c. Banding is a better procedure than Sclerotherapy
d. Can be used in conjunction with balloon tamponade
e. Causes variceal thrombosis

27. Which of the following is TRUE regarding Splenic surgery?

a. After splenectomy the circulating platelet count falls


b. It is the first-line management for patients with idiopathic thrombocytopenic purpura
c. Isolated tears in the spleen can be repaired
d. A delayed Splenic rupture usually occurs within 48 hours
e. Post-splenectomy infection is usually caused by gram negative bacilli

28. Strangulating small bowel obstruction is a common feature in all of the following EXCEPT

a. Small bowel Volvulus


b. High small bowel obstruction
c. An incarcerated external hernia
d. Obstructed patients with constant severe abdominal pain
e. A closed loop obstruction

29. Which of the following is true regarding Crohn’s disease?

a. Rarely fistulates
b. Crypt abscesses are a characteristic feature
c. Does not involve colon
d. Is characterized by non-caseating epithelioid granulomas
e. Toxic mega colon is a common complication

30. All of the following statements regarding acute small bowel mesenteric ischemia are TRUE, EXCEPT
a. The abdominal pain is out of proportion to the physical findings
b. A history of atrial fibrillation is a common association
c. The inferior mesenteric artery is usually occluded
d. Arterial infarction is more commo0n than venous infarction
e. About 50% of patients die of acute event

31. Which of the following statements is TRUE regarding large bowel obstruction?

a. Caecal perforation is more likely if ileocaecal valve is competent


b. It is usually caused by Volvulus of colon
c. Vomiting is an early feature
d. Abdominal distension is unremarkable
e. The distended colon o n Plain X-Ray of abdomen shows concertina pattern
32. Which of the following statements TRUE regarding colorectal cancer?

a. Less than 10% of all cancers are in rectum and sigmoid colon
b. Low anterior resection is recommended for growth within 3 cm from anal verge
c. The 5 year survival in Duke C is 70%
d. Right-sided colonic carcinomas present usually with anaemia
e. Alteration in bowel habits is uncommon I n left-sided cancer

33. All of the following statements are TGRUE regarding Diverticular disease of colon EXCEPT

a. Colovesical fistula occurs in 5% of cases


b. Occasionally presents with massive rectal haemorrhage
c. Colonic resection and primary anastomosis is the treatment of choice for severe acute
diverticulitis
d. In 90% cases Sigmoid colon is involved
e. Diverticular disease is uncommon in Africans and Asians who consume high fibre diet

34. Ulcerative colitis is characterised by which of the following?

a. Mucosal skip lesions


b. Rectal sparing
c. Depletion of goblet cell mucin
d. Transmural inflammation
e. Non-caseating granulomas

35. Which of following statements is characteristically TRUE of ischiorectal abscesses?

a. They are usually situated above Levator ani


b. Usually associated with severe constitutional symptoms
c. They resolve with broad-spectrum antibiotics
d. They are common in patient with ulcerative colitis
e. Usually drained trans-rectally

36. Which of the following statements is TRUE regarding Pilonidal sinus?

a. Is usually congenital
b. Is best treated by prolonged antibiotic therapy
c. Affects women more than men
d. A tuft of loose hair is usually found in the sinus
e. Characteristically occurs over the nape of neck

37. A 25 year old married lady who missed her last period had sudden pain in right iliac fossa which later
on became generalized. On examination she is pale. Her pulse is 110/min, abdominal tenderness and
guarding. The most likely diagnosis is:

Perforated appendix

a. Ruptured tubal pregnancy


b. Amebic typhlitis
c. Ileal perforation
d. Perforated duodenal ulcer
38. A new born is brought to the hospital with history of inability to pass Meconium and abdominal
distension. Abdominal X-ray showed dilated colon upto the level of rectum. Rectal biopsy revealed
absence of ganglia in Auerbach’s plexus. The treatment of choice at this stage is:

a. Pull through sleeve of rectum (Soave’s operation)


b. Posterior pull through (Duhamel’s operation)
c. Loop Colostomy proximal to transitional zone
d. Anal pull through (Swenson’s proctocolectomy)
e. Rectal wash outs with tap water

39. A 7 year old child, living in Kachi Abaadi, is brought with recurrent abdominal pain for the last 6
months. Recently he had a severe attack of colicky abdominal pain, vomiting and absolute constipation.
Plain x-ray abdomen shows multiple fluid levels. Haemoglobin is 10gm/dl, TLC-9400/µl, DLC-polymorphs
75%, lymphocytes 15%, eosinophils 10%. The most probable cause of intestinal obstruction is :

a. Intussusception
b. Volvulus of small bowel
c. Roundworm bolus
d. Meckel’s diverticulum
e. Ladd’s band

40. Colles’ fracture is defined as fracture of distal radius in adults with

a. Dorsal angulation and fracture of ulnar styloid


b. Volar angulation and fracture of ulnar styloid
c. Dorsal angulation and intra-articular extension
d. Volar angulation and intra-articular extension
e. No displacement

41. A 35 year old lady is suffering form recurrent bouts of abdominal pain and mass formation with loud
gurgling sounds in the abdomen for the last 6 months. There is history of alternating diarrhoea and
constipation. Barium follow – through X-ray reveals deformity of caecum and stricture of the ileocaecal
regions. She also has discharging sinuses in the neck. The most probable diagnosis is:

a. Crohn’s disease
b. Ileocaecal tuberculosis
c. Carcinoma of caecum
d. Amoeboma of caecum
e. Ulcerative colitis

42. The appropriate treatment of acute osteomyelitis in a child after 72 hours of onset of the symptoms
is:

a. Drainage of pus and IV antibiotics


b. I.V antibiotics only
c. Drainage of pus and immobilization
d. Immobilization alone
e. Sequestrectomy
43. A 40 years old man had an accident and developed fracture of the left temporal bone. He was
brought to the emergency room where he had deterioration in the level of consciousness with right
hemiparesis and dilated left pupil. What is t he most likely diagnosis?

a. Acute subdural haematoma


b. Acute extradual haematoma
c. Cerebral concussion
d. Brain laceration
e. Cervical spine injury

44. The sac of a Richter’s hernia contains

a. A small loop of gut


b. Vermiform appendix
c. Fallopian tube
d. A part of the circumference of the gut
e. Greater omentum

45. Which of the following techniques of repair for inguinal hernia I n an elderly person results in the
lowest rate of recurrence?

a. Bassini’s
b. Lichtenstein’s
c. Darn
d. McEvedy’s
e. Lytle’s

46. A 45 year old lady who is a known case of ulcerative colitis of 10 years duration and is on oral
steroids is brought to emergency with acute abdominal pain and distension of 24 hours duration.
Physical examination revealed Temp 39 C, pulse 120/min, abdominal tenderness and absent bowel
sounds. Plain X-ray abdomen showed marked dilatation of colon. Total leukocyte count is 19000/µl. In
spite of fluid resuscitation and I.V antibiotics the patient is not responding. The most appropriate
treatment is:

a. Decompression colonoscopy
b. Caecostomy
c. Ileostomy
d. Total collectomy and ileostomy
e. Intravenous steroid therapy

47. Which of the following statements is TRUE regarding a 10 year old boy who presented with a 4-hour
history of pain, swelling and redness of one side of scrotum? There is no history of dysuria. His body
temperature is 37ºC.

a. He is likely to have acute epididymitis


b. He should have early surgical exploration to exclude testicular torsion
c. A diagnosis of testicular lymphoma should be considered
d. If the affected testis is infracted, the unaffected side should be left alone
e. He is likely to have mumps orchitis

48. For stage II disease of seminoma testis, the treatment is radical Orchiectomy followed by

a. Surveillance only
b. Radiation
c. Chemotherapy
d. Retroperitonal Lymph node dissection
e. Combined chemo and radiation
49. A 25 year old male is brought to the emergency room for evaluation after a high speed motor vehicle
accident. As a part of his secondary survey, the patient is found to have blood at the external urethral
meatus. What is the next appropriate manoeuvre required in this patient?

a. Foley catheter insertion


b. Urethrogram
c. IVU
d. Computed topographic scan
e. Diagnostic Peritoneal Lavage

50. A 2 year old male child is brought to the hospital with the complaints of difficult voiding and a weak
urinary stream. Physical examination reveals distended urinary bladder. Ultrasound shows bladder
thickening and trabeculation, bilateral hydro-ureter and hydronephrosis. Micturating cystogram showed
dilatation of proximal urethra. All of the following statements are true about the disease in this child
EXCEPT:

a. About 1/3rd of the children can be diagnosed by prenatal ultrasound


b. The prognosis is good if treated at early stage
c. Urinary tract infection is usually associated
d. Estimation of serum urea and creatinine is mandatory
e. The treatment of choice is repeated urethral dilatation
51. A fifty year old lady who had Choledochoduodenostomy for multiple calculi in a 4 cm dilated
common bile duct two years ago has recently developed jaundice. Lab results show S.Bilirubin 8 mg/dl,
S.alkaline phosphatise raised 2 times normal. Which of the following investigations will be most helpful
in arriving at the diagnosis?

a. Ultrasonography of abdomen

b. I.V cholangiography

c. ERCP

d. E-HIDA scan

e. CT scan

52. Regarding post-splenectomy sequelae in a child all of the following statements are true EXCEPT

a. Platelet count rises


b. The incidence of infections is highest in children under two years
c. The most common pathogen is Staph.Aureus
d. Life long chemoprophylaxis is required
e. Vaccination is required to prevent infections

53. A 50 years old patient who had an episode of acute pancreatitis 6 weeks ago complains of persistent
epigastric pain. Abdominal ultra-sonogram shows a 10 cm smooth cystic lesion situated in the lesser sac.
Treatment of choice in this patient is:

a. External Drainage
b. Cystogastrostomy
c. Pancreaticoduodenectomy
d. Resection of the cyst
e. Continued observation
54. A 35 year old man was brought in emergency with acute upper GI haemorrhage. Endoscopy showed
gastro-oesophageal varices. Portovenography showed splenic vein thrombosis with patent Portal vein.
The appropriate treatment would be:

a. TIPSS(trans jugular intrahepatic porto-systemic stent shunt)


b. End to side portocaval shunt
c. Splenectomy
d. Mesocaval shunt
e. Distal spleno-renal shunt (Warren shunt)
55. Which of the following procedures in Portal hypertension is followed by lowest incidence of Hepatic
Encephalopathy?

a. End to side port-caval shunt


b. Side to side porta-caval shunt
c. Proximal spleno-renal shunt
d. Distal spleno-renal shunt (Warren shunt)
e. Mesocaval shunt

56. A blood stained nipple discharge in a female is usually associated with

a. Mammary duct ectasia


b. An intra-ductal papilloma
c. A Fibroadenoma
d. Fibrocystic disease of breast
e. Fat necrosis of breast

57. A 45 year old lady has a 6.5 cm hard mass in right breast. On examination the overlying skin is red
and has a peau-de-orange appearance. The Axillary nodes are enlarge and matted together. FNAC shows
anaplastic carcinoma of breast. Tumour is ER/PR negative. X-Ray chest, bone scan and ultrasound
abdomen are normal. The most appropriate treatment in this patient is:

a. Modified radical mastectomy


b. Chemotherapy followed by mastectomy
c. Mastectomy followed by Radiotherapy
d. Mastectomy followed by Chemotherapy
e. Mastectomy followed by anti-oestrogen drugs

58. In Papillary thyroid carcinoma all of the following statements are true EXCEPT

a. The tumour spreads mainly to bones


b. It is hormone dependent
c. A thyroid isotope scan is most likely to show a cold area
d. Radio-iodine is the treatment of choice for secondary deposits
e. Cervical lymphadenopathy is common

59. A 58 year old female who had partial gastrectomy for duodenal ulcer 12 years ago presents to you
with generalized bone pains and bone tenderness. Radiological exam of pelvis shows pseudo-fractures.
Repeated serum calcium levels are normal. These symptoms are due to which of the following?

a. Osteomalacia
b. Vitamin B12 deficiency
c. Bile salt malabsorption
d. Hyperparathyroidism
e. Small stomach syndrome

60. A 50 years old man is admitted with Haematemesis. Upper GI Endoscopy reveals a bleeding vessel in
an ulcer in the posterior wall of first part of duodenum. The artery which needs to be under run for
controlling bleeding is:

a. Right gastric artery


b. Right gastro-epiploic artery
c. Superior pancreatico duodenal artery
d. Gastro-duodenal artery
e. Superior mesenteric artery

Das könnte Ihnen auch gefallen